Вы находитесь на странице: 1из 140

NOTES

Geography Proper
Generally, geographers before the 70s were concerned with delineating and depicting spaces (ie
cartography, or map-making). The job of geographers was only to make maps and describe how
tall stuff was and where it is. Then, we got satellites, which made geography obsolete. So
geographers decided to try to save their jobs by inventing critical geography - essentially they
transitioned from writing about what the spaces are (ie where borders are) to writing about how
we define spaces (ie what predispositions we approach them with [Afghanistan and Pakistan are
"war-torn countries" or "war zones"]).
A great example of this is Cuba - think of it right now. What did you think of? Probably an island
isolated in the middle of the ocean somewhere, right? Turns out, Cuba is only 90 miles (that's
about half the width of Ohio) from the coast of Florida. But the way that we talk about Cuba as
an island or an isolated country or economically isolated shapes our perception of Cuba as
somewhere locked off from the rest of the world by ocean. In other words, those 90 miles are
some of the longest miles in the entire world.
So, essentially, we're looking for messed up ways that we perceive Mexico/Cuba/Venezuela, and
critiquing them.

Framework Notes
Because geographers' jobs depend on it, there are amazing framework articles - most
geographers write that a prior condition to working in a space is the way we define it (ie the only
reason we act the way we do is because we define spaces solely in terms of their economic value).
This makes for compelling arguments about the role of the judge - that he/she should be a
critical geographer, which means that some things (the way we map our policies onto the world)
are important, and some things aren't (for example, here's the title of an article written by a
geographer: "Against dialogue: Why being critical means taking sides rather than learning how
to play the policy research game").

Ways We Use Borders
1. Exclude - reference Cuba example above.

2. Regulate the Interior - we include to exclude better. For example, take Cuba again: we initially
thought that we could Americanize (ie anti-Communize) them by excluding them from
economic engagement. Now, however, we realize that that totally failed - they just hate us more.
So, we engage in neoliberal (deregulating) policies (ie economic engagement) to include them,
and Americanize them that way.

3. Police the Border - ie Mexico. We construct Mexico as a lawless nation of dirty immigrants
that "took 'er jerbs" to justify militarizing the border. The Conway cards are an excellent
example.

4. Instrumentalize - ie Venezuela's oil supply. We define Venezuela as a space that has a flow of
oil directed toward us (that is, make Venezuela a place that doesn't send oil to Nicaragua)

Links
Cuba
1. Agamben-y stuff - regulate the interior, described above.
2. Tropicality - there are excellent articles about the way that we define nations as "tropical"
nations and why that's messed up.
3. Travel - interesting concept discussed in geography, haven't thought of a real argument for it,
but will probably have to do something with how we define the home space and take short
excursions into the foreign space.
4. Island metaphor - lots of good articles in the context of why this is bad.

Mexico
1. Border - Conway cards, above.
2. Instrumentalization - define them as a space that exports to America so we can get their oils.
3. Environment
4. Making Mexico 51st state

Venezuela
1. Instrumentalization - same as description above and Mexico aff, but specifically in the context
of the Petrocaribe deal, which is basically a deal where Venezuela says "screw you US, we're
gonna sell oil to our commie friends for way less than the world oil price" (funny story, Chavez
once sold one ton of oil for 16,000 pairs of pants).


TOP SHELF

1NCS
GENERIC 1NC

Economic engagement is an imperialist tool used to forward US geopolitical
dominance economic influence perpetuates the North/South geographical divide
which makes war inevitable
Jones, Jones, and Woods, 04 (Martin Jones* - PhD in Human Geography from the University of Manchester, Rhys
Jones; Professor of Human Geography at the University
of Wales Aberystwyth** - Professor in Human Geography @ the University of Wales Aberystwyth, Michael Woods*** - PhD in
Human Geography from Bristol University; Professor of Human Geography and Director of the Institute of Geography and Earth
Sciences @ the University of Wales Aberystwyth, 2004, AN INTRODUCTION TO POLITICAL GEOGRAPHY Space, place and
politics, http://118.97.161.124/perpus-
fkip/Perpustakaan/Geography/Geografi%20manusia/Pengantar%20Geografi%20Politik.pdf) MD
Political domination can take on many forms. At its most basic and uncompromising, it is based on military relationships between
two or more parties. Much of the rationale behind the proliferation of nuclear weapons during the
Cold War, for instance, was based upon the West and the Easts need to secure strategic military
and, therefore, political advantage over their enemies. This became the main justication for the
global political and military face-off between East and West that characterised the international
relations of the Cold War. A more recent example has been the nuclear stand-off between India and Pakistan over the disputed
province of Kashmir (Dodds 2000: 1036). Once again, overt displays of the military might of the two countries have been used as
a means of securing strategic, military and political advantage within the region. Political forms of geopolitical
domination can also occur in more subtle and hidden ways. A good instance of this is the
persistent military inuence of the United States in neighbouring countries in the Caribbean,
Central and South America (see Dodds 2000: 57). The most infamous examples of these more covert
efforts by the United States to influence the internal politics of other independent states have
been in Guatemala, Nicaragua and Cuba. These latter examples also begin to demonstrate the
strong connections between political and economic aspects of geopolitical strategy, where
political interference is accompanied by various forms of financial aid. A key method of
securing geopolitical inuence and dominance in recent years has been the nancial and
technological aid offered by dominant countries to other, needy countries. In many ways, if military
might represents the stick of international relations, then nancial aid is the carrot. Numerous examples exist to demonstrate
the role of economic influence in shaping international geopolitical relations. In the period after the Iraqi invasion of Kuwait in
1990, for instance, there was much debate in the international community concerning the best way to secure the freedom of the
latter. Much of the political shenanigans of the period took place in the corridors of the United Nations in New York. The famous
journalist John Pilger (1992) has noted how the United States tried to use its economic muscle as a way of
securing the support of other states for its plan to mount an invasion of Kuwait and Iraq. In this
respect, its main efforts were directed towards the non-permanent members of the Security Council of the United Nations, which,
at that time, included one of the poorest states in the world, Yemen. It is a little-known fact that Yemen voted not to support an
invasion of the Middle East by American led UN forces. In the immediate aftermath of the vote, it is alleged by Pilger (1992), the
Yemeni ambassador to the United Nations was informed by his US counterpart that that was the most costly decision he had ever
made. In the following weeks, $70 million of proposed US aid to Yemen was cancelled, the World Bank and the International
Monetary Fund began to question the economic practices of the Yemeni state and 800,000 Yemeni workers were expelled from
Saudi Arabia. As Dodds (2000) has argued, occurrences such as these are part of a broader range of
economic strategies that help certain Northern states to achieve geopolitical dominance over
Southern countries. The influence of industrialised countries over institutions such as the World Bank, the International
Monetary Fund and the World Trade Organisation has been particularly important. It has helped to generate an additional layer
of compliance within international relations. The best example of this process is the so-called structural adjustment programmes
of the World Bank, which seek to constrain the range of economic and political policies that can be pursued by less industrialised
countries (Dodds 2000: 17; see also Krasner 2001: 289). The criticism levelled at these programmes is that they reify a
particularly industrialised model of development on southern states and, as such, represent a new form of
informal imperialism by northern states. In many ways, these examples illustrate the strong
connections between geopolitics and the broader international political economy (see Agnew and Corbridge 1995).

This makes imperialistic violence, war and destruction inevitable Latin America
becomes a playground for the elite to commit violence
Grandin 06 (Empire's Workshop: Latin America, the United States, and the Rise of the New Imperialism, Greg Grandin,
Macmillan, May 2, 2006 BRW)
The ARGENTINE WRITER Jorge Luis Borges once remarked that the lack of camels in the Koran proves its Middle Eastern
provenance: only a native author, he explained, could have so taken the animal for granted as not to mention it. Perhaps a similar
familiarity explains the absence of Latin America in recent discussions about the United States and its empire. Though Latin
America has played an indispensable role in the rise of the United States to global power, it
elicits little curiosity from its neighbor to the north. "Latin America doesn't matter, Richard Nixon advised a
young Donald Rumsfeld, who was casting about for career opportunities. Long as weve been in it, people dont give one damn
about Latin America.' Likewise today. In their search for historical precedents for our current imperial moment, intellectuals
invoke postwar reconstructions of Germany and japan, ancient Rome and nineteenth-century
Britain but consistently ignore the one place where the United States has projected its influence
for more than two centuries. "People dont give one shit" about the place, Nixon said.: Vi/ere it not
for Borgess insight, this studied indifference to Latin America would seem ironic, for the region has long served as a workshop of
empire, the place where the United States elaborated tactics of extraterritorial administration and acquired its conception of itself as
an empire like no other before it. The Western hemisphere was to be the staging ground for a new empire for liberty," a phrase used
by Thomas Jefferson specifically in reference to Spanish Florida and Cuba. Unlike European empires, ours was supposed to entail a
concert of equal, sovereign democratic American republics, with shared interests and values, led but not dominated by the United
Statesa conception of empire that remains Washingtons guiding vision. The same direction of influence is evident in any number
of examples. The United Statess engagement with the developing world after World War II, for
instance, is often viewed as an extension of its postwar policies in Europe and japan, yet that view has it exactly backwards.
Washingtons first attempts, in fact, to restructure another countrys economy took place in the developing
worldin Mexico in the years after the American Civil War and in Cuba following the Spanish-American War. We should do for
Europe on a large seale, remarked the U.S. ambassador to England in 1914, "essentially what we did for Cuba on a small scale and
thereby usher in a new era of human history. Likewise, most discussions of George W. Bushs foreign policy focus
on the supposed innovation of a small group of neoconservative intellectuals in asserting the
right to unilateral preemptive military action both to defend national security and to advance
American ideals. But neither the neocons dire view of a crisis-ridden world that justifies the use
of unilateral and brutal American military power nor their utopian vision of the same world
made whole and happy by that power is new. Both have been fully in operation in Washingtons approach to Latin
America for over a century. The history of the United States in Latin America is cluttered with
preemptive" interventions that even the most stalwart champions of U.S. hegemony have
trouble defending. From the mid-nineteenth to the early twentieth century, the U.S. military sharpened its
lighting skills and developed its modernday organizational structure largely in constant conflict
with Latin Americain its drive west when it occupied Mexico in the midnineteenth century aml took more than half of that
countrys national territory. And in its push south: by 1930, Washington had sent gunboats into Latin American ports over six
thousand times, invaded Cuba, Mexico (again), Guatemala, and Honduras, fought protracted guerrilla wars in the Dominican
Republic, Nicaragua, and Haiti, annexed Puerto Rico, and taken a piece of Colombia to create both the Panamanian nation and the
Panama Canal. For their part, American corporations and financial houses came to dominate the
economies of Mexico, the Caribbean, and Central America, as well as large parts of South
America, apprenticing themselves in overseas expansion before they headed elsewhere, to Asia,
Africa, and Europe. Yet Latin America did more than serve as a staging ground for the United Statess
early push toward empire. The region provided a school where foreign policy officials and
intellectuals could learn to apply what political scientists like to call soft powerthat is, the
spread of Americas authority through nonnilitary means, through commerce, cultural exchange,
and multilateral cooperation} At first, the United States proved a reluctant student. It took decades of mounting Latin
American anti-imperialist resistance, including armed resistance, to force Washington to abandon its militarism. But abandon it it
finally did, at least for a short time. In the early 1930s, Franklin D. Roosevelt promised that henceforth the United States would be a
"good neighbor," that it would recognize the absolute sovereignty of individual nations, renounce its right to engage in unilateral
interventions, and make concessions to economic nationalists. Rather than weaken U.S. influence in the Western
Hemisphere, this newfound moderation in fact institutionalized Washingtons authority,
drawing Latin .American republics tighter into its political, economic, and cultural orbit through
a series of multilateral treaties and regional organizations. The Good Neighbor policy was the model for the
European and Asian alliance system, providing a blueprint for Americas empire by invitation, as one historian famously described
Washingtons rise to unprecedented heights of world power} But even as Washington was working out the contours of its kinder,
gentler empire in postwar Western Europe and japan, back in the birthplace of American soft power it was rearming. Latin
America has once again became a school where the United States studied how to
execute imperial violence through proxies. After World War II, in the name of containing Communism,
the United States, mostly through the actions of local allies, executed or encouraged coups in, among other places, Guatemala,
Brazil, Chile, Uruguay, and Argentina and patronized a brutal mercenary war in Nicaragua. Latin America became a
laboratory for counter insurgency, as military officials and covert operators applied insights learned in the re-gion to
Southeast Asia, Africa, and the Middle East. By the end of the Cold War, Latin American security forces trained, funded, equipped,
and incited by Washington had executed a reign of bloody terrorhundreds of thousands killed, an
equal number tortured, millions driven into exilefrom which the region has yet to fully
recover. This reign of terror has had consequences more far-reaching than the damage done to
Latin America itself, for it was this rehabilitation of hard power that directly influenced
Americas latest episode of imperial overreach in the wake of 9/1 1. It is often noted in passing
that a number of the current administrations officials, advisers, and hangers-on are veterans of Ronald
Reagans Central American policy in the 1980s, which included the patronage of anti-Communist governments in El Salvador and
Guatemala and anti-Communist insurgents in Nicaragua. The list includes Elliott Abrams, Bushs current deputy national security
adviser in charge of promoting democracy throughout the world; john Negroponte, former U.N. ambassador, envoy to Iraq, and now
intelligence czar; Otto Reich, secretary of state for the Western Hemisphere during Bushs first term; and Robert Kagan, an
ardent advocate of U.S. global hegemony. john Poindexter, convicted of lying to Congress, conspiracy, and destroying
evidence in the IranContra scandal during his tenure as Reagans national security adviser, was appointed by Rumsfeld to oversee
the Pentagons stillborn Total Information Awareness program. john Bolton, ambassador to the United Nations and an arch-
unilateralist, served as Reagans point man in the justice Department to stonewall investigations into Iran-Contra.; Yet the links
between the current Bush administrations revolution in foreign policy and Reagans hard line in Central America are even more
profound than the simple recycling of personnel. It was Central America, and Latin America more broadly,
where an insurgent New Right first coalesced, as conservative activists used the region to
respond to the crisis of the 1970s, a crisis provoked not only by Americas defeat in Vietnam but
by a deep economic recession and a culture of skeptical antimilitarism and political dissent that
spread in the wars wake. Indeed, Reagans Central American wars can best be understood as a dress rehearsal for what is
going on now in the Middle East. It was in these wars where the coalition made up of neoconservatives,
Christian evangelicals, free marketers, and nationalists that today stands behind George W. Bushs
expansive foreign policy first came together. There they had near free rein to bring the full power of the United
States against a much weaker enemy in order to exorcise the ghost of Vietnamand, in so doing, begin the transformation of
Americas foreign policy and domestic culture. A critical element of that transformation entailed shifting the rationale of American
diplomacy away from containment to rollback, from one primarily justified in terms of national defense to one charged with
advancing what Bush likes to call a global democratic revolution. The domestic fight over how to respond to
revolutionary nationalism in Central America allowed conservative ideologues to remoralize
both American diplomacy and capitalism, to counteract the cynicism that had seeped into both
popular culture and the political establishment regarding the deployment of U.S. power in the
world. Thus they pushed the Republican Party away from its foreign policy pragmatism to the idealism that now defines the war
on terror as a world crusade of free-market nation building. At the same time, the conflicts in Nicaragua, El Salvador, and
Guatemala allowed New Right militarists to find ways to bypass the restrictions enacted by Congress and the courts in the wake of
Vietnam that limited the executive branchs ability to fight wars, conduct covert operations, and carry out domestic surveillance of
political activists. The Reagan White House perfected new techniques to manipulate the media, Congress, and public opinion while
at the same time re empowering domestic law enforcement agencies to monitor and harass political dissidents. These techniques, as
we shall see, prefigured initiatives now found in the PR campaign to build support for the war in Iraq and in the Patriot Act,
reinvigorating the national security state in ways that resonate to this day. The Central American wars also provided the New
Christian Right its first extensive experience in foreign affairs, as the White House mobilized evangelical activists in order to
neutralize domestic opponents of a belligerent foreign policy. It was here where New Right Christian theologians
first joined with secular nationalists to elaborate an ethical justification for a rejuvenated
militarism. In other words, it was in Central America where the Republican Party first combined the
three elements that give todays imperialism its moral force: punitive idealism, free-market
absolutism, and right-wing Christian mobilization. The first justified a belligerent diplomacy not just for the sake
of national security but to advance freedom. The second sanctified property rights and the unencumbered free market as the moral
core of the freedom it was Americas duty to export. The third backed up these ideals with social power, as the Republican Party
learned how to channel the passions of its evangelical base into the international arena. 'lb focus, therefore, exclusively on
neoconservative intellectuals, as much of the commentary attempting to identify the origins of the new imperialism does, deflects
attention away from the long history of American expansion. The intellectual architects of the Bush Doctrine are but part of a larger
resurgence of nationalist militarism, serving as the ideologues of an American revanchism fired by a lethal combination of
humiliation in Vietnam and vindication in the Cold War, of which Central America was the tragic endgame.

The alternative is to reject the 1AC in order to politicize the affirmatives
conception of geography discourse analysis solves
Jones, Jones, and Woods, 04 (Martin Jones* - PhD in Human Geography from the University of Manchester, Rhys
Jones; Professor of Human Geography at the University
of Wales Aberystwyth** - Professor in Human Geography @ the University of Wales Aberystwyth, Michael Woods*** - PhD in
Human Geography from Bristol University; Professor of Human Geography and Director of the Institute of Geography and Earth
Sciences @ the University of Wales Aberystwyth, 2004, AN INTRODUCTION TO POLITICAL GEOGRAPHY Space, place and
politics, http://118.97.161.124/perpus-
fkip/Perpustakaan/Geography/Geografi%20manusia/Pengantar%20Geografi%20Politik.pdf) MD
Far more influential have been two conceptual developments which served to further politicise
the outlook of human geography as a whole. The rst of these was the so-called cultural turn of the late 1980s
and 1990s which promoted a new understanding of culture as the product of discourses through which people signify their identity
and experiences and which are constantly contested and renegotiated (see Jackson 1989; Mitchell 2000). Consequently, issues
of power and resistance were positioned as central to the analysis of cultural geographies,
generating signicant clusters of research on questions of identity and place, including national
identity and citizenship; conflict and contestation between cultural discourses; geographies of
resistance; the role of landscape in conveying and challenging power; and micro-geographies
of politics, including investigation of the body as a site of oppression and resistance (see for example Pile and Keith 1997;
Sharp et al. 2000). These themes are discussed further in Chapters 5, 7 and 8. Moreover, the new cultural geography
drew on the conceptual writings of post-structuralist thinkers such as Michel Foucault, Jacques Derrida,
Gilles Deleuze and Flix Guattari, and postcolonial theorists such as Homi Bhabha, for whom the relation of power
and space was a key concern (see Box 1.3). A number of different strands of post-structuralist thought have been
introduced into political geography, including ideas about difference in research on the cultural politics of identity and the use of
Derridas method of deconstruction in critical geopolitics (see below). However, it is the work of Michel Foucault that has arguably
had the greatest influence in political geography, in particular through the development and application of two key concepts. The
rst of these is discourse, which Foucault redened as referring to the ensemble of social practices through
which the world is made meaningful but which are also dynamic and contested (Box 1.4). In books such as The Order
of Things (1973 [1966]) and The Archaeology of Knowledge (1974 [1969]) Foucault examined the articulation of discursive
practices and thus established precedents as to how discourses might be analysed. These ideas have been fundamental to the
development of geographical work on cultural politics and of critical geopolitics, as well as to the development of
discourse analysis as a methodological approach which is now widely used across political
geography. The second key concept is governmentality, by which Foucault refers to the means by which
government renders society governable. Governmentality is essentially about the use of particular apparatuses of
knowledge and has been employed in recent years in work on the state and citizenship (see Chapter 8). A signicant
aspect of both discourse analysis and governmentality is the potential they allow
for exploration of the incorporation of space itself as a tool in the exercise of
power. Much of Foucaults writing was concerned with power, but he rejected conventional notions of power as a property
that is possessed, focusing instead on how power is exercised and how it circulates through society. Foucault stated that
space is fundamental in any exercise of power (Rabinow 1984: 252), and this principle underlies much of his
work on disciplinary power. His best known illustration of this is his discussion of Jeremy Benthams panopticon (Foucault 1977:
ch. 3). The panopticon was a proposal for an ideal prison, the spatial arrangement of which would effectively force prisoners to
discipline themselves. The panopticon would be built in a circular arrangement with all the cells facing a central observation tower.
The circle meant that prisoners could not see or communicate with each other, but also by means of backlighting from a small
external window it allowed prisoners to be constantly visible via a large internal window from the observation tower, whose own
windows had blinds to prevent prisoners seeing in. The prisoners could not know whether they were being watched at any
particular time, but had to presume that they were under constant surveillance and therefore act within the rules. As Foucault
describes, the major effect of the Panopticon [is] to induce in the inmate a state of conscious and permanent visibility that assures
the automatic functioning of power. So to arrange things that the surveillance is permanent in its effects, even if it is
discontinuous in its action; that the perfection of power should tend to render its actual exercise unnecessary; that this
architectural apparatus should be a machine for creating and sustaining a power relation independent of the person who exercises
it. (Foucault 1979: 201)



LINKS

GENERIC



1NC GENERIC

Economic engagement is an imperialist tool used to forward US geopolitical
dominance economic influence perpetuates North/South warfare
Jones, Jones, and Woods, 04 (Martin Jones* - PhD in Human Geography from the University of Manchester, Rhys
Jones; Professor of Human Geography at the University
of Wales Aberystwyth** - Professor in Human Geography @ the University of Wales Aberystwyth, Michael Woods*** - PhD in
Human Geography from Bristol University; Professor of Human Geography and Director of the Institute of Geography and Earth
Sciences @ the University of Wales Aberystwyth, 2004, AN INTRODUCTION TO POLITICAL GEOGRAPHY Space, place and
politics, http://118.97.161.124/perpus-
fkip/Perpustakaan/Geography/Geografi%20manusia/Pengantar%20Geografi%20Politik.pdf) MD
Political domination can take on many forms. At its most basic and uncompromising, it is based on military relationships between
two or more parties. Much of the rationale behind the proliferation of nuclear weapons during the
Cold War, for instance, was based upon the West and the Easts need to secure strategic military
and, therefore, political advantage over their enemies. This became the main justication for the
global political and military face-off between East and West that characterised the international
relations of the Cold War. A more recent example has been the nuclear stand-off between India and Pakistan over the disputed
province of Kashmir (Dodds 2000: 1036). Once again, overt displays of the military might of the two countries have been used as
a means of securing strategic, military and political advantage within the region. Political forms of geopolitical
domination can also occur in more subtle and hidden ways. A good instance of this is the
persistent military inuence of the United States in neighbouring countries in the Caribbean,
Central and South America (see Dodds 2000: 57). The most infamous examples of these more covert
efforts by the United States to influence the internal politics of other independent states have
been in Guatemala, Nicaragua and Cuba. These latter examples also begin to demonstrate the
strong connections between political and economic aspects of geopolitical strategy, where
political interference is accompanied by various forms of financial aid. A key method of
securing geopolitical inuence and dominance in recent years has been the nancial and
technological aid offered by dominant countries to other, needy countries. In many ways, if military
might represents the stick of international relations, then nancial aid is the carrot. Numerous examples exist to demonstrate
the role of economic influence in shaping international geopolitical relations. In the period after the Iraqi invasion of Kuwait in
1990, for instance, there was much debate in the international community concerning the best way to secure the freedom of the
latter. Much of the political shenanigans of the period took place in the corridors of the United Nations in New York. The famous
journalist John Pilger (1992) has noted how the United States tried to use its economic muscle as a way of
securing the support of other states for its plan to mount an invasion of Kuwait and Iraq. In this
respect, its main efforts were directed towards the non-permanent members of the Security Council of the United Nations, which,
at that time, included one of the poorest states in the world, Yemen. It is a little-known fact that Yemen voted not to support an
invasion of the Middle East by American led UN forces. In the immediate aftermath of the vote, it is alleged by Pilger (1992), the
Yemeni ambassador to the United Nations was informed by his US counterpart that that was the most costly decision he had ever
made. In the following weeks, $70 million of proposed US aid to Yemen was cancelled, the World Bank and the International
Monetary Fund began to question the economic practices of the Yemeni state and 800,000 Yemeni workers were expelled from
Saudi Arabia. As Dodds (2000) has argued, occurrences such as these are part of a broader range of
economic strategies that help certain Northern states to achieve geopolitical dominance over
Southern countries. The influence of industrialised countries over institutions such as the World Bank, the International
Monetary Fund and the World Trade Organisation has been particularly important. It has helped to generate an additional layer
of compliance within international relations. The best example of this process is the so-called structural adjustment programmes
of the World Bank, which seek to constrain the range of economic and political policies that can be pursued by less industrialised
countries (Dodds 2000: 17; see also Krasner 2001: 289). The criticism levelled at these programmes is that they reify a
particularly industrialised model of development on southern states and, as such, represent a new form of
informal imperialism by northern states. In many ways, these examples illustrate the strong
connections between geopolitics and the broader international political economy (see Agnew and Corbridge 1995).



2NC GENERIC

The aff remakes the world as flat economic engagement is an overmapping of the
Global South as irresponsible; <removing the Cuban embargo/engaging in trade
with Venezuela/extracting resources from Mexico> is the Norths attempt to
deterritorialize the political geography of the South
Sparke, no date professor of geography at the Jackson School of International Studies (Matthew Sparke, no date [after
2006], Everywhere But Always Somewhere: Critical Geographies of the Global South,
http://faculty.washington.edu/sparke/Everywhere.pdf)//CC
The truth of course isthat Friedman had been imagining that the world wasflat
long before the trip to Infosys. In fact his earlier best-seller guide to globalization articulated exactly
the same imaginative geography with most of the same clichs about the level playing field. Globalization,
Friedman explained in The Lexus and the Olive Tree, was creating a single, integrated, open plain. Today that
plain grows wider, faster and more open every day, as more walls get blown down and more
countries get absorbed. And thats why today there is no more First World, Second World or Third
World. Theresjust the Fast World the world ofthe wide open plain and the Slow World the world of
those who either fall by the way-side or choose to live away from the plain in some artificially
walled-off valley oftheir own (1999: 41). In this version the Global South wasterritorialized asthe
Slow World, the world ofthe olive treesthat was contrasted with the fast world of the Lexusin the
old bookstitle. For Friedman it was a world brought to imaginary life by looking through an orientalist eyethat he developed when
working as a Middle Eastreporter based in Lebanon (where perhapsthe paradigmatic walled-off way-side appeared to him in the
form of the Bekaa Valley). Thatsame epic encounter and its attendant moral topographies to use Melanie
McCalisters media savvy terms(McCalister, 2001) continuesto overshadow The World is Flat where the
sufferings of the slow offline world are always already posed as self-inflicted wounds, the
penalty of deliberate disconnection from the good gamesmanship of the level playing field.
The successful ignoring of history in Friedmans presentism is common in the wider boasts about globalization and is easily
identified (Cooper, 2000;Coronil, 2001). By simply remembering the history of colonialism, oil exploitation
and U.S.-enabled authoritarianism in the Middle East, for example, Friedmansrecycled McWorld versus
Jihadstoryline can be seen as mere coverfor what, by contrast, Timothy Mitchell (2002) valuably posits asthe
more historically hybrid force-fields ofMcJihad. Butreading The World is Flat afterreading a
critical Global South geography such as El Fisgons How to Succeed at Globalization also makesit possible to
develop a double consciousness about the abstract triangulation of geography that accompanies
Friedmanssuccessful ignoring of history. Hisfirst angle away from reality isto ignore his own
privilege and American politics of location. Hissecond isto impose the flat world as a complete
end of geography, a borderless, frictionless, wired and post-Wall end-space to match the End of
History (Fukuyama, 1992). And histhird angle isto then impose equally decontextualizing but
culturalist mapsto explain why there might be exceptionsto the one world flatness of the level
playing field. Following a long tradition ofnorthernist environmental determinism stretching from
Ellsworth Huntington to David Landes(1998), local,supposedly natural or endogenousfactors are thereby
introduced by Friedman to explain why the Global South failsto succeed on the playing field of
globalization. Historical-geographies of connection to the globalsystem through colonialism,
neocolonialism, exploitation and oppression are thereby all angled aside. To invoke the double
consciousnessin El Fisgns critical irony, here we see the power of a kind of global real estate redlining in
action: location, location, location (that is, assume and normalize locations of privilege, ignore and
deterritorialize locations of inequality, and blame and externalize locations of difference).


The 1ac representations of Latin America are patronizing and historically
incomplete
Slater 97 (David, Ph.D from London School of Economics and
Professor Emeritus of Geography at Loughborough University,
Geopolitical imaginations across the North-South divide: issues
of difference, development and power, Political Geography Vol.
16 Issue 8, November 1997, pp. 631-653, Muse, slim_)
The most sustained critique of modernization theory emanated from Latin America and consisted of three main counter-
arguments: Ci) the characterization of the developing world, found in the work of Western
modernization theorists was considered to be simplistic-Third World societies were not given
any history of their own and their history was portrayed as only beginning with their contact
with the West; (ii) following a linear view of development, the already modernized societies were presented as
offering the horizon, the future for the so-called traditional society, which, by adopting Western
innovations, could gradually modernize-Ianni (1971), Santos (1979) and Zea (1970), in their own ways, and together with many other
writers, argued that in our America the evolution of analytical thought had its own specificities and
complexities and belonged on no such subordinating continuum; and (iii) within the modernization
perspective, relations between North and South, between the already modernized societies and the
traditional societies of the periphery, were contextualized as being essentially beneficial for the
developing world-the critical response here was to argue that the historical reality of these relations showed that the
impact of Western interventions in the Third World had been of a predominantly negative
nature, with reference being made to slavery, colonial conquest and imperialism.

Otherizing political representations frame the U.S. as a superior mentor,
suppressing a history of violent encounters
Slater 97 (David, Ph.D from London School of Economics and
Professor Emeritus of Geography at Loughborough University,
Geopolitical imaginations across the North-South divide: issues
of difference, development and power, Political Geography Vol.
16 Issue 8, November 1997, pp. 631-653, Muse, slim_)
As indicated above, categorizations such as First World/Third World, South-North have been employed
within one interpretative arena to draw our attention to the nature of global disparities. Furthermore, their
usage raises significant questions concerning the representation of the other in international
relations. For instance, they can encourage us to examine the dominant forms of enframing non-
Western others that have been deployed across a long sweep of geopolitical history. Overall it is important to bear in
mind that not only is knowledge power but that also power institutionalized in organizations such as the IMF
and the World Bank creates new forms of knowledge and new languages of intervention. As
Foucault (1979: 27) memorably explained, in his investigation of the birth of the prison, power and knowledge directly imply one
another-there is no power relation without the correlative constitution of a field of knowledge, nor any knowledge that does not
presuppose and constitute at the same time power relations.i3 In the ambit of development institutions, the grammar of
structural adjustment and good governance exemplify recent emplacements of orthodox
strategy-they are examples of power producing domains of objects and rituals of truth. But equally in
official reports on development we can also locate discursive elements that connect with another ostensibly distant past. Hence,
when the World Bank notes that adjustment lending has been part of the landscape of the
developing world for over a decade and that a few countries have clearly graduated (e.g. Chile and
Thailand), whilst others are on the road to graduation (World Bank, 1332: 68>, there is a clear link with older notions
of tutelage, trusteeship, guidance and mission. A sense of mission has formed a salient feature of many
geopolitical encounters, and one rather key illustration can be traced back to the last century, in the history of US-
Latin American relations.

Economic engagement with Latin America is undergirded by the imaginary of a
shared regional identity
Garca 8 (Emilio Pantojas Garca is Acting Director of the Graduate School of Business and Senior Researcher of the Center
for Social Research at the University of Puerto Rico, Ro Piedras Campus. He was President of the Caribbean Studies Association
(2004-05) and has published widely on Caribbean economic development issues. Economic Integration and Carribean Identity:
Convergences and Divergences. Carribean Studies 36:1, January-June 2008 [SG])

The literature on economic integration in the Caribbean and Latin America traditionally and primarily
focuses on economic, institutional and geopolitical issues. Regional economic exchanges (trade, investment),
transportation, economic structures, political institutions, as well as geographical and demographic factors are high on the list of
issues / variables pondered and discussed when speaking about regional integration. Cultural and ideological issues are seldom
considered central to the viability of regional integration. The notion that forging a shared Caribbean identity is, or may be, a
necessary precondition to articulate a political project of regional integration is rarely, and only recently, included in the agenda of
regional bodies dealing with economic integration. It appears as if geography, the location on the rim of the Caribbean Sea or on
the "Caribbean Basin," and [End Page 54] the shared historical traits of "plantation economies," are assumed
to constitute the basis of a shared identity. This technocratic view of economic integration centers
on geographic proximity, and assumes that economic exchanges and shared history naturally beget a
"regional" identity. The geopolitics of WWII created a framework of international relations that divided the world into
spheres of influence. The Eastern or Socialist bloc, the North Atlantic alliance between Europe and the United States are but two
examples of this emerging view of the World. This geopolitical cum technocratic view of regions, in turn influenced post war
reconstruction economics and became institutionalized with the creation of the Regional Science Association in 1954 (Isard et al.
1998). The new imaginary of the "regional sciences" focused on geography and economic, demographic, political and institutional
linkages and interactions in defining the makeup of a region. Historical, cultural and ideological affinities were
assumed in construing Europe, Latin America, Africa and Asia as regions. The regional science imaginary in turn
constructed smaller, geographically and historically similar spaces, into subregions, such as Central America, the Andean region and
the Caribbean. The technocratic constructs called regions and subregions became new analytical units for reconstruction and
development economics. Inspired by the success in coordinating the production, delivery and use of weapons on a massive scale
during WW II, development and reconstruction economics advocated the creation of regional economic communities as a means of
taking advantages of economies of scales and complementarities to further economic reconstruction and development. In 1957,
following this new paradigm, the European Economic Community was created. And in 1960, the European Free Trade Association
followed.

Geographic distinctions between the interior and exterior frame the US as a
hegemonic tutor, justifying imperialism and intervention
Slater 97 (David, Ph.D from London School of Economics and
Professor Emeritus of Geography at Loughborough University,
Geopolitical imaginations across the North-South divide: issues
of difference, development and power, Political Geography Vol.
16 Issue 8, November 1997, pp. 631-653, Muse, slim_)
Images of a Father-child distinction did not exhaust the array of symbolic representations used to underpin a variety of penetrations
and transgressions of other sovereignties. President Woodrow Wilson, for example, in the context of political change in
Mexico, introduced the idea that, when properly directed, there is no people not fited for self-
government, and relatedly, in the 193Os, President Franklin D. Roosevelt, in the framework of his Good Neighbor Policy toward
Latin America, suggested that it was our duty to prevent starvation or chaos among the Cuban people (Hunt, 1987: 140). In these
and similar examples, a key point is that within the overall framework of a geopolitics of tutelage the idea
was being fostered that the United States was responsible for good order and political
development. Furthermore, in accordance with a sense of tutelage a distinction was drawn between the people
of the Latin American republics and their governments. With its history of anti-colonial struggle, the United
States was keen to preserve an image of being on the side of the people , and of being
opposed to the imperialisms of the Old World. In this context then, US interventions have sometimes
been legitimized as being in support of a Latin American people subjected to an
unrepresentative and undemocratic government. In a more recent period, for instance, the 1983 invasion of
Grenada was partly justified as a rescue mission-the people of Grenada were being rescued and their rights safeguarded against
their own corrupt and tyrannical government (Weber, 1995). The will to tutelage often flows out of a belief in a
kind of geopolitical predestination, a sense of imperial mission, which is broader and deeper, and arguably more
powerful as a motivating force than the more restricted drive of business interests, or the search for profits, or
new investment outlets. The will to guide and order can also be seen as present in the world of statecraft and geostrategy, where the
politics of rational calculation and realist interventions are not independent of the drive to
civilize. Moreover, I would argue that the invocation of mission, the desire to possess and civilize the other
precedes the issue of security. Threats, dangers, destabilizations only assume their deepest psycho- political charge in
relation to the perception of a mission under threat.29 Equally, the perception of mission flows across territorial
boundaries; it constructs an inside and an outside, a domestic and a foreign that are
invested with connected but distinct political meanings.

Binary representations create spatial rifts between the modernized US and the
backward Latin America
Slater 97 (David, Ph.D from London School of Economics and
Professor Emeritus of Geography at Loughborough University,
Geopolitical imaginations across the North-South divide: issues
of difference, development and power, Political Geography Vol.
16 Issue 8, November 1997, pp. 631-653, Muse, slim_)
The anthropologist Maybury-Lewis (1992: 37) reminds us that since Europeans saw themselves as the bearers of
progress, to induce native peoples to live according to European norms and values was a moral
imperative. Today, the undermining of tribal ways of life can be justified in the name of development; societies that have for
centuries been oriented towards self-sufficiency and long-term management of their resources are considered
backward or as being obstacles to development. In the post-war era, development was very much associated with
being modern, and the 1950s and 1960s were witness to the rise and fall of modernization theory. The will to be modern has
designated two forms of separation. First, we can discern a separation or break in time-the contrast between a modern
now and a traditional, archaic past; and second, a separation in space-a contra distinction between the modern societies
of the West and the posited traditional societies of Africa, Asia and Latin America. These processes of
separation were accompanied by a series of revolutions in science, technology, administration and economy-for Latour (1993: 130) a
veritable bulldozer operation behind which the past disappeared for ever, but in front of which-the future opened up. The second
separation, a kind of geopolitical rift, called into play the need for diffusion. For the traditional to
become modern, spatial expansion and diffusion were necessary. Through a process of adopting
the diffused innovations of modernization-capital, science and technology, entrepre- neurship, Western
values and institutional arrangements-there would be a transition-a movement from traditional to modern.
The belief in the need for diffusion and incorporation again went with a sense of mission. For example, commenting on the first
World Bank mission to Colombia in 1949, Lauchlin Currie, former Harvard economist and official in the Roosevelt
administration, and head of that mission, wrote in 1979 that he had had a reformers zeal, and that Colombia with
such a marvellous number of practically insoluble problems was truly an economic missionarys
paradise (quoted in Escobar, 1995: 55). A sense of mission was not unconnected to a representation of social change in the 1950s
that consisted of a posited three worlds of development (Pletsch, 1981).

Rhetoric of modernization masks the flaws of the First World, furthering binary
representations
Slater 97 (David, Ph.D from London School of Economics and
Professor Emeritus of Geography at Loughborough University,
Geopolitical imaginations across the North-South divide: issues
of difference, development and power, Political Geography Vol.
16 Issue 8, November 1997, pp. 631-653, Muse, slim_)
A further problem of the modernization narrative, equally applicable to neo-liberal interpretations of world
development, concerns the ways in which the First World or the North is itself represented. In most
critical appraisals of modernization and neo-liberal perspectives, the thrust of the argument has centred on the depiction of the
societies of the South and the nature of the relations between North and South or West and non-West. But in both orthodox
interpretations one can find idealized representations of the inner reality of the West, especially in the case
of modernization theory where the scope of analysis was more extensive and the perspective more multi-dimensional. Those other
darker sides of the histories of First World societies, including the phenomena of social
polarization and exclusion, organized crime, corruption, violence, racism and the existence of
fascist regimes frequently remained hidden. One also wonders why within the space of the post-modern more
attention has not been given to the interrupting effect of another kind of inversion where we might examine the
presence of the savage and the traditional inside the civilized and the modern. This aspect of
questioning the way the West is portrayed also needs to be connected to relations with non-
Western others. Here I am stressing the significance of the imbrication of inside and outside and
the need to move away from analytical frames which perhaps over-profile one force in a binary division
implicitly invested with an enduring ethos of stability

Western representations of Latin America are rooted in an Anglo-American
tradition and impose flawed binaries
Slater 97 (David, Ph.D from London School of Economics and
Professor Emeritus of Geography at Loughborough University,
Geopolitical imaginations across the North-South divide: issues
of difference, development and power, Political Geography Vol.
16 Issue 8, November 1997, pp. 631-653, Muse, slim_)
5. Finally, with respect to issues involved in the production and dissemination of knowledge, we have
another example of an important overlapping of inside and outside which involves global politics,
learning and cultural understanding. Thinking about the geopolitics of knowledge in terms of the ethics
of encounter, the reference to a North- South divide or West/non-West distinction, poses a series
of questions for the future. The category West, traditionally associated with Western Europe, North
America, and an outlying Australasia and contrasted with the East in Cold War terminology, has been
charged with a meaning that goes beyond geography. In the case of military intervention in the Southern Cone,
for example, and specifically in relation to state terror in Argentina, defence of the West as a mythical
construction was an important element of the military regimes overall discourse, as reflected in the
fact that one Admiral commented that the West today is a state of the soul, no longer tied to
geography (quoted in Graziano, 1992: 123). More recently, in debates on multi-culturalism and the relationship between
literature and a traditional canon of prioritized texts, or in exchanges on a posited clash of civilizations (Huntington, 19931, the
West versus the Rest has been invoked as an organizing split. North/South categorizations are
more customarily rooted in the environment and development literature, but with both sets of geo-
historical categories we are faced with a certain interpretative duality. On the one hand, these binary
divisions invite dis-aggregation and the underscoring of heterogeneities within them, not only
within the South but also the North. As regards industrialization, technological progress and socio-
economic diversification, the countries of East Asia hardly conform to our older images of Third
World underdevelopment, and they are themselves being taken as a model for countries inside
the West. Again, how do we situate the variegated regimes of the Middle East region where geopolitics, oil, sheikdoms and
religious movements create their own specific currents of political change? Or, within the West, to what extent is the
literature on socio-political and cultural change implicitly anchored in a view of the West as an
Anglo-American construction? Do generalizations based on the historical experiences of Britain, France, and Germany
apply to countries such as Spain, Portugal, Ireland and Italy? On the other hand, I would argue that there is a unifying
meaning, albeit a precarious one, to the North-South divide and this lies in the history of
geopolitical relations and questions of power and representation. There is a geopolitical charge or cathexis, a
kind of psycho-political phenomenon, embodied in the histories of oppositional nationalisms and varied forms of
resistance to the diffusion of metropolitan powers, that has been formed over a long period of North/South
encounters. The societies of the South have always had to react to initiatives and projects emanating from the North; their
own realities cannot be comprehended outside the histories of external penetrations and
interventions, which have structured and in turn have been affected by internal specificities. Moreover, as I indicated
previously, North/South inequalities continue to present an ethical question that is also part of the history of geopolitical
interventions.

Latin America is a colonialist constructionit subverts indigenous cultures.
Holloway 10 Thomas H. Holloway, Professor of Latin American History at the University of California at Davis, former
Director of the Hemispheric Institute on the Americas, former President of the Latin American Studies Association, former
Executive Secretary of the Conference on Latin American History, 2010 ("Latin America: Whats in a Name?, Essay Published in A
Companion to Latin American History, Published by Wiley-Blackwell, Available Online at
http://www.academia.edu/202121/Latin_America_Whats_in_a_Name, Accessed 06-23-2013 [NN])
But there are other questions that need to be posed, in the age of identity politics and the assertion
of alternative ethnicities and nationalisms. By its historical and intellectual origins and the
claims of pan-Latinism, the term Latin America privileges those groups who descend from Latin
peoples: Spain and Portugal (but not, ironically enough, the French-speaking populations of Canada or the Caribbean).
By another set of criteria, what is now commonly called Latin America might be subdivided into
those regions where the indigenous heritage is strong and native identity has reemerged to claim
political space, especially in Mesoamerica and the Andean region; Afro-Latin America,
especially the circum-Caribbean region and much of Brazil; and Euro-Latin America, in which
relatively massive immigration from 1870 to the Great Depression of the 1930s transformed the demographic
and cultural makeup of southern Brazil, Uruguay, and Argentina (Rojas Mix 1991). In other words,
Latin America as a term ignores or claims dominance over other cultures in the
region, which have recently come to reassert their distinctive traditions, including a plethora
of languages spoken by tens of millions of indigenous peoplenone of which have any
relationship to Spanish or Portuguese (or Latin) beyond a scattering of loan words. The current
condition of peoples of indigenous and African heritage has a historical relationship to
conquest, colonialism, subjugation, forced assimilation, exploitation,
marginalization, and exclusion. Those are not processes to celebrate and use as the basis
for national or regional identity challenging the hegemony of the Anglo-Saxon race, as was the
thrust of pan-Latinism of yore. But they are basis for claiming cultural and political spaceas well as
territory and access to resourceswithin Latin America, today and into the future.




1NC HEG

The hegemonic conceptualization of places as lacking stability is flawed and
reproduces the violent effects they claim to solve
Bialasiewicz et al, 07 a Department of Geography, Royal Holloway University of London (Luiza Bialasiewicz, 2007,
Performing security: The imaginative geographies of current US strategy, Political Geography, 405-422)//ah
It is telling just how spatialised some of these specications become when worked through in detail.
Already in 2000, PNAC argued that the major military mission is no longer to deter Soviet expansionism, but to secure and
expand zones of democratic peace; deter rise of new great-power competitor; defend key
regions; exploit transformation of war (PNAC, 2000: 2). They suggested that rather than the Cold Wars potential
global war across many theatres, the concern now is for several potential theatre wars spread across the globe
fought against separate and distinct adversaries pursuing separate and distinct goals (2000: 2, 3).
To counter such threats, the US needs to station its troops broadly, and their presence in critical
regions around the world is the visible expression of the extent of Americas status as a
superpower and as the guarantor of liberty, peace and stability (2000: 14). They claimed that while
US security interests have expanded, and that its forces provide the rst line of defense in what may
be 410 L. Bialasiewicz et al. / Political Geography 26 (2007) 405e422Author's personal copy described as the American security
perimeter, at the same time the worldwide archipelago of U.S. military installations has contracted (2000: 14, 15). Because
the security perimeter has expanded slowly but inexorably since the end of the Cold War, US forces e
the cavalry on the new American frontier e must be positioned to reect the shifting strategic
landscape (2000: 14, 15). Equally, their use of the term homeland drew strongly on its use in the Clinton administration e and
pregured the creation of the Ofce for Homeland Security under G.W. Bush, with the concept strengthened by both the PATRIOT
acts and the establishment of U.S. Northern Command. Again, it is essential that we conceptualize these
strategies as both containing and making imaginative geographies; specifying the
ways the world is and, in so doing, actively (re)making that same world. This goes
beyond merely the military action or aid programmes that governments follow, but indicates a
wider concern with the production of ways of seeing the world, which percolate through media,
popular imaginations as well as political strategy. These performative imaginative geographies are at the heart of
this paper and will re-occur throughout it. Our concern lies specically with the ways in which the US portrays e and
over the past decade has portrayed e certain parts of the world as requiring involvement, as
threats, as zones of instability, as rogue states, states of concern, as global hotspots, as well
as the associated suggestion that by bringing these within the integrated zones of democratic
peace, US security e both economically and militarily e can be preserved. Of course, the translation
of such imaginations into actual practice (and certainly results) is never as simple as some might like
to suggest. Nonetheless, what we wish to highlight here is how these strategies, in essence,
produce the effect they name. This, again, is nothing new: the United States has long constituted its
identity at least in part through discourses of danger that materialize others as a threat (see
Campbell, 1992). Equally, much has been written about the new set of threats and enemies that emerged to ll
the post-Soviet void e from radical Islam through the war on drugs to rogue states (for a critical
analyses see, among others, Benjamin & Simon, 2003; Stokes, 2005; on the genealogies of the idea of rogue states see
Blum, 2002; Litwak, 2000).


2NC HEG

Kagan and Thayer have geographically determinist views of the world their
scholarship and production of security policy is shaped by geoeconomic logic that
seeks to secure American expansionism at the expense of all else
Morrissey, 10 professor of geography at national University of Ireland, 2010, Antipode, Closing the Neoliberal Gap: Risk
and Regulation in the Long War of Securitization, Vol. 43 No. 3)//CC
From popular IR commentators to frequently cited Strategic Studies experts, imperial geopolitics has once again become a key discourse in
theorizing the contestation for global power. Bradley Thayer openly makes the case for a geopolitical grand
strategy for American Empire (Layne and Thayer 2007); Robert Kagan sees a bi-polar geopolitical world
divided into liberal zones and autocratic zones (Kagan 2008:4); while Robert Kaplan (2009) conceives of a
Mackinderinspired geopolitical world defined by a simplistic geographical determinism that requires controlling by military interventionism. Indeed,
as argued elsewhere, the recent trajectory ofUS Strategic Studies reveals an emboldening of reductive
and imperial geographically determinist geopolitics in the scripting of US national security
and foreign policy (Morrissey 2009a:37). Building upon work over the last two decades in the subfield of critical geopolitics that has
focused on interrogating the discursive and material production of geopolitical space, geographers in recent years have variously resisted such
essentialist and dangerous geopolitical formulations by offering important counter-geographies of our contemporaryworld (Coleman 2004; Dalby 1991;
Dowler and Sharp 2001; Hyndman 2007; Kearns 2009; O Tuathail 1996; Roberts, Secor and Sparke 2003).1 In addition to the critique
of imperial geopolitics, its apologists and enthusiasts, geographers have increasingly theorized
geoeconomics as capturing more closely the dominant concern of international relations in a
globalized, if uneven, economic world. Deborah Cowen and Neil Smith (2009:2425), for instance, point to recent shifts that
challenge geopolitical conceptions of space, power and security and illuminate the
recalibration of geopolitics by market logics and geoeconomics. By using the example of recent changes in US
maritime border security prompted by the impossibility of the geopolitical border given the geoeconomic reframing of insecurity, they lay bare how
market calculation has come to supplant the geopolitical logic of state territoriality, and how
US geopolitical power is earned via global geoeconomic extension (Cowen and Smith 2009:32, 34, 43). For this
paper, recognizing and interrogating the neoliberal securitization project at the heart of CENTCOM necessarily involves thinking through how US
geopolitical power is solidified by geoeconomic calculation and practice, but what I want to do initially is to discuss briefly the papers use of the key
terms geoeconomic and securitization.


Heg advantages scripted by clowns like Barnett exemplify the military-strategic
studies complex that seek to map predermined worldviews onto geographical
reality in a violent fashion
Morrissey, 11 professor at National U Ireland (John Morrissey, 2011, Antipode, Architects of empire: The military-strategic
studies complex and the scripting of US national strategy, 43.2)//CC
The recent work of Simon Dalby, Stephen Graham, Derek Gregory and others is both insightful and urgent in illuminating the huge discursive efforts
in the US-led war on terror in constructing and reconstructing key spaces of the Middle East
as little more than receiving points for US military ordnance (Graham 2005:6; cf Dalby 2007b; Gregory 2004). As
outlined earlier, there is of course a long history of the US military, and its strategic studies advisors, mobilizing abstract geostrategic discourses of the
Middle East (Klein 1994). The lead-up to the Gulf War in 1991, for example, was a particularly fertile period for airing reductive military visions
(Sidaway 1998); and there is a continuum of essentialist scriptings of the Middle East that extend back to at least the late 1970s when the military
strategic studies complex began to assiduously assert US geopolitical and geoeconomic designs for the region in the name of national security
(Morrissey 2008). These strategic studies scriptings have collectively served to establish a register of ageographical spaces, have long spoken of terrains
and not worlds, and have been typically indifferent to the lives of Others (Epstein 1987; Record 1981a; Ullman et al 1996). Critical to our
reading of the militarystrategic studies complex, moreover, is the recognition that it does not
operate outside of the political, decision-making process; as shown above in relation to the
Center for Strategic and Budgetary Assessments. Upon taking up office in 1981, the Reagan administration actively
consulted with the Institute for Foreign Policy Analysis in planning an effective US geopolitical strategy for the Middle East, and promptly followed its
recommendations (and those of its chief specialist, Jeffrey Record) in initiating, and budgeting for, US Central Command as a
military necessity to defend US national interests in the Gulf (Record 1981a). The long-standing
influence on US foreign policy of American pro Israel lobby groups and think tanks has been
recently demonstrated by John Mearsheimer and Stephen Walt (2006). Others have shown the influence of the Project for the New American
Century on the current Bush administrations particular brand of aggressive foreign policy (Dalby 2006). And one of the architects of
that policy, Donald Rumsfeld, as Secretary of Defense, was not averse to sitting down for panel discussions to
review the findings of, for example, Brookings Institution surveys (US Department of Defense 2003). It is
important to remember too that many of the leading Pentagon and Congressional advisors on
the Middle East, such as Kenneth Katzman, for instance, are typically also research analysts in
strategic studies institutes (Katzman is an external researcher for the Strategic Studies Institute
at the US Army War College); thus enabling the governmentstrategic studies loop (Katzman
2006). Thomas Barnett, too, who worked as the Assistant for Strategic Futures in the Office of
Force Transformation at the DoD from the end of 2001 to mid 2003 simultaneously held a
professorship in strategic studies at the WarfareAnalysis and Research Department at
theUSNavalWar College in Newport, Rhode Island. His combined DoD and strategic studies
work culminated in the publication of his influential and commercially successful The Pentagons New
Maps in 2004, in which he envisages a new grand strategy for the USA in a post-Cold War and post-9/11 age: closing
the gaps of neoliberal economic order across the globe (Barnett 2004; cf Dalby 2007a). Such academic strategic scriptings of
US national security have long proved a supporting and legitimating intellectual cache for
military action; they have been instrumental in the advancement of what Bradley Klein calls a
cultural hegemony of organized state violence (1988a:136). A recent case in point was provided by the current Commander
of the Multi-National Force in Iraq, General David Petraeus. Writing in 2006, the much-heralded military saviour for the
Iraq War did not just see an infantry surge as the key to success. He recognized too the
importance of what has become a buzz word in US military circles in recent years, culture:
Knowledge of the cultural terrain can be as important as, and sometimes even more important
than, the knowledge of the geographical terrain. This observation acknowledges that people are, in many respects, the
decisive terrain, and that we must study that terrain in the same way that we have always studied the geographical terrain (2006:51).

1NC INSTRUMENTALIZATION

They instrumentalize Latin America as a well of resources, perpetuating
dependency on the U.S. master narrative
Joseph 98 (Gilbert M., Ph.D. in Latin American History from Yale
University, Professor of History and International Studies at
Yale University, "Close Encounters: Toward a New Cultural
History of U.S.-Latin American Relations," pp. 3-46, slim_)
New theorists of "imperialism," for example, focus on the U.S. (or European) center's penetration of the Latin American periphery.
Imperi- alism's main branches are held to be political, military, and economic; secondarily it involved the inexorable transfer-
indeed, virtual imposi- tion-of a kind of cultural compost, the so-called American way of life. Concerned mainly with the
question of uneven power relations between nation-states and with the tensions created by exports of
capital to social formations that were in a less "advanced" state of development. this view has
presented the growing and multifaceted connection between the United States and Latin America as a
relationship between two distinct political entities and two economies. American businessmen,
diplomats, and military personnel abroad are typically portrayed as instruments of an alliance
between capital and the state to conquer markets, tap cheap sources of raw materials, and
consolidate an asymmetrical relationship of power." In similar fashion, dependency and world-
systems models take off from a series of inequalities located in international trade and finance.
They then proceed to map out a complex network of relationships by means of which local governments. ruling elites,
political parties, and institutions in civil society have become involved in the reproduction of a
structural condition- "dependency " -that prevented the "peripheral" countries of the region
from achieving the levels of development of the northern "metropolis" or "core." While less tied to notions of the
metropoIis's expansion or "spi|lover" into the periphery than theories of imperialism (or "revisionist" U.S. diplomatic historians),
dependency fonnulations have retained the central idea of "penetration"; this time, however, the vehicle of penetration was an
ensemble of U.S. capital, technology, and culture." The rationale of the capitalist system remained the same: the
reproduction of a highly skewed pattem of accumulation that rewarded the productivity of the
North via the exploitation and impoverishment of the South. Now, however, local actors were implicated in
the relationship from the start: accomplices, or "compradors," in an "infrastructure of dependence" that drained resources and
creativity, reduced the sphere of liberty, and repro- duced the syndrome of poverty." Thus, the master narrative of
"dependency," like that of "imperial- ism," ha presupposed a bipolar relationship that
subsumed difference (regional, class, racial/ethnic, gender, generational) into the service of a greater machinery that set
limits. extracted surpluses, established hier- archies, and shaped identities. Both narratives have depicted the United States
(or the "core" nations of the world-system) at the controls of a great "neocolonial" enterprise, managing a
stream of flows unified by the logic of profits, power, and a single hegemonic culture. From the center
flowed commodities; capital; technology; cultural artifacts; and military power, equipment, and
expertise-in order to reproduce more of the same. In the periphery. these narratives often suggested, there were only forces and
agents that abetted or constrained these flows. Nations were frequently personified and gendered: each had
its own national interest and manly persona, and acted in compliance with it or betrayed it, de- pending on the
degree to which dependency had advanced. Of course, by imagining national entities motivated almost
exclusively by economic interest, dependentistas challenged the self-loathing notion, first preached
by nineteenth-century elites, that the "national character" was culturally incapable of economic modernization: too indolent,
improvident, and un- savvy to be a serious contender in the race for "progress" "2 Nevertheless. this one-dimensional
perspective of "comprador elites" had the effect of redetining locals as foreigners, and preempted the
examination of other relations, shared assumptions, and emotional and other affinities between foreign agents and local elites."




1NC THE

The word the in front of usfg is a violent act of geopower
Thrift 2000[Nigel Prof of Geography @ Univ. of Bristol. "Its's the Little Things" Geopolitical Traditions. A Century of
Geopolitical Thought: in Atkinson, D., Dodds, K. (eds) Page 383-385 Google Print]
words function to bring about geopolitical change and it is not possible to do so as long as
geopolitical forces continue to be framed as "big" and "commanding" (with all the masculine overtones.)
Some of the most potent geopolitical forces are, I suspect, lurking in the 'little' 'details' of people's
lives, what is "carried" in the specific variabilities of their activities' (Shotter and Billig 1998: 23), in the context of utterances.
And these variabilities have immediate consequences. Thus, As Bakhtin notes, and is confirmed by the work in
conversational analysis, 'we sensitively catch the smallest shift in intonation, the slightest interruption of voices
in anything of importance to us in another person's practical everyday discourse. All those verbal sideward glances,
reservations, loopholes, hints, thrusts do not slip past our ear, are not foreign to our own lips'
(Bakhtin 1984: 201). And we in turn show our stance to what they do or say also in fleeting bodily reactions, facial expressions,
sounds of approval or disapproval, etc. Indeed, even in the continuously responsive unfolding of non-linguistic activities between
ourselves and others - in a dance, a handshake, or even a mere collision on the street - we are actively aware of whether the other's
motives are, so to speak, 'in tune' or 'at odds' with ours. And in our sense of their attunement or lack of it, we can sense their attitude
to us as intimate or distant, friendly or hostile, deferential or arrogant, and so on. (Shotter and Billig 1998: 23). Thus, very effective
work has been done in disciplines like anthropology and discursive psychology (Billig 1995, 1997) which attempts to provide a sense
of how national identity and an accompanying geopolitical stance are inscribed through the
smallest of details. Thus, for example, national identity is not accomplished in grand displays which
incite the citizen to wave the flag in a fit of patriotic fervour. Instead, it goes on in more mundane
citations: it is done unobtrusively on the margins of conscious awareness by little words such as 'the'
and 'we'. Each day we read or hear phrases such as 'the prime minister', 'the nation', or 'the weather'.
The definite article assumes deictically national borders. It points to the homeland: but while we, the
readers or listeners, understand the pointing, we do not follow it with our consciousness - it is a 'seen but unnoticed'
feature of our everyday discourse. (Shotter and Billig 1998: 20). Such work goes some way towards
understanding the deep, often unconscious aggressions which lurk behind so much geopolitical
'reasoning', which through small details build a sense of 'us' as not like 'them', and from which
political programmes then flow as infractions are identified and made legible. In these few brief
comments, I hoped to have outlined a parallel agenda for critical geopolitics, one still based on discourse, but on
discourse understood in a broader way, and one which is less taken in by representation and more attuned to
actual practices. In turn, such an agenda leads us away from interpretation of hyperbolic written and
drawn rhetorics (which, I suspect, are often read by only a few and taken in by even fewer) towards the (I hesitate to say 'real')
work of discourse, the constant hum of practices and their attendant territorializations within
whcih geopower ferments and sometimes boils over.

1NC STATE

The state restructures geographical characteristics to reinforce objectives that
glorify the Government
Gill 2010 (Nick. "New state-theoretic approaches to asylum and refugee geographies." Progress in Human Geography 34.5
(2010): 626-645. http://phg.sagepub.com/content/34/5/626.short BRW)
Political-economic theories of the restructuring of the state in the face of post-Fordist, Schumpertarian pressures employ a notion of
the thorough imbrication of state by society, so that the state is understood as nothing more than a form-determined condensation
of social forces (Jessop, 1990; 2002; Brenner, 2004). The state is seen here as a means of interacting between social factions, for
example in ways that are considered legal, jurisdictional and penal. In this sense the state is viewed as a social
relation, underscoring the contingency of state power upon social action and interaction (Jessop,
2007). Understood in this way, political economists within and outside geography have examined the profusion of
scales upon which state-like social relationships occur (Brenner, 1999; Goodwin et al ., 2002; Jones et al ., 2004). They have argued
that the state relation is being hollowed out from its national nexus to both subnational and supranational scales (Roberts and
Devine, 2003). The politicaleconomic state literature arising within sociology consequently escapes from both the separate-spheres
assumption by conceiving of the state as a social relation and the tendency to associate states with national-level
polities by examining its de- and reterritorialization at a variety of scales (Brenner, 1999). A minority of
forced migration scholars have taken these theorizations of states seriously and drawn attention to the ability of states to direct
asylum flows not directly but through the governance of a range of actors involved in the asylum sector, whose location with respect
to the state is often unclear or ambivalent (Lahav, 1998; Zolberg, 1999; Guiraudon, 2002; Samers, 2003). These actors might
include privately contracted detention custody officers, police officers, judges, immigration officials,
security staff, asylum advocacy groups, charity organizations, airline and shipping employees,
health and education service providers and communities of refugees in destination countries.
Increasing attention is being given both to the factors that influence this diverse set of actors and to the role of the state in
configuring such influences. Lahav and Guiraudon (2000), for example, outline the exteriorization of state control over asylum
migration through three interrelated strategies upward, downward and outward divestment of responsibility from the central
state. Upwardly, states are increasingly engaged in international collusion in the area of migration
control, for example through shared security measures (Cholewinski, 2000), coordinated use of transit countries as
buffer zones to reduce applications to popular destination countries (Collinson, 1996), and the wholesale transfer of legal
mechanisms for the governing of refugees and asylum seekers from national to international law (Noll,
2000). Downwardly, local government has extended its activities in checking the legal status of immigrants, and local public-sector
organizations such as police forces and hospitals have become increasingly active both in the verification of immigration status and
in the subsequent withholding of services from those without status (Cohen, 2002; Groenendijk, 2002). The third trend identified by
Lahav and Guiraudon (2000), alongside the upward and downward shift in government responsibility, is an outward shift in
responsibility away from states towards social actors. This shift, they argue, has been brought about by legal innovations that render
social actors increasingly responsible for the policing of asylum seekers. The levying of fines by the state upon private transport
companies if they are discovered to be transporting clandestine immigrants represents one such example, the levying of fines upon
the named contacts of would-be immigrants in destination countries if immigrants paperwork is found wanting constitutes another,
and fines levied against immigrants themselves for the short-term costs of their own
incarceration represents a third example of this trend (Guiraudon and Joppke, 2001; Guiraudon, 2002). These
measures are indicative of a shift away from direct state policy towards governance and
autonomization in the asylum migration control arena, accompanied by state withdrawal and minimization. While
the political-economy of the states geographical restructuring is a useful lens through which to
understand responses to global pressures, there is, however, still an implicitly essentialist notion of a
state from which powers have been exteriorized. For example, the form of governance that Lahav and
Guiraudon(2000)discuss relies upon legal innovations and sanctions, thereby not admitting that social factions may be driving,
rather than simply driven by, asylum law. What is more, this notion of governance privileges legal power which is guaranteed by the
eventual threat offorce. Such a privileging reproduces realist notions of the state and can obscure
alternative forms of power (see Allen, 2003). Rather than overriding or appealing to exogenous subaltern interests, some
forms of power contest the very aspirations of subjects themselves, thereby owing less to legal and institutional governance than to
techniques that act upon what a volitional subject actively seeks to achieve (Foucault, 1979; Lukes, 2005). That these more insipid,
governmental forms of power are also often referred to as governance in debates about state
restructuring, albeit in a Foucauldian sense, is deeply unhelpful (Walters, 2004). While the notions of state de- and
reterritorialization are undoubtedly more sophisticated than a simple state-versus-society approach, employing at the very least a
dialectic concept of the relation between state and society (see Pierson, 2004), there remains a sense in which the two are ultimately
separate domains, that the state is still driving social factions to act in calculated ways and that the means by which this is achieved
are through relatively overt, legal and financial mechanisms that preclude ideological struggle. Poststructural state
theorizing among geographers seeks to open up new ways of thinking about the state, and is
being both applied and developed by scholars who are engaging explicitly with issues facing
forced migrants and refugees. In particular, a number of geographers have become concerned
about the consequences of an essentialized state concept and have theorized the state differently
in order to ameliorate some of these. Painter (2006) for example, motivated by a concern for the importance of
situated, everyday practices in the (re)production of state effects, draws upon the work of Philip Abrams (1988) and Timothy
Mitchell (1999) to suggest that the study of the state might usefully be substituted for the study of the notion or idea of the state.
While it is, for Painter, almost impossible to pin down precisely what the state is, the more interesting question is to examine what
the state is taken to be, by whom and in which historical periods. In this way, attention is then allowed to focus upon
the social effects of particular understandings of the state, especially among state actors
themselves, although this category is deeply contested in this view (see also Bourdieu, 1994). The shifting, extending
and deepening of particular notions of statehood throughout society a process that Painter
describes as statization may go some way towards explaining the changing behaviour of distant social actors without
recourse to legal or coercive state powers. If Painters work can be understood as a way to reimagine the social power of the state,
other geographers have engaged in a range of attempts to reimagine its spatial forms. Staeheli and Mitchell (2004), for example, are
motivated by a concern that essential notions of the state cannot capture the mixing of public and private domains in the current
epoch, and that this failure to focus on the rearrangement of public and private space leaves our understanding of democratic
processes wanting. Similarly, Radcliffe (2001) responds to the difficulty of viewing the state as a bounded and territorially static
entity by drawing attention to the networked transnationalization of state power itself. The broader geopolitical and institutional
settings for transnational connections, she writes, demonstrate first, the continued salience of state power, and second, the ways in
which transnational connections are in themselves bound up with the states reproduction (Radcliffe, 2001: 20).

1NC TROPICALITY
Representations of tropicality justify imperialist intervention
Frenkel 96 (Stephen, Ph.D. in Geography from Syracuse
University, specializing in social/cultural geography with a focus
on Latin America, Assistant Professor of International Studies at
Portland State University, Jungle Stories: North American
Representations of Tropical Panama, Geographical Review, Vol.
86, No. 3, Latin American Geography (Jul., 1996), pp. 317-
333, slim_)
'During the late nineteenth and early twentieth centuries, as tropical Central America came under increasing
U.S. influence, U.S. policymakers, businesspeople, missionaries, and bureaucrats began to
transform the region to meet their needs.' They built railways, led military invasions, established banana
and coffee plantations, and eventually dug a canal across Panama. Their published accounts and artistic renderings
of Central America drew on more generalized, archetypal ideas in the art, history, literature, and
photographs of tropics around the world to form a specific discourse about the Central American tropics.2
Two opposing narratives constituted this discourse: positive ones about Edenic paradises, fertile
soil, and exotic beauty; and negative ones about moral laxity, dangerous landscapes, disease,
and the threatening abundance of the jungle. These varied ways of seeing Central America revealed
themselves in more than just semantic representations; they influenced U.S. actions and policies
in the tropics. These contradictory narratives were used to legitimate imperialist intervention and actions in
the Panama Canal Zone in the early twentieth century.

2NC TROPICALITY

Tropical discourse homogenizes and constructs flawed representations of Latin
America
Frenkel 96 (Stephen, Ph.D. in Geography from Syracuse
University, specializing in social/cultural geography with a focus
on Latin America, Assistant Professor of International Studies at
Portland State University, Jungle Stories: North American
Representations of Tropical Panama, Geographical Review, Vol.
86, No. 3, Latin American Geography (Jul., 1996), pp. 317-
333, slim_)
In this geographical and historical context, the character of tropical climates was frequently
expressed in subjective terms, even when placed within a so-called scientific framework. Turn-of-
the-century U.S. natural science textbooks typically included a classification of the tropical flora,
fauna, temperatures, and diseases in Central America. Such descriptions, however, were frequently
intertwined with an author's opinions concerning heat, disease, dark-skinned peoples, hot or spicy
foods, exotic fruit, fecund vegetation, and economic underdevelopment. For example, in his 1918 Handbook of
Commercial Geography, Geo. G. Chisholm describes the specific rainfall amounts, humidity, and temperature that are characteristic
of the tropics while noting the "excessive" heat and "irksome" humidity (p. 23). "Scientific" discourse on the tropics
was full of value-laden descriptions. Without doubt, the label tropical has been used to stereotype
and homogenize a wide range of places, from Singapore to Sierra Leone. Even so, the discourse is heavily
influenced by a distinctive set of regional identities. In Western discourse, archetypal tropical
representations can be identified for Central America, for West Africa, and for the South Pacific. Perhaps most
famous among these is the overwhelmingly posi-tive Eurocentric portrayal of the South Pacific. Based on the gendered, Edenic
visions of Captain James Cook, Louis-Antoine de Bougainville, and Paul Gaugin, a recog- nizable image was set by the early
twentieth century. Indeed, it was so well ingrained that when Alec Waugh arrived in Tahiti in 1930, he wearily commented: " [T] he
South Seas are terribly vieuxjeu. They have been so written about and painted. Long before you get to them you know precisely what
you are to find" (Waugh 1930, 20). By contrast, based on a well-deserved reputation for extremely high death rates, representations
of the West African tropics invoked fears of death and disease. "The deadliest spot on earth" was how British doctors described the
region to Mary Kings- ley prior to her 1893 journey (Kingsley 1987, 12). As epidemiological danger com- bined with racial prejudice,
travelers to the West African coast were warned to prepare "for lonely ports of call, for sickening heat, for swarming multitudes of
blacks" (Davis 1907, 8). This discourse, of course, hit its zenith in Joseph Conrad's Heart of Darkness (1910). Such views are still
reproduced in sources as varied as jour- nalistic "rough-guide" accounts of the horrors of West African politics and literary
anthologies of "rain-forest" fiction like Tales from the Jungle: A Rainforest Reader (Katz and Chapin 1995), which continue to
include as staples selections from Conrad and Kingsley. Representations of the American tropics likewise
developed a recognizable char- acter. More so than descriptions of the South Pacific or West Africa, ideas about the
American tropics were ambivalent. As the geographer Susan Place writes, Since their first encounters with Latin
America, Europeans have expressed mixed feelings about the tropical rain forest. The lure of
fabulous wealth and the hope of finding El Dorado have wrestled with the dread of mythical beings and
horrible dis- eases in the green hell. Accounts of the tropical rain forest, whether novels, travel journals, or
scientific reports, reveal at least as much about their authors as they do about the forest. Every writer represents to a certain
extent the prevailing worldview of his or her time and culture, but perceptions of the rain forest are
also filtered through the lens of meanings created by the individual's experiences and beliefs.
(Place 1993, 1) Much of the strongly positive sense of the American tropics was in place by the early nineteenth century. A number of
commentators, including Kathryn Man- thorne (1989) and Fredrick B. Pike (1992) have suggested that North Americans in-
terpolated the region's character from only a few sources, including newspaper articles, artists' reproductions, and lavishly
illustrated travelers' accounts such as John Lloyd Stephens's 1841 Incidents of Travel in CentralAmerica, Chiapas and Yuca- tan.
Given that the U.S. populace knew little of the region, visions of South and Cen- tral America were easily
lumped together. So it was that, with comparatively little geographical specificity, a "unified pictorial
consciousness of Latin America emerged in the United States ... in direct response to a lacuna of knowledge.
Its image as a land of scientific wonders, golden riches, and edenic innocence could be main-
tained only so long as accurate information and direct experience were at a mini- mum" (Manthorne
1989, 60-61).

Derogatory representations of tropical regions justify instrumentality and
colonialism
Frenkel 96 (Stephen, Ph.D. in Geography from Syracuse
University, specializing in social/cultural geography with a focus
on Latin America, Assistant Professor of International Studies at
Portland State University, Jungle Stories: North American
Representations of Tropical Panama, Geographical Review, Vol.
86, No. 3, Latin American Geography (Jul., 1996), pp. 317-
333, slim_)
Visitors marveled at the speed at which plants grew, creating a veritable "flood of tropical
vegetation" (Tomes 1855, 78). Even a century later, an author observed that "the tropical cousin of the tree that grows in
Brooklyn will likely grow somewhere between two and nine times as fast in Panama or Honduras" (Wilson 1951, 5). Given these
assumptions, the commonplace conclusion was that tropical living was as easy as reaching to the
closest tree for sustenance. Ellsworth Huntington expressed this thought when he somewhat jokingly opined that
in tropical regions "the native has nothing to do except lie under the trees and wait for the fruit
to drop into his mouth" (Huntington 1922, 281). This contrasted with the perceived hard life of winter
and work in the temperate zone. Such positive descriptions were often used to promote agricultural
enterprise. Because tropical land was available for the taking with minimal effort by U.S. imperi-
alists, they came up with a variety of schemes to promote rubber, coffee, and banana
plantations. Drawing heavily on the idea of tropical fertility, these schemes suggested that one's
choice of crop was as simple as deciding what would bring the highest price on the world market
(a claim which is still being made today [Slater 1995, 115]). All that an investor needed was the input of labor and
technology. Implicit in such representations was that the indigenous population had
been unable to provide adequate labor or technology, thereby explaining the
availability of land (Adams 1914, 203). Investors equated an "untapped" natural landscape with
profit. Reference might be made to "the wondrous wealth of the Isthmian forest" (Otis 1867, go) or to the fer- tility of the soil.
"Stick an umbrella in the ground over night" said one commentator, "and you'll have an umbrella tree in the morning" (Putnam
1913, 89). Even after a keen awareness of the limitations of the soil developed, the land was still portrayed as an
extraordinary, if temporary, resource. Plantation profits "would amply justify the exhaustion of
the land" (Crowther 1929, 245).




AFF SPECIFIC

1NC BORDER SECURITY

Mexican geography is the intersection of US neoliberal and security practices
border security is the Norths attempt to secure the harms of economic expansion
and development, perpetuating militarism and widespread social injustice in the
global South
Coleman, 05 (Matt Coleman Department of Geography, UCLA, 2005, U.S. statecraft and the U.S.Mexico border as
security/economy nexus) MD
Although not denying the symbolic importance of U.S. geopolitical practice in the border region, Nevins (2002) suggests that
border policing cannot be condensed to a media event. Rather, Nevins counsels that as a real-world militarized
practice responsible for large numbers of migrant deaths (see also Eschbach, Hagan, Rodriguez, Hernandez-
Leon, & Bailey, 1999), border policing tends to complement the neoliberalization of the border in
that it concerns a xenophobic and hypernationalist instatement of borderland law and order
against ows of migrants unleashed by the liberalization of rural and urban Mexico. In this sense,
Nevins names the U.S. a gatekeeper state which manages the migratory fallout of U.S.-led
Mexican market restructuring. The gatekeeper state, Nevins argues, provides extraterritorial
opportunities for national territory-based capital (thus intensifying the process of globalization)
while, somewhat paradoxically, providing security against the perceived social costs unleashed
by globalization (2002: 178). Both Andreas and Nevins caution against a theorization of U.S. statecraft as a coherent
phenomenon. However, the tendency in both projects is, generally, to look for the points of coincidence between U.S. geopolitical
and geoeconomic practice in the border region. This tendency is front and center in Andreas analysis, in which border
policing is presented more or less functionally in terms of a larger U.S. project of continental
neoliberalization. For example, Andreas seems to suggest that U.S. geopolitical practice (i.e. customs and
immigration policing) exists as a second-order theatrical foil to U.S. geoeconomic interests (i.e. free
trade), such that real U.S. trade interests are strategically accompanied by parallel security
images. In contrast, the incoherences of U.S. policy in the borderlands specically, the struggle between the U.S. free trade
and border policing agendas, in both local and national contexts are stated much more forcefully in Nevins work which is
concerned to highlight how dierent boundary regulatory regimes relate to the [contradictory] security
and opportunity components of the modern territorial state (2002: 178). This said, in his specic empirical
discussion of U.S. border policing in the mid- to late-1990s, Nevins argument tends to reconcile U.S. geopolitical practice in the
borderlands with U.S. geoeconomic interests. For Nevins, the U.S.-led NAFTAization of the border region and
U.S. border policing go hand in hand as far as both are corresponding disciplinary practices, the
former focused on markets and the latter focused on bodies disenfranchized by economic
restructuring. Importantly, Nevins does not suggest that this coincidence is intended, but rather that economic
liberalization has compelled the militarization of the U.S. Southwest border as migrants are pushed
northwards. In this sense, Nevins considers the patterned meeting of U.S. geopolitical and geoeconomic policies in the region
rather than their intended coherences.

The impact is unending warfare waged against hostile others
Coleman, 05 (Matt Coleman Department of Geography, UCLA, 2005, U.S. statecraft and the U.S.Mexico border as
security/economy nexus) MD
Importantly, the border as security/economy nexus suggests a geography of statecraft in which geopolitical and geoeconomic power
is not a singular, coherent capacity neatly pooled up at the center of the sovereign state (Agnew, 1999; Jessop, 1990; Murphy,
1996) and then deployed spatially, as if the state is a juridicoeconomic whole (Gordon, 1991) of symbiotic legal-military and market
access projects with the border as its place of focus. Rather, as a Janus-faced geography (Anderson & ODowd, 1999), the
border as security/economy nexus is literally a strategic terrain where countervailing projects of
statecraft come to bear on one another. This brings us back to the problem of the nonlocal production of the border
introduced at the outset of this essay. As designs conceived nonlocally in which the border gures from
afar as an uncomplicated landscape amenable to either rebordering or debordering national
interests, U.S. geopolitical and geoeconomic practices are much more complicated at the local
scale where they come face-to-face with one another. From the remote perspective of U.S.
congressional lawmakers and executive policymakers the border might appear to be a simple, instrumental
frontier open to geopolitical and geoeconomic takeovers. However, these nonlocal territorial
intentionalities of control (Yuval-Davis, 2003) come apart in terms of the local circumstances each
produce. In this sense, U.S. geopolitics and geoeconomics in the border region, although clearly
geographical in their respective colonizations of the border as a malleable frontier, are anti-
geographical in that their instrumental mappings of territory obscure the complex place-based
realities of their deployment (Dalby, 1998; O Tuathail & Agnew, 1992). It follows that U.S. statecraft in
the borderlands can be read as a fraught bundle of geopolitical and geoeconomic storylines
rather than as a coherent sovereign script (O Tuathail, 2002). But such storylines are about more
than competing and countervailing practices in a narrow sense because statecraft is not simply
about strategy. For instance, on a cultural register, statecraft is about how an imputed body politic comes
to know and celebrate itself in relation to identications of danger (for example, see Burke, 2002; Weldes,
Laey, Gusterson, & Duvall, 1999). If we note, then, that geopolitical and geoeconomic practices are not just
organizational phenomena but also articulations of identity (Albert, Jacobsen, & Lapid, 2001; Paasi, 1996),
to admit the convoluted character of U.S. geopolitical and geoeconomic practice implicating
the U.S.Mexico border region is to call into question not just the concept of U.S. foreign policy
as a coherent mode of government, but further the notion of a singular identity informing U.S.
statecraft. Recent work by Mead (2001) helps to unpack the multiple identities undergirding U.S. geopolitical and geoeconomic
strategy in the U.S.Mexico border region. For example, the geopolitical representation of the border as a
geography in need of policing discloses a realist Jacksonian identity in U.S. statecraft. In this
cultural mindset, the border marks a gap between an exceptional, popular domestic realm of
citizenship and community, and an anarchic outside world of danger to be kept at bay through
heavily militarized borders. This Jacksonian impulse understands its beleaguering others in
the last instance to be neither accommodating nor redeemable. The result is a conviction that
hostile others a confused collection of undocumented migrants, narcotics trackers,
criminal aliens, and terrorists, dened in conated foreign policy and public policy contexts
need to be fought o through sustained frontier warfare in the form of hard borders and tough
immigration legislation.

2NC BORDER SECURITY

The aff ignores the personal experiences of forced migrants through the
construction of the border as a state-less geographical zone that decides who
stays in and who stays out this form of apartheid politics makes racism,
exclusion, and violence inevitable
Gill 2010 (Nick. "New state-theoretic approaches to asylum and refugee geographies." Progress in Human Geography 34.5
(2010): 626-645. http://phg.sagepub.com/content/34/5/626.short BRW)
Playing upon the symbiotic relationship between refugees and states, Hyndman and Mountz draw upon the work of Giorgio
Agamben (1988) to identify the ways in which the forced migrant is routinely placed outside formal state
spaces, in an increasing variety of exceptional zones and sites at which normal legal protection is
suspended. The strategic non-presence of the state allows states to simultaneously commit to a range
of progressive international agreements concerning the rights of migrants and then to avoid the
responsibilities that inhere in these agreements through the maintenance of zones of uncertainty and
legal ambiguity. Perversely, the additional policing and security measures that extra-territorial, extra-state zones require, such
as the networks of carceral institutions for terror suspects that are ostensibly outside normal
legislative processes, means that the very places that claim to be absent of the state are often home
to more state actors and state-like institutions than most areas and people within state
jurisdictions and protection. Hence, a focus on the strategic absence of the state promises not only to offer important
lines of inquiry relating to the treatment of asylum seekers who are positioned in ambiguous non-places and understood in terms of
the non-categories that result, but also to problematize claims to the presence or non-presence of the state itself. Another way in
which geographers are rethinking the relationship between state and society is through the
concept of governmentality. As Brown and Boyle (2000) suggest: State power is no longer simply the
power to wage war or pass laws, it also lies in very ordinary, mundane bureaucratic practices.
Specifically: a states own knowledge of its population powerfully frames the conditions and terms
through which its citizens can see themselves as a nation. In this way, they come to govern themselves
through the states mentality. (Brown and Boyle, 2000: 89) Govermentality offers a way to understand how individual behaviours
in asylum and refugee contexts can be elicited by the state, not through any legal or forceful activities, but by engendering within
subjects the desire to conduct themselves in one way or another. Such an ability extends the more common understandings of
governance, which may rely upon financial or forceful means of eliciting the self-policing of subjects, to encompass a range of
different forms of power, such as persuasion, seduction and ideological inculcation, that generate a deeper degree of autonomization
among subjects (Allen, 2003; Gramsci, 2006). This approach foregrounds not only the psychology of state actors, but also the
competing influences over this psychology, indicating the importance of social allegiance to state programmes. Silvey (2007)
explores the importance of the development of governmentality within migration studies, pointing out the ability of governmentality
studies to break free from the realist confines of coercive state power and its consequent ability to reveal the importance of local,
situated decision-making and the struggles that occur around these. Once again, the contested role of actors and agency are
reiterated in her description of the growing engagement with governmentality in the migration field: Whereas much classical work
emphasizes states manifestations of centralized, sovereign power, the growing body of critical work highlights governmentality and
the dispersion of power beyond formal state apparatuses. For migration research, this analytical shift
encourages greater attention not only to discursive production of migrants bodies, national borders,
and citizen subjects, but also to the everyday mediations of exclusion/inclusion by actors involved in
these circuits of migration and governance. (Silvey, 2007: 268) The actors involved in circuits of migration and
governance therefore become key sites of contestation and resistance in their own right when a governmentality
perspective is adopted. The volitional allegiance of actors within the asylum sector is contested through such governmental
techniques as institutional cultures (Du vell and Jordan, 2003), the language with which the asylum issue is discussed and debated
(Turton, 2003), media depictions of asylum seekers (Cwerner, 2004; Finney and Robinson, 2007), policy document representations
(Malkki, 1996; Weber, 2003) and their spatial and temporal management (Gill, 2009). It is the combination of these techniques that
can enlist influential asylum-sector actors into conducting themselves in ways that they envision are state-serving. The breadth
of interventions made by political scientists, sociologists and geographers highlights the fact that
the dominance of an essentialist notion of a strict division between state and society, and the
consequences of this division, are being radically questioned. While it is clear that there remain some
significant differences between the various reactions to the consequences of state essentialism in the study of asylum migration,
there are nevertheless commonalities in the approaches of radical commentators that point towards the emergence of a
distinctively different intellectual project that seeks to provide an alternative way of
understanding the relationship between states and forced migrants. At the risk of generalization, it
seems useful to sketch out these key characteristics of an emerging critical asylum geography.

Southern border securitization is an attempt to separate the south from north
Bialasiewicz, 11 Department of Geography, Royal Holloway University of London (Luiza Bialasiewicz, 2011, Borders above
all, Political Geography 30 (2011) 299300)//ah
Such fears were driven by a shocking geographical realization, as an editorial in Italian newspaper La
Repubblica put it: that the southern shore is very close. It is practically here (Diamanti 2011). Too close,
despite all the attempts of the previous years to transform the Mediterranean into a tightly sealed wall and to keep the
southern neighbours as far away as possible. The securitisation of EUropes southern borders has
become a fundamental EU priority. For instance, the European Neighbourhood Programme, launched in 2003
with the aim of fostering stability and peace at the Unions external borders by creating a ring of
friends, increasingly has moved to an explicitly security-led agenda. Migration control in particular
has become a key priority of the ENP, with EUropes neighbours increasingly called upon to act as EUropes policemen
(see Guild 2010, and the special section of Geopolitics, 2011). Henk van Houtum (2010) has described this new buffer zone
geopolitics as the progressive installation of a cordon sanitaire at the borders of the Union. It is
a cordon sanitaire whose main aim it has been to keep dangerous ows (of irregular migrants or other un-
wanteds) from even approaching the borders of the Union, with increasing emphasis on the off-shoring and out-
sourcing of EU border management (Bigo & Tsoukala, 2008; van Houtum & Pijpers, 2007). The de-bordering and externalisation
of EUropes borders has been different from similar developments in North America, however, if only because the EUs border-
work has been implemented thus far through a uid assemblage of agreements and actors, with considerable slippage between the
bordering practices of Member States and what is done on behalf of the Union.

Immigration policy is the spawn of biopolitical management of populations
Wiebel, 10 Phd from University of Iowa (Jon Christopher Wiebel, 2010, Beyond the border: on rhetoric, U.S. immigration,
and governmentality http://ir.uiowa.edu/cgi/viewcontent.cgi?article=2091&context=etd)//ah
While Andreas' research illustrates how contemporary border enforcement strategy reflects a concern over the mobility of
populations, it stops short of considering how policies implicated in immigration control function to govern,
in the sense of shaping and managing the conduct of, populations both inside and outside the territorial
boundaries of the United States. This is due to a modern conception of police in which police is simply "the instrument
by which one prevents the occurrence of certain disorders" (Foucault, 2007, p. 354). This is most evident when Andreas concludes
that border controls along the U.S.-Mexico boundary are "less about military defense . . . and more about the policing of CTAs
[clandestine transnational actors], with terrorists, drug traffickers, undocumented migrants, and smugglers leading the list of state
targets" (p. 107). As such, policing is conceptualized by Andreas as a practice oriented toward the prevention of disorder and
illegality. Immigration policy merely exists as a mechanism that imposes "constraints upon
particular bodies (Rose & Miller, 1992, p. 174). But what if we are to take seriously Connelly's (1996) suggestion that through
migration policy the state involves itself in shaping the mobility of a population? Can the emphasis on
policing within border enforcement strategy be understood not just as an attempt to eliminate what is taken to be a particular
disorder (illegal immigration), but also as an attempt to manage and shape the movement of populations? scholarship of the
function of police within liberal democratic states points to such a possibility (Dubber & Valverde, 2008). Anchored in Foucault's
concept of governmentality, scholars from a variety of disciplines have sought to understand police power as
a "modern technology of governance" oriented toward "maximizing the public welfare . . . as opposed to, say, the
power to do justice" (Dubber & Valverde, 2008, p. ix). Of particular importance is Levi's (2008) analysis of anti-crime ordinances
implemented by the city of Chicago in the early 1990s. Designed to eliminate the loitering of gang members which was characterized
as a method for intimidating local residents, Levi highlights how these ordinances 12 functioned to do more than eliminate a
particular disorder. Analyzing the discourses associated with the ordinances, Levi unpacks a conception of police that takes policing
to be part of "an array of techniques for governing" (p. 181). Put another way Levi considers how the Chicago anti-crime ordinances
sought to influence the capacities and activities of population in ways other than the imposition of coercive force. This is epitomized
by his claim that these ordinances "enjoyed little success; however, they did manage to keep people from becoming idle" (Levi, p.
189).

Bordernlands are perceived the wild zones of the global south through this
conceptualization wars have become blurred and rendered inevitable
Gregory, 11 PhD, Prof. at Department of Geography, University of British Columbia (Derek Gregory, May 2011, The
everywhere war The Geographical Journal, Vol. 177 No. 3, pp. 238250)//ah
Dufeld (2001, 309) once described the borderlands as an imagined geographical space where, in the
eyes of metropolitan actors and agencies, the characteristics of brutality, excess and breakdown
predominate. There, in the wild zones of the global South, wars are supposed to occur through
greed and sectarian gain, social fabric is destroyed and developmental gains reversed, non-
combatants killed, humanitarian assistance abused and all civility abandoned. This imaginative
geography folds in and out of the rhetorical distinction between our wars wars conducted by
advanced militaries that are supposed to be surgical, sensitive and scrupulous and their wars.
In reality, however, the boundaries are blurred and each bleeds into its other (Gregory 2010). Thus the US-
led invasion of Afghanistan in October 2001 combined a longdistance, high-altitude war from the air with a ground war spearheaded
by the warlords and militias of the Northern Alliance operating with US infantry and Special Forces; counterinsurgency in
Afghanistan and Iraq has involved the co-option of ragtag militias to supplement US military operations; and in Afghanistan the US
Army pays off warlords and ultimately perhaps even the Taliban to ensure that its overland supply chain is protected from attack
(Report of the Majority Staff 2010). In mapping these borderlands which are also shadowlands, spaces
that enter European and American imaginaries in phantasmatic form, barely known but vividly
imagined we jibe against the limits of cartographic and so of geopolitical reason. From Ratzels view
of der Krieg als Schule des Raumes to Lacostes stinging denunciation la gographie, a sert, dabord, faire la guerre the
deadly liaison between modern war and modern geography has been conducted
in resolutely territorial terms . To be sure, the genealogy of territory has multiple valences, and Ratzels Raum is
not Lacostes espace, but a critical analysis of the everywhere war requires cartographic reason to be
supplemented by other, more labile spatialities. This is not only a matter of transcending the
geopolitical, connecting it to the biopolitical and the geo-economic, but also of tracking space as
a doing, precarious, partially open and never complete. It is in something of this spirit that Bauman (2002, 83)
identies the planetary frontierlands as staging grounds of todays wars, where efforts to pin the
divisions and mutual enmities to the ground seldom bring results. In the course of interminable
frontierland warfare, so he argues, trenches are seldom dug, adversaries are constantly on the move and have become for all
intents and purposes extraterritorial. I am not sure about the last (Bauman is evidently thinking of al Qaeda, which is scarcely the
summation of late modern war), but this is an arresting if impressionistic canvas and the uidity conveyed by Baumans broad
brush-strokes needs to be eshed out. After the US-led invasion of Iraq it was commonplace to distinguish the Green Zone and its
satellites (the US political-military bastion in Baghdad and its penumbra of Forward Operating Bases) from the red zone that was
everywhere else. But this categorical division is misleading. The colours seeped into and swirled around one another, so that
occupied Iraq became not so much a patchwork of green zones and red zones as a thoroughly militarised landscape saturated in
varying intensities of brown (khaki): intensities because within this warscape military and paramilitary violence could descend at
any moment without warning, and within it precarious local orders were constantly forming and re-forming. I think this is what
Anderson (2011) means when he describes insurgencies oscillating between extended periods of absence as a function of their
dispersion and moments of disruptive, punctual presence, but these variable intensities entrain all sides in todays wars amongst
the people and most of all those caught in the middle.





1NC DRUGS

The borders of the US and Mexico have been transformed into the first worlds
open wound the war on drugs has become the guise by which the State
attempts to manage the inferior migrant population
Gregory, 11 PhD, Prof. at Department of Geography, University of British Columbia (Derek Gregory, May 2011, The
everywhere war The Geographical Journal, Royal Geographical Society, Vol. 177 No. 3, pp. 238250)//ah
The United StatesMexico borderlands are an ambiguous space too, Amexica, famously described
by Anzalda (1987, 25) as una herida abierta [an open wound] where the Third World grates against the First
and bleeds. Before a scab can form, she continued, it hemorrhages again, the lifeblood of two worlds merging to form a third
country a border culture. Coleman (2005) sees the border as a trickster gure, at once being opened to the passage of capital and
commodities under the sign of neoliberalism and closed to the movement of migrants who are often themselves victims of
neoliberalism, and he is right to treat de-bordering and re-bordering as a tense and countervailing constellation of transnational,
national and local practices. But Anzaldas original sanguinary metaphor seems ever more appropriate as the border has
come to be performed as the front-line in what Vulliamy (2010, 12) calls the rst real twentyrst
century war because, he says, it is also a postpolitical war. What he has in mind is the trans-border war on
drugs, but his characterisations fail to capture the twentieth-century histories that are embedded in the conict, the
intimate connections between narcotrafcking and the Mexican state, and the ways in which this increasingly
militarised campaign forms one plane in a multi-dimensional battlespace where drug
trafckers and undocumented migrants are being transformed into insurgents
and terrorists . All of these violent geographies are freighted with political implications. In
the 1980s the major drug trafcking routes into the United States from Central and South America ran through the Caribbean, and
there have been persistent (with the Institute of British Geographers)claims that CIA support for the Contra rebels in Nicaragua
involved at least tacit support for an ongoing arms-for-drugs exchange. The increasing public scrutiny of these arrangements
combined with the success of counter-narcotics operations to prompt Colombian drug cartels to develop new trafcking routes
through Mexicos border cities. These routes were controlled by one major Mexican trafcker, Flix Gallardo, but when he was
imprisoned in 1989 he had his lieutenants divide the border into territories and reach an accommodation with Mexican authorities
so that they could concentrate on ghting what he saw as the real enemy: the agents of the United States. These arrangements soon
broke down. Fighting between the new regional cartels spiralled into a battle for prots through territorial expansion, and as the
violence intensied the state militarised its response. Federal troops had long been used to destroy marijuana and poppy elds in
rural areas, especially in the mountainous Golden Triangle that spans Chihuahua, Sinaloa and Durango, but despite their raids
domestic production soared. The army was ill-prepared for the switch to interdiction in cities, where its record proved even more
mixed and the consequences far bloodier: more than 35,000 people have been killed in the last 4 years,
more than the toll in Afghanistan over the same period. Fifty thousand federal troops and thousands more
private security contractors, many of them employed by US security companies, are now deployed. The conventional and I
dare say dominant reading treats all these deaths as conned to those caught up in the drug trade.
Leaving on one side those who are literally caught, trapped in the trade by spiralling circles of
poverty set spinning by the rapid neoliberalisation of the economy, some Mexican scholars insist that the
victims include human rights activists, community leaders and labour organisers. Certainly, Mexico is
no stranger to military repression. During the dirty war from the 1960s through to the 1980s, the Army was given
carte blanche to put down student demonstrations and guerilla groups, and it carried out disappearances
and illegal detentions, torture and killings on such a scale that the United States noted an emerging
security problem. The cloak for these bloody operations was the Cold War, and some scholars believe that the
drug war now serves as a convenient cover for the renewed criminalisation of social protest. When President Calderon
describes the campaign as a war, therefore, the word is freighted with layers of political meaning. He and his ministers
constantly speak in these terms, and Calderon has even compared the ght against the cartels to Mexicos celebrated defeat of an
invading French expeditionary force on 5 May 1862; but for many Mexicans the reverberations are more recent than Cinco de Mayo.
Yet the declaration has to be seen as something more than the intensied militarisation of
security. It is no longer a matter of organised crime, El Universal declared in a June 2010 editorial, but rather
of the loss of the state. Calderon said much the same on 4 August 2010: it [has] become a challenge to the state,
an attempt to replace the state, he claimed, because the cartels are trying to impose a monopoly by
force of arms, and are even trying to impose their own laws.




1NC GREEN TECH

Economic engagement over green issues greenwashes the status quo of hegemonic
inequalities and frenetic consumerismthey merely reproduce existing
geopolitical relations which makes extinction via sacrificial politics inevitable.
Luke, 08 (Timothy W, Department of Political Science at Virginia Polytechnic Institute, The Politics of True Convenience or
Inconvenient Truth: Struggles Over How to Sustain Capitalism, Democracy, and Ecology in the 21st Century, Environment and
Planning, Vol. 40, pg 1811-1824//[SG])
This analysis is a brief critique of market-friendly, growth-perpetuating natural capitalism, particularly that
present form which recognizes the past excesses of commercial development as the most likely causes of global climate change, as it
revalorizes this industrial waste to concentrate and accelerate future commercial expansion. Such efforts seek to maintain the
dynamic destructive developmental project of global exchange without making any radical criticism of
its intrinsic inequalities, embedded waste, and frenetic consumerism. A thorough critique of this economic
formation is a much larger undertaking than can be completed in one paper. Some of the key contradictions and basic conflicts,
however, can be typified in a close critical reading of the broader cultural politics and political economy expressed in the
environmental advocacy and thinking of Al Gore Jr, especially in his Nobel Prize winning activities on climate change in works like
An Inconvenient Truth (2006a) or Earth in the Balance (2006b). Such a critique is meant neither to dismiss the dangers of global
climate change nor to derogate the findings of ongoing scientific research, like that done by the co-awardees of the 2007 Nobel Peace
Prize with Al Gore, namely, the Intergovernmental Panel on Climate Change (IPCC). Instead, it is meant to begin a pointed
reassessment of how today's global climate-change debates often are too entangled in the reproduction of
existing power relations. At best, they often green-wrap corporate technocracy with renewed
institutional legitimacy that `greenwashes' an unsustainable economic status quo in the refreshing, but
not cleansing, waters of sustainable development. While there is a need for systemic reforms in economic regulation, technological
innovation, and social distribution, Gore's many engagements with big business, venture capital, and global media do not seem to
promise such a transformative change. For some so-called US neoconservatives, especially in the days after September 11, 2001,
redefining America's global `responsibility' to manage the current world system of states, economies,
and societies became an almost obsessive calling. To Kagan (2003, page 96) this necessity was quite clear: everyone in the
world must acknowledge and accept ``the new reality of American hegemony''. Others, like President George W Bush [43rd US
President, hereinafter Bush (43)], determined this moment as a historic turning point where America must extend ``the benefits of
freedom across the globe ... to bring the hope of democracy, development, free markets, and free trade to every corner of the world''
(Bush, 2002, page 2). This hegemony was regarded as a true convenience for Washington, but its risky workings also hold
Earth in the balance with a host of inconvenient truths, including philosophical and political struggles
over how to sustain capitalism, democracy, and ecology (Beck, 1992; Gore, 2006b; Luke, 1999) in the 21st
century. For some the world will not be enough until Bush's vision is made real; for others the planet might not survive
long enough for all to even see any of this action. Something went awry for Kagan, Bush, and the USA on the road from
Kabul to Baghdad. Nonetheless, one must not mistake the reversals encountered by the Bush (43) administration in Iraq as a sign
that all Americans will shrink from the struggles entailed by managing world affairs. Indeed, President Bush (43)'s defeated
Democratic opponent from 2000, Al Gore Jr, arguably is thinking even bigger, better, and bolder thoughts about this task by
outlining his own program of moral imperatives for the USA to serve as the planetary protector of the Earth and all its human and
nonhuman inhabitants. An ironic, but inconvenient, truth about Gore's long march through the institutions of the American
government (Gore, 2006a; 2006b) is how much his An Inconvenient Truth also aims, albeit after taking a softer path, ``to bring the
hope of democracy, development, free markets, and free trade to every corner of the world''. In fact, Gore's worries (1992) about
Earth hanging ``in the balance'' of ecological collapse appear to be morphing the old ``land ethic'' of Aldo
Leopold into a new ``planetary ethic'' for managing the entire planet from the United States of America
for maximum ecological and economic sustainability (Luke, 2005a, pages 228 ^ 238). This analysis critically
examines one very visible version of such a `planetarian ethics', namely, the works of Al Gore on the ``inconvenient truth'' of global
warming (Luke, 2005b, pages 154 ^ 171). By reconsidering how Gore articulates his view of the truth, to whom this truth appears to
speak, and what social forces are likely to embrace his truth statements for dealing with the Earth's environment at a planetary scale
of operation in various world scientific, economic, and political organizations, it also assesses the struggle over sustaining
development and democracy in this century. While it appears morally just, one must ask if projects like Gore's Alliance for Climate
Protection, or the recently announced Global Roundtable on Climate Change (GROCC), might only continue existing strategies for
global economic growth that still serve inequitable geopolitical agendas after the missteps of the Bush (43) administration from New
York to Baghdad to New Orleans. In a sense, much of this planetarian ethic was captured by the Nobel Prize Committee in its 2007
Peace Prize statement as it praised the IPCC and Albert Arnold (Al) Gore Jr ``for their efforts to build up and disseminate greater
knowledge about man-made climate change, and to lay the foundations for the measures that are needed to counteract such change''
(Norwegian Nobel Committee, 2007). Indeed, who builds up this knowledge for whom, and then how, what, when, and why
counteractive measures are to be made are central concerns for a planetarian politics in the coming decades. The intellectual roots
spreading underneath Gore's program spring from common `sustainable development' thinking. In other words, how can the US
sustain the true convenience of national prosperity amidst an environment facing too many inconvenient truths regarding the
inequitable and irrational use of energy, resources, and information in a new global economy (Luke, 2005a)? That sustainable
development is an ecopolitical project which might be neither sustainable nor developmental becomes irrelevant (Friedman, 2006).
It is a palatable approach to ``green-wrap'' the economic and political project of ``sustainable degradation'' (Luke, 2006, pages 99 ^
112) already now fully in play. Arguably, the world has already overshot its renewability limits since the 1970s. Memorable events
like Earth Day 1970 and 1990 arguably bookend those decades in which cycles of sustainable degradation became a material reality.
In his writings, Gore recounts how he too has fretted over ecological losses, which were first marked by others over thirty, forty, or
fifty years ago (McNeill, 2000). Still, he claims that conditions today are dangerously different. Quantitative increases in many
industrial pollutants are adducing qualitative changes in the Earth's environment and the equity of its global economy (Lamb, 1977;
Long, 2004; Maslin, 2004). Hence, the next decade is a decisive conjuncture that demands radical action.
1NC HUMAN TRAFFICKING

The affs attempt to prevent human trafficking rests on the assumption that the
state is an ethical and rational actor, ignoring the this epistemology sanitizes
state directed violence and exclusion while incentivizing the very evils the aff
claims to solve.
Gill, 10 (Nick. "New state-theoretic approaches to asylum and refugee geographies." Progress in Human Geography 34.5 (2010):
626-645. http://phg.sagepub.com/content/34/5/626.short BRW)
First, the debate surrounding how states should respond to the situation of the worlds refugees is structured around the ethical
dilemma that asylum seekers and refugees pose to recipient states (Schuster, 2003; Gibney, 2004; Ruhs and Chang, 2004). On the
one hand, cosmopolitans argue that the state should not accord primacy to the rights of citizens
over non-citizens (Singer and Singer, 1988; Singer, 1993). On the other hand, particularists argue that the states very
function is to further the interests of citizens, even at the expense of non-citizens if necessary (Hendrickson, 1992). A number of
academics have offered compromises between these ethical poles, such as the extension of asylum to those in greatest need
(Dummett, 1992), to those nearest to receiving states either geographically or culturally (Walzer, 1983; Miller, 1988), or
indiscriminately, but only up until the point at which an unacceptably adverse impact upon incumbent nationals welfare is
experienced (Gibney, 2004). All sides of this debate, however, depicts the state as a deliberative rational
actor, located outside society and capable of making relatively clean interventions into the social
realm on the basis of moral or ethical principles, without compromising its own distinction from
the social order. By using this conception, the ethical debate employs a conceptual separation between
state and society, bestowing the former with competences and capacities that are independent from, and ontologically prior to,
the latter. This seminal ethical debate may have structured and delimited the ways in which forced
migration and refugee issues have subsequently been conceptualized both within and beyond
geography. Second, numerous academic commentators have suggested that, all other things being equal, it is in western,
developed destination states interests to welcome refugees for the economic benefits they offer to host societies. The projected
contraction of the working aged population of the EU-25 from 67 % to 57 % of the total by 2050, while the number of people aged
over 65 simultaneously rises from 16 % to 30 % , underpins the European demographic case (Castles, 2006). Refugees who have the
social and financial capital, as well as the personal resilience, to escape violent situations represent a particularly welcome addition
to the workforce (Stewart, 2003; van Hear, 2004). From the perspective of an instrumental state, this is especially true if refugees
and asylum seekers occupy insecure labour-market positions, allowing firms to exploit a highly skilled population at relatively low
cost (Samers, 2005). Against these arguments for asylum-seeker entry, post-Fordist firms have been depicted in some studies as
relatively internationally mobile, capable of locating labour-intensive production functions wherever labour is cheapest, thereby
dispensing with the requirement to import cheap labour and with the need to exploit migrant communities (Cohen, 1987; Krugman,
1995). Others have argued against the exploitation of migrant labour on the basis that labour unrest can be highly disruptive and
inefficient (Bradley et al ., 2000), as well as socially costly (Castles and Miller, 2003; Mahnig, 2004). Still more have suggested that
the proliferation of part-time, flexible working practices in mainstream, western economies has been sufficient to meet the need for
flexibility and cheap labour (Marie, 2000; Williams and Windebank, 2001). Immigrants effect upon native wages constitutes further
grounds for concern. Although a small number of authors argue that the impact of immigrants upon native wages is significant
(Borjas, 1999; 2006; Angrist and Kugler 2001), most authors conclude that the effect is negligible (Bean and Stevens, 2003;
Cornelius and Rosenblum, 2004; Hatton and Williamson, 2004; Constant and Zimmermann, 2005). They nevertheless see the
unequal distribution of the competitive wage effects of immigration as problematic, however, because low-income households tend
to bear a disproportionate share of the effect (Scheve and Slaughter, 2001; Cornelius and Rosenblum, 2004; Hix and Noury, 2007).
Importantly, even if this is not always the case or is not always significant, the very perception that immigrants are associated with
wage declines is enough to render the support of liberal borders an exceptionally risky political strategy that may be enough to
deliver substantial support to the political Right (Swank and Betz, 2003). Even throughout this debate, however, the state is
essentialized. Assessing the pros and cons of supporting asylum migration from the perspective of states interests makes the
peculiar assumption that there is such a thing as a national interest. In reality, the difficulty of accessing the preferences of national
populations is endemic, even within a liberal democracy (Dunn, 1992). Furthermore, the idea that the institutional and legal
mechanisms of the state are representative of national interests, even if such interests could be derived, has been lambasted by left-
wing scholars. They draw attention both to the ways in which the state apparatus can be appropriated by classes and factions
(Miliband, 1973) and to the inbuilt tendenciesofstates, whoeverruns them, toprivilege capital-owning classes (Poulantzas, 1978;
Jessop, 1990). For these reasons, the state is liable to systematically underrepresent different factions and classes within society,
making the notion that state policy towards asylum seekers represents objective consensus appear extremely precarious. Once again,
academics working on these issues may have endured a dearth of work that radically questions the assumptions upon which key
debates are predicated A third debate about asylum seekers and refugees concerns the effectiveness of policies designed to control
asylum flows (Neumayer, 2004; Samers, 2004). There is widespread disagreement between academics about the effectiveness of
state policy in a globalized world. On the one hand, there are a number of reasons to view nation states as commanding effective
control over their borders. The sheer magnitude of public expenditure indicates, in the first instance, that government policy must
be having an effect. Surely the public sector in the UK, for example,could notbesowastefulastopour over 2 billion a year into a lost
cause? Accordingly, some quantitative studies provide support for the efficacy of states in maintaining borders. Hatton (2004) for
example, analyses asylum migration from Africa, Asia and eastern Europe to 14 western European countries and finds that the
implementation of a single deterrence policy is associated with a 10 % decline in requests for asylum to the state that has
implemented the policy. On the other hand, transnationalist scholars have emphasized the global structural factors that prevent
states from operating successful border-control policies (Castles, 2006; Koser, 2007). Transnationalism takes as its point of
departure not the landscape of national state territories, but the routes, networks and patterns of migrants and migrant
communities. As social, economic and cultural linkages between communities that are located in different countries have
strengthened, due in part to the communicative and technological developments that produce globalization, transnationalists have
been able to point out the increasing unsuitability of nation states as an appropriate lens through which to understand and take
stock of these developments (Black, 2001; Al-Ali and Koser, 2002; Castles, 2004; van Hear, 2006; Koser, 2007). In their view, the
transnationalization of migratory patterns has served to undermine the nation state as a discrete destination or container of migrant
experiences, communities and networks (Koser and Pinkerton, 2002; van Hear, 2002). Transnationalists can also cite numerous
quantitative studies that support their point of view (Bocker and Havinga, 1998; Cornelius and Rosenblum, 2004; Thielemann,
2004). What is more, the evidence of the complementarity between globalization and migrant smuggling networks underscores the
difficulties globalization presents for immigration control (Salt and Hogarth, 2000). As globalization has gathered pace,
commentators have argued, the cost of organization and performing cross-boundary smuggling operations has fallen. At the same
time, the potential gains have risen due to the erection of a profusion of legal and physical barriers to entry,
designed precisely to curb smuggling activities. This cost structure has incentivized human
smugglers, who are consequently beginning to operate with more sophisticated business models
and on a larger scale(CohenandRai,2000;Koser, 2007). While compelling, however, these disagreements serve, once again,
to reify the statesociety divide. Academic commentators on both sides of the debate often assume borders to be both national and
under the exclusive control of the state. This abstracts from broader processes of asylum-seeker exclusion by privileging the national
scale and by factoring out processes of asylum-seeker exclusion that may not be statebased or state-driven. This indicates a need for
a set of theoretical concepts that allow us to examine in greater detail the role of social factors in the exclusion
of asylum seekers at both subnational and supranational scales as well as through geographical
concepts that draw more upon relationality, network and place than upon the often artificially
constructed category of scale (Guild, 2002;Marston et al .,2005).By exclusively examining national border-
control policies, the debate about their effectiveness threatens to obscure alternative drivers of
asylum-seeker exclusion. What is more, the assertion that border control does not work because of the transnationalization
of asylum-seeker flows pitches a sophisticated understanding of migration patterns and flows against an impoverished, overly
territorial theory of the state. Transnationalists who argue that networks of migrant routes and communities do not follow the
administrative boundaries of states (and that states are therefore ill equipped to control them), use an implicit theory of the state
that models state power as static, contained by its own borders and constrained by its own boundaries (for a critique, see Taylor,
2003). In this conception of the state, state power is relatively immobile, territorially rooted and incapable of networked
transmission of its own. By employing a territorial, contained concept of the state, transnationalists preclude the possibility that the
state is capable of mirroring the transnationalization of migrant flows by working through the commitments and comportment of
dispersed social actors themselves (see, for example, Larner, 2007). This observation challenges static, territorial views of the state
and state power and confuses any clean distinction between state and social domains. The risk from the perspective of
geographers engaging in these fields is that to enter into the debate about the effectiveness of government policy in
controlling asylum and migration flows is to take on, and thereby to ratify, the implication that state controls are
essentially national in scale and essentially immobile. A fourth debate that has received widespread attention
within and beyond geographical engagement with asylum-seeker and refugee issues is the degree to which states are free to put their
policies to work, even if they do command effective border-control mechanisms. Numerous authors have claimed that the
proliferation of human rights norms and rules, embodied in humanitarian treaties such as the Geneva Convention (1951) and the
Universal Declaration of Human Rights (1948) have begun to impose increasingly stringent constraints upon the sorts of activities
nation states are authorized to engage in when policing their borders (Hollifield, 1992; Jacobson, 1996; Soysal, 2004). With the
cooperation of international, legally recognized institutions such as the European Union, the European Court of Human Rights and
the United Nations, the implications of these treaties are slowly being rolled out into national law (Ife, 2001; Nicol, 2004). A number
of studies have raised objections to this view. The notion that national sovereignty has been ceded to international levels has been
problematized by studies that examine the ways in which international collusion has actually served national interests, both in terms
of the pursuit of an imagined, shared security agenda and in terms of legal legitimacy (Cholewinski, 2000; Lavernex, 2001; Byrne et
al ., 2003). Others have drawn attention to the ability of nation states to opt out of humanitarian rules or to ignore them in the
absence of credible enforcement mechanisms (Hathaway, 1990; Schuster, 2003; Welch and Schuster, 2005). A final, more
surprising, objection is that states have actually begun to refer to international legal obligations to avoid their previous
responsibilities. Rather than ignoring international humanitarian rules, states have been able to defer responsibility upwards for a
rangeof issues relating to asylum, especially in the legal sphere (Nicol, 2004). Where the idea of constraints acting over states may
have enjoyed more currency, however, is in the context of domestic, internal resistance to tough immigration policies. Immigrant
communities, for example, have been argued by some political theorists to constitute an increasingly powerful lobbying force that is
able to frustrate states aspirations for tougher immigration policies from within (Freeman, 1995; Money, 1999). According to this
school, the alignment of migrants interests and the interests of the private sector in securing relatively free access and the
minimization of state intervention has been seen to provide migrant communities with powerful allies in their pursuit of liberal
borders, affording them privileged access to the state apparatus (Freeman, 1995; 2001). Furthermore, migrants have been supported
by the emergence of a vocal coalition of liberal activists, lobbyists and civil society organizations (Castles and Miller, 2003). This has
rendered the state as exposed to charges of racism and xenophobia as it is to charges of excessive liberalism (Solomos, 1993; Jupp,
2002; Schuster and Solomos, 2004). These domestic checks upon states treatment of migrants are particularly potent during
periods of social unrest and high immigrant unemployment that constitute burdens to the state as well as to the host society (Studlar
and Layton-Henry, 1990; Mahnig, 2004). This liberal coalition has gained further momentum as trade unions have reviewed their
traditionally restrictive stance on immigration (Goldin, 1994). Labour movements are increasingly choosing to see immigrants as
potential new members in the face of declining domestic support, rather than as threats to native workers (Haus, 2002; Watts,
2002). This is especially welcome in labour markets that are poorly regulated, where the distinction between economic migrants and
refugees or forced migrants often becomes blurred, meaning that forced migrants can become reliant upon union support despite
the fact that they should not officially be working (see Wills, 2005). 1 Joppke (1998: 59) consequently concludes that [N]ot external,
but internal constraints have prevented liberal states from shielding themselves completely from global refugee movements. Yet
again, however, these debates are framed in such a way as to reify the division between state and society. Although this debate comes
closest to recognizing the blurred distinction between state and social forces, through the importance of internal resistance that
operates from within the state apparatus to exert power over it, the very language of constraints implies an
antagonism between social and state spheres that presupposes their distinction. By
conceptualizing and talking about constraints a sense is preserved in which there is a
continuing separation between society and state because it is only by virtue of this separation
that the former can be opposed to the latter. In all four debates, therefore, participants often make
assumptions about the state that frame the discussions that have taken place. In general, the state is
assumed to occupy a separate position to society and to regulate it from a position of exteriority, usually at the national level. III
The consequences of an essential state concept State theorists have been critical of the assumed separation of state
and society (the so-called separate spheres assumption) (Abrams, 1988; Mitchell,1999;Ferguson and Gupta,2002). This distinction
imagines a relatively autonomous sphere of the state that intervenes in, regulates or affects
another autonomous sphere labelled society (Sharma and Gupta, 2006). Central to concerns over the separate
spheres assumption is the observation that the boundary between state and society has proven remarkably elusive over a number of
decades of state focused research. Although various scholars have attempted to pin down exactly what constitutes the state
(Skocpol, 1985; Nordlinger, 1988), precise definitions have often been contested because social influences tend to pervade even the
most central and powerful institutions of governments (Mitchell, 1991; Jessop, 2001). For this reason Abrams argues that We have
come to take the state for granted ... while remaining spectacularly unclear as to what the state is (Abrams, 1988: 59). Policy-
centric accounts of the state, such as those that revolve around the effectiveness of policy or the
degree to which policy-makers are constrained, threaten to do the same because all those activities that
perform the practice of migrant exclusion have been ascribed to the state ex ante , without thoroughly
interrogating what it is about them that makes them state practices. In the particular context of forced migrants
and refugees, the employment of the separate spheres assumption and the implicitly essentialist
conception of the state that this belies has at least four specific effects, each of which constitute grounds to be cautious when
employing an essential state concept. First, it threatens to obscure the agency of social forces and social actors in the exclusion and
subjugation of refugees and asylum-seeking communities. In debates about the effectiveness of border policy, for
example, social actors are depicted as intrinsically resistant to any involvement in policies that
exclude migrants by implication, needing to be legally obliged, or financially induced, to partake
in them. Yet Lahav and Guiraudon (2000) record the eagerness with which American vigilante border patrols at
the Mexican border have pursued state recognition and ratification of their activities, while Koslowski (2001) recounts
the enthusiasm with which eastern European countries accepted their new responsibilities as European gatekeepers
upon accession. In general, when immigration law allows for discretion at the border, the result
tends to be a greater number of exclusionary practices, not fewer. Weber (2003), for example, contends that
there are several examples of immigration detention practices of dubious legality which have come to be officially sanctioned,
implying that the law is not so much an imposition upon society as a crystallization of pre-existing exclusionary practices (for a
discussion of the law as an effect, not a cause, see MacKinnon, 1989). In the British context, detention to prevent crime, for instance,
subsequently appeared in the Immigration and Asylum Act (1999), detention in order to speed up processing when claims are
perceived to be unfounded was subsequently officially made into law and upheld as best practice, and the targeted detention of
particular nationalities of asylum seekers who are perceived to have poor chances of a successful asylum claim was subsequently
formalized and legalized (Weber, 2003: 253). These observations undermine the notion that states have to calculate and impose
exclusionary practices upon an unwilling or uncompliant social sphere. A second reason why geographers should be
skeptical about the causal power of the state in general is the fact that the legislative and policy-
enshrined objectives of states are reflective of complex processes of political sociology within
and around the state apparatus, involving an array of competing actors with conflicting and
diverse objectives. The state, understood as a relatively coherent actor, masks the competition
for institutional capture that occurs around many policy arenas and debates. The ongoing rescaling of
asylum policy from the national to the European level, for example, has exposed the selective and differentiated response rates of a
number of groups of actors in competition for political influence (Lahav and Guiraudon, 2000).


1NC MEXICAN IMMIGRATION

Making borders a little more porous instantiates disciplinary capture through
mobility regimeswe must refuse the map altogether.
Tsianos et al 8 (Vassilis, teaches sociology at the University of Hamburg, Germany, Dimitris Papadopoulos teaches social
theory at Cardiff University, Niamh Stephenson teaches social science at the University of New South Wales. Escape Routes:
Control and Subversion in the 21st Century Pluto Press)

Liminal porocratic institutions' governance of dynamic migration movements involves steering migrants into
scaled time zones so as to produce governable subjects of mobility from ungovernable streams. Time
is mobility. The humanitarian dilemma of the European border regime lies in the need to institutionalise the difference between
sanctioned, cross-border labour migration on the one hand, and asylum law and juridical protection measures on the other. This in
turn generates camps as heterotopias of sovereignty from which criminalised labour, new migrational
experiences and biographies emerge. Various studies on the US-Mexican border (De Genova, 2005) and on the
south-east European area (Andrijasevic, 2006) illustrate that the productive function of the border regime does not primarily
consist of the capacity to stem or block migration flows. Rather, the effective governing of border porosity operates
through registering movement and disciplining migrants in the camp stations as subjects of flexible,
postliberal social order and labour. This form of governing is what we call porocracy, achieving global inclusion
in the realm of productivity through the deceleration of migration flows.



1NC LAND REFORM

Land reform programs remove the unevenness of Latin Americas geographical
structure and isolate economic variables and political goals to justify the plan
Fraser 08(Fraser, Alistair 3 (Jul 2008 GEOGRAPHY AND LAND REFORM Fraser, AlistairGeographical Review98.): 309-321.
ttp://search.proquest.com.proxy.lib.umich.edu/docview/225330662?accountid=14667 BRW)
For vastly different reasons, numerous states in Asia, Africa, and Latin America pursued land-reform programs
during the "developmentalist moment" between 1950 and 1970 (Bernstein 2002, 434). In Latin America, for example,
the United States-backed Alliance for Progress promoted land reform as a way to stamp out the
threat of communism; in other places, land reform was intended to assist realizing socialist or communist visions. Although
such variation reflected "fundamentally different conceptions" of development, the approaches shared a "conception
of the fundamentally reactionary character of pre-capitalist landed property" (p. 438). They also shared
another characteristic: the state's prominent role as guardian, provider, and manager. For example, the state would expropriate or
purchase land, reallocate resources to and often protect-from imports, for example-landreform beneficiaries, or set in motion
mechanisms to support agriculture on redistributed land. But amid the then-emergent neoliberal projects in the 1970s and 1980s,
the state's central position as the driver of land reform became untenable. Neoliberal orthodoxy demanded "rolling
back" the state-for example, via privatizing state-run enterprises-at the same time as states were
pressured to "roll out" other market friendly adjustments (Peck and Tickell 2007). State-led redistributive land
reform fell from grace in this context. In its place, the World Bank set forth and then helped fund the rolling out of a type of land
reform known as market-led agrarian reform (MLAR) in Mexico, the Philippines, South Africa, and a range of other places
(Deininger and Binswanger 1999; see also Borras 2003). MLAR calls for a shift away from state-led, supply-driven
approaches and toward a market-friendly, negotiated, and demand-driven style of land reform;
negotiated and demand-driven, that is, because landowners must be "willing sellers" and beneficiaries must have
demonstrated their determination to use the land commercially. MLAR entails acquiring land from so-called willing sellers rather
than via expropriation and delivering land for commercial, rather than subsistence, purposes and only if beneficiaries demonstrate
their determination to acquire it. In other words, MLAR is a demand-driven model. The rolling out of this "new wave" of
land reform alters the meaning of land questions in the contemporary period (Bernstein 2002): Land
reform is now supposed to be about economic growth and market efficiencies, rather than land-
rights claims, alleviation of poverty, or banishment of predatory, precapitalist property holders.
The different meaning of land reform under MLAR approaches poses a range of new research questions, particularly about how
likely the approaches are to succeed and how to measure any such success. Various other developments since the mid-to-late 19905
have catapulted land reform back onto the agenda. In some places, such as Zimbabwe, a controversial form of redistributive or "fast-
track" land reform has occurred (Bernstein 2004, esp. 210-220; see also Moyo and Yeros 2005). Indeed, no place better exemplifies
the deserved centrality of land questions and land reforms to debates regarding development than Zimbabwe. Land reform also
occupies a prominent place in debates about South Africa. Postapartheid land-reform policies, partly unfolding in the shadow of
Zimbabwe's efforts, have progressed slowly but-to some extent-surely. The land question in South Africa frequently hits the
headlines, often outside the country, where the governing party's treatment of private-property rights in general and rights of white
farmers in particular generates considerable interest, not least among editorial staffs of Europe-based media organizations. In
addition, stateled reforms have begun and look likely to expand in Venezuela and Bolivia, reforms that have the potential to attract
the ire of conservative, liberal, and, especially, neoliberal critics of Latin America's new left wing but that may also address inequities
and landlessness and thereby validate claims about the resurgence of landless people's movements (Moyo and Yeros 2005). Land
reform has also emerged in places outside the developing-world arenas in which it has had a prominent career and in which most
research has occurred. Scotland, for example, has an ongoing and innovative program (Mackenzie 2006a, 2006b). In some countries
in Eastern Europe and other parts of the former Soviet empire, moreover, land reform has surfaced as an issue of considerable
importance (Dawidson 2005 ). What these cases indicate is the enduring legacy of colonial-or imperial-era land expropriations and
associated maldistributions. GEOGRAPHICAL RESEARCH ON LAND REFORM Geographers have noted the resurgence of land
reform. In what follows in this section I discuss a selection of what I perceive to be some of the most promising recent
geographical studies of land reform; recent, that is, because my interest is in exploring the potential for new connections
among geographers who are currently studying the topic. But first a clarification: Land reform is by no means a-or, indeed, the-
central issue in all of this literature; rather, in many instances it is just part of the backdrop, a contextual and
often highly contingent matter. Consider, for example, Deborah Potts's fascinating research, which examines some
effects of Zimbabwe's economic meltdown on contemporary urbanization (2006). The economic collapse, she argues, has
"so undermined the economic advantages of the city that, in terms of rural versus urban living
standards, most recent migrants judged that it had either not been 'worth' migrating to the city
or felt they had not gained anything"

1NC EMPIRE

Their concept of Empire as a flat space deterritorializes the Global South
subjectivities do not move without resistance
Sparke, no date professor of geography at the Jackson School of International Studies (Matthew Sparke, no date [after
2006], Everywhere But Always Somewhere: Critical Geographies of the Global South,
http://faculty.washington.edu/sparke/Everywhere.pdf)//CC
Another lesson of place specific counter-mappingsisthat they underline the danger of imagining
the Global South in abstract termsthat merely reverse the false universalism of flat world
discourse. Michael Hardt and Antonio Negrissingular concept of a globalMultitudestruggling in and through
the globalspace ofEmpirefallsinto exactly thisidealist trap. It istherefore no accident that despite obviously
holding much more critical worldviews Hardt and Negrireduplicate some ofthe same deterritorialization
in Friedmans account when they imagine the geography of Empire as a kind ofsmooth space
across which subjectivities glide withoutsubstantial resistance (2000, 198). From the perspective of
even the most mobile inhabitants of the Global South Mexican workersin the U.S .,
Indonesian workersin Saudi Arabia, Filipino workersin Singapore, Turkish workersin Germany, for example the concept of
gliding across national boundaries withoutsubstantial resistance surely soundslike a bad joke. The
only class of globalsubject for whom such gliding is a reality isthe classthat Friedman writesfor: the
transnational capitalist classfor whom flat world discourse is a daily reality or at least an ever-
present advertising idea through which to try to live (Sklair, 2001;Roberts, 2004).

COUNTRY SPECIFIC


1NC CUBA
Their definitions of Cuba are imbued with colonialist and instrumentalizing
metaphors
Prez 8 (Louis A., Ph.D. University of New Mexico, Professor of
History at University of North Carolina, "Cuba in the American
Imagination: Metaphor and the Imperial Ethos," slim_)
Cuba came to the attention of the world at large principally by way of figurative depiction, more
precisely, in the form of metaphors imbued with colonial meanings: in the sixteenth century as "the Key of
the New World" ("la Llave del Nuevo Mundo"), "the Key to the Gulf" ("la Llave del Golfo"), and "the Bulwark of
the West Indies" ("el Antemural de las Indias Occidentales"); in the nineteenth century as "the Queen of the Antilles,"
"the Pearl of the Antilles," "the Gem of the Antilles," and "the richest jewel in the royal crown," by which
time, too, it had earned the designation of "the Ever Faithful Isle" ("la Siempre Fidelisima Isla"). Metaphorical
representation also developed into the principal mode by which the Americans propounded the
possession of Cuba as a matter indispensable to the future well-being of the United States. To
advance a plausible claim to a territory governed by Spain, and to which its inhabitants presumed rightful succession to rule,
required the Americans to create a parallel reality by which they persuaded themselves-and sought
to persuade others-that Cuba rightfully belonged to them, not only, however, and indeed not even principally,
as a matter of self-interest but as a function of providential purpose and moral propriety. Metaphorical
constructs were central to the process by which national interest was enacted as idealized
purpose: at once a combination of denial and dissimulation, a source of entitlement, and a means of
empowerment. To understand the North American use of metaphor is to gain insight into the use of
cultural models and social relationships in which the U.S. imperial project was conditioned.
Metaphors of Cuba served to advance U.S. interests and were, in turn, mediated by racial
attitudes and gender hierarchies, on one hand, and prescience of destiny, on the other. They worked best within those
belief systems from which Americans obtained their cues concerning matters of civic duty and moral conduct and, indeed, were the
principal means by which intent of purpose and reception of meaning were transacted. Figurative depiction drew into
complicity all who shared a common cultural system from which collectively to receive the
meaning desired of metaphor, what Herbert Clark and Catherine Marshall described as "mutual knowledge based on
com- munity membership?"


2NC CUBA

They depict Cuba as unruly and lawless, to be subjugated and controlled by
American imperialism
Slater 97 (David, Ph.D from London School of Economics and
Professor Emeritus of Geography at Loughborough University,
Geopolitical imaginations across the North-South divide: issues
of difference, development and power, Political Geography Vol.
16 Issue 8, November 1997, pp. 631-653, Muse, slim_)
By the beginning of the twentieth century, notions of manifest destiny and civilizing missions, taken together with
the Monroe Doctrine of 1823, had become a prominent feature in the formulation of foreign policy. Motivated by a sense of
mission, which was linked to territorial and economic gain (LaFeber, 1963), the United States assumed its share of the
White Mans Burden,l and in the years from 1898 to 1915 it acquired an empire that embraced territories in the Pacific Ocean and
the nearer Caribbean sea. The latter region, taken together with Central America, came to be regarded
as Americas backyard.22 The use of such a term is in itself revealing since the backyard is a vital
part of the American familys geography. It is a place that evinces deep feelings about control
and ownership-assumed and yet vital to the familys security. The backyard is a space that is walled
off against intruders; it is a zone for play, experimentation and control, a place that acts as a laboratory for
ideas that can be tried out beyond its walls.23 Perhaps no place in this imaginary space assumed as much significance as the island
of Cuba. Towards the end of the last century, as Cuba was brought closer into the orbit of United States
power, it was asserted that it is manifest destiny that the commerce and the progress of the
island shall follow American channels and adopt American forms (Benjamin, 1990: 54). A guiding
motivation of political and economic tutelage was clearly reflected in the nature of US
intervention and occupation of Cuba at the turn of the century. As a result of US armed intervention and subsequent
military occupation Cuba ceded territory for the establishment of a foreign naval base (Guantanamo),24 agreed
to a significant curtailment of its national sovereignty and authorized future US interventions The
Permanent Treaty of 1903, known as the Platt Amendment, included an article which stipulated the right of the United States to
intervene for the preservation of Cuban independence.l Power over Cuba was also expressed through the dissolution of the
institutions of the Cuban independence movement-the Liberation Army, the Provisional Government and the Cuban Revolutionary
Party, originally founded by Jose Marti. United States tutelage over Cuba also included the transfer of over 1000 Cuban teachers to
Harvard for training in US teaching methods, and Protestant evangelists established almost 90 schools in Catholic Cuba between
1898 and 1901. In addition, teams of US experts placed the mineral, agricultural and human resources of
the island under their scientific gaze. The significance of tutelage was reflected through the
visual representation of Cuba inside the United States . At the end of the war with Spain, the
independence struggle of the Cubans was characterized by a portrayal of Cuba as a Latin maiden, vulnerable and in need of
protection (see Fig. I). However, after the war of Independence was won, there was a re- direction of Cuban
nationalist aspirations against US influence, and as a consequence the portrayal of Cuba inside the United
States tended to shift from that of a vulnerable Latin female to a thankless and unruly Black
brat, in need of discipline and guidance (see Fig. L?). The Father-child distancing was intensified
by the racial distinction of the Father being white and the child black.27

Metaphors of U.S. benevolence monopolize imperialist representations of Cuba
Prez 8 (Louis A., Ph.D. University of New Mexico, Professor of
History at University of North Carolina, "Cuba in the American
Imagination: Metaphor and the Imperial Ethos," slim_)
Certainly this had to do with Americans thinking highly of themselves, but it also had to do with an abiding concern to
be thought well of by others. In- deed, both were vital to the ways that the Americans came to
define and defend their claim to Cuba. The American way to imperialism was inscribed within
cultural forms as sources of usable modes of knowledge and deployed by way of metaphorical constructs as
usable models of conduct. Cuba entered the American imagination early in the nineteenth century
principally by way of metaphor: depictions fashioned as a function of self-interest, almost always in the form of
moral imperative in which the exercise of power was represented as the performance of
beneficence. It is not that the metaphorical motifs the Americans used to represent Cuba were necessarily original or unique to
the United States. On the contrary, the vernacular of empire reaches deeply into the history of colonial narratives. What was
different about Cuba, and what will be argued in the pages that follow, was the prominence of metaphor as a mode
of North American discourse, which is to say, the prominence of metaphor in the production of knowledge.
What was different about Cuba was the degree to which metaphor so utterly displaced alternative
cognitive possibilities. Virtually all the metaphors in the stock of imperial tropes were fully aggregated into a single narrative
of remarkable endurance-itself evidence of the power of the pathology that was Cuba.

Their portrayal of Cuba as key to U.S. well-being perpetuates a self-fulfilling
prophecy of manifest destiny
Prez 8 (Louis A., Ph.D. University of New Mexico, Professor of
History at University of North Carolina, "Cuba in the American
Imagination: Metaphor and the Imperial Ethos," slim_)
But it is also true that, for all the ways that Cuba stands as an embodiment of American imperial practice, it is
at the same time different- so different, in fact, that it must be considered as a case apart. Cuba seized hold of the North American
imagination early in the nineteenth century. What made awareness of Cuba particularly significant were the ways that it acted
on the formation of the American consciousness of nationhood. The destiny of the nation
seemed inextricably bound to the fate of the island. It was impossible to imagine the former
without attention to the latter. All through the nineteenth century, the Americans brooded over the anomaly that was
Cuba: imagined as within sight, but seen as beyond reach; vital to the national interest of the
United States, but in the possession of Spain. To imagine Cuba as indispensable to the national
well-being was to make possession of the island a necessity. The proposition of necessity itself
assumed something of a self-fulfilling prophesy, akin to a prophetic logic that could not be
explained in any way other than a matter of destiny. The security and perhaps - many insisted - even the very
survival of the North American Union seemed to depend on the acquisition of Cuba. The men and women who gave thought to
affairs of state, as elected leaders and appointed oflicials; as news- paper editors and magazine publishers; as entrepreneurs,
industrialists, and investors; as poets and playwrights; as lyricists, journalists, and novelists; and an ever-expanding electorate-
almost all who contemplated the future well- being of the nation were persuaded that possession of Cuba was a matter of national
necessity. Not everyone agreed, of course. It was with a sense of exasperation that Vermont senator ]acob Collamer protested in 1859
that "the idea that the pos- session of Cuba is necessary to the actual existence of this country, is a mere figment of the imagination."
But that was exactly the point: the convention- ally wise were indeed persuaded that possession of Cuba
was indispensable to the "actual existence" of the United States. And, as will be argued in the pages that
follow, precisely because Cuba revealed itself as a "figment of the imagi- nation," the island inscribed itself deeply into
the very certainties by which Americans arrived at a sense of themselves as a nationality and as a
nation.


1NC MEXICO

Portrayal of Mexico is rooted in Manifest Destiny and the divide between civilized
and the barbarian
Slater 97 (David, Ph.D from London School of Economics and
Professor Emeritus of Geography at Loughborough University,
Geopolitical imaginations across the North-South divide: issues
of difference, development and power, Political Geography Vol.
16 Issue 8, November 1997, pp. 631-653, Muse, slim_)
The US-Mexican War of 1846-1848 provides a particularly pertinent example of a geopolitical
expansion that was informed by an underlying belief in Anglo-Saxon superiority. An armed clash between
Mexican and US troops near the Rio Grande, with the loss of American lives, led the then President of the United States to declare
War- President Polk declared that Mexico had passed the boundary of the United States, invaded our
territory and shed American blood upon American soil. At this time, the Rio Grande had never been
recognized as United States territory, but the 1846-1848 War enabled the US to expand into
Mexican territory, and through the Treaty of Guadalupe Hidalgo the United States acquired the
present-day states of California, New Mexico, Nevada and parts of Colorado, Arizona and Utah,
a total of over 850 000 square kilometres of Mexican land.15 In 1847, the New York! Herald suggested that
the universal Yankee nation can regenerate the people of Mexico in a few years, but more common were views which posited
an unbridgeable gulf between the civilized and the barbarian. The Cincinnati Casket, for instance, urged that
a war between enlightened nations would shock humanity, but an occasional conflict with
barbarians must be expected the Mexicans will be led by this war to think of their weakness and inferiority. The
dominant mood of the time was aptly captured in the American Whig Review, which noted that Mexico was poor, distracted,
in anarchy and almost in ruins-what could she do to stay the hand of our power, to impede the march of our
greatness? We are Anglo- Saxon Americans; it was our destiny to possess and to rule this
continent . we were a chosen people, and this was our allotted inheritance, and we must drive out all other nations before us.i
Territorial expansionism into Mexico was characterized by a contentious debate over the perceived advantages and disadvantages of
incorporating a people deemed to be so palpably inferior. In the cabinet of the time, Secretary of State James Buchanan often
expressed his fear of the admission of any large number of Mexicans to the Union, and in the wake of the 1848 Treaty, one
newspaper, the Louisville Democrat, expressed the opinion that the United States had obtained not
the best boundary, but all the territory of value that we can get without taking the people (quoted
in Horsman, 1981: 245-246). Efforts to acquire all of Mexico in the 1846-48 period raised overwhelming racialist objections to
having the Mexicans brought into the Union. Although the Mexicans were a neighbouring people known to the people of the United
States for generations, they were considered generally unacceptable to Americans as a part of the body politic.s It was a widely-held
belief throughout the nineteenth century, and particularly before the Civil War, that people who were not capable of self-government
should not participate in the governing of Anglo-Saxons. In this context, the later expansion which took place between 1898
and 1916, and which concerned the acquisition of non-contiguous territory inhabited by races considered by the
dominant sentiment within the United States to be inferior,i9 was accompanied by an evasion of the American
constitutional principle that all citizens of a republic ought to enjoy an equality of rights. In other
words, as Weston (1972) has shown in his analysis of the influence of racial assumptions on American foreign policy, the ideology of
racial superiority tended to compromise the principles of equality of rights and opportunity, a factor, of course, that also impinged
on domestic politics. An international relations theorist recently observed that the alternative worlds destroyed and
suppressed within modern cartography become available only when the global map is given
historical depth (Shapiro, 1994: 483). The US-Mexican War can provide an example of the need to
recover such an historical and, one can add, geopolitical depth. Specifically, in the time of NAFTA, and in some dimensions
the Miamization of Mexico, it is clear that todays US-Mexico relations are frequently marked by a geopolitics
of amnesia. A Chicano film producer, Paul Espinoza, is currently preparing a documentary on the US-Mexican War as
part of a multi-cultural project to improve the level of understanding in both societies of their interactive geopolitical histories.20
The project has already come under fire from conservative forces in the US Congress, but has received sufficient independent
financial support to guarantee broadcasting in the near future, around 150 years since the outbreak of hostilities. It is highly likely
that the documentary, supported by a range of Mexican and United States academics and journalists, will resuscitate interest and
provoke controversy over the ways in which the historical and geopolitical significance of mid-nineteenth
century events can be represented today; it provides a cogent example of the continuing significance of the geopolitics of
memory, and can be used as part of a wider project to de-naturalize and desediment given assumptions about
border zones and the meanings of territorial sovereignty. Questions of territorial loss and territorial
acquisition, and of the history of geopolitical interventions are intrinsic to the US-Mexican encounter, and
their recovery as thematic markers can be seen as an expression of a will to understand and rethink the patterns of interactive
representation.


2NC MEXICO

Especially true in Mexicoeconomic engagement frames Mexico as a laboratory
for U.S. foreign relations abroad and solidifies its geopolitical interests
Gilderhus 5 (Mark T. Gilderhus is a professor of history at Texas Christian University in Ft. Worth and the Lyndon B.
Johnson Endowed Chair Holder in U.S. history. As a diplomatic historian, his specialty has focused on U.S.-Latin American
relations. His recent publications include The Second Century: U.S.Latin American Relations Since 1889 (Scholarly Resources,
2000). Forming an Informal Empire without Colonies: U.S.-Latin American Relations. Latin American Research Review 40:3,
2005, 312-325//[SG])

Last but assuredly not least, John Mason Hart's Empire and Revolution exemplifies the best tendencies in recent scholarship.2 This
magisterial volume represents a lifetime of labor in academe and contributes significantly to the literature. Hart's main theme
concerns the impact of economic influences shaping modern Mexico, especially the activities of enterprising
American investors and entrepreneurs. For Americans, the quest for informal empire produced diverse reactions among
Mexicans while forming in the larger context an ongoing process of give and take. For Hart, the unfolding of American purposes
south of the border has displayed a remarkable consistency, emphasizing always the acquisition of wealth and power. For Mexicans,
in contrast, adaptations have featured various responses, sometimes accommodating and sometimes not. [End Page 323] In this
sweeping, sprawling, insightful book, Hart shows with an abundance of specificity, depth, and detail how U.S. corporate interests
acquired so much influence over their southern neighbor. He begins by recalling William Appleman Williams's description of
Latin America as a laboratory for U.S. foreign relations, that is, a place where U.S. elites could
experiment with various techniques of informal empire for expanding their power and influence. In
Hart's view, Mexico became a prototype where Americans tried out such devices as "partnerships with local
elites, cooperative arrangements among multinationals . . . interventions . . . [and] outright invasions" (5). As he
shows, subsequent applications elsewhere form important parts in the early history of globalization.

The United States integration of Mexico is not benign instead US policy attempts
to instrumentalize Mexico as a sweat shop to produce profits
Petras, 02 Analyst at the Centre for Research on Globalisation (James Petras, 3 April 2002, U.S. Offensive in Latin America:
Golpes, Retreat and Radicalization, http://www.globalresearch.ca/articles/PET204Ap.html)//ah
Washington's militarization strategy is also evident in Ecuador, Bolivia, and Paraguay, where client regimes,
stripped of any popular legitimacy, impose Washington's formula of free markets in Latin America and
protectionism and subsidies in the United States. In Brazil and Mexico, Washington relies heavily on political and diplomatic
instruments. In the case of Mexico, Washington has direct entre to the administration of Vicente Fox in economic
policy, and a virtual agent in the Foreign Minister, Jorge Castaneda. The goal of Mexican subordination to U.S.
policy is not in question, as Fox and Castaneda are in total agreement. What is in question is the effectiveness of the regime in
implementing that policy. Fox's effort to convert southern Mexico and Central America into one big U.S. assembly
plant, tourist, and petroleum center (the Puebla-Panama Plan) has run into substantial opposition. The massive shift of
U.S. capital to cheaper labor in China has provoked large-scale unemployment in Mexican border towns. The so-called reciprocal
benefits of integration are glaringly absent. U.S. dumping of corn and other agricultural commodities has devastated
Mexican farmers and peasants. The U.S. takeover of all sectors of the Mexican economy (finance,
telecommunications, services, etc.) has led to massive outflows of profits and royalty payments. In foreign
affairs, Washington's influence has never been greater, as Castaneda crudely mouths the policies of the U.S. Defense Department
and CIA-declaring unconditional support for the U.S. policy in Afghanistan and any future military interventions, and grossly
intervening in Cuban internal politics, provoking the worst incident in Cuban-Mexican diplomatic relations in recent history.
Castaneda's anti-Cuban interventions on behalf of Washington backfired, with the great majority of the Mexican political class
calling for his censure or resignation. Yet, it is clear that the mere presence of such an unabashed promoter of U.S. policy in the Fox
Administration is indicative of Washington's aggressive conquest of space in the Mexican political system. The powerful
presence of U.S. corporations, banks, and numerous regional and local client politicians
facilitates the recolonization of Mexico-against an increasingly restive and impoverished labor
force.

The economic integration of Mexico is the Norths attempt to tame the wild zones
spawned in the global south
Tuathail, 96 Ph.D. in Political Geography from Syracuse UniversityProfessor of Government and International Affairs and
Director of the Government and International Affairs program, Virginia Polytechnic Institute, (Gearid Tuathail, 1996, Critical
Geopolitics The Politics of Writing Global Space
http://www.nuevageopolitica.com/resources/Textos_Geopolitica/Tuathail,%20Critical%20Geopolitics.pdf)//ah
Third, overdetermining the problematic of the writing of global space at the end of the twentieth century is the receding power of the
state relative to the global economy in mastering space. Indeed, in many parts of the world, space is no longer mastered
by the state at all but by regional warlords and criminal gangs with connections to the global economy. On the
African continent, as Robert Kaplan has argued, places like the Sudan, Nigeria, Sierra Leone, Liberia, Zaire, Rwanda, Burundi, and
Somalia are nominally states but, in practice, are something else.49 Most of Angolas diamonds, Cambo dias timber, Perus cocaine,
and Afghanistans poppy plants are controlled, exploited, and sold to the international market by bandit groups operating beyond
the state. Even in industrial and industri alizing regions, governmental authority is in retreat in the
VISIONS AND VERTIGO 197face of successful contraband economies mediating the global and the
local outside the national. In all of the worlds states, significant portions of the economy operate outside the law and
official statistics. In Russia, for example, organized crime, in partnership with former Communist Party capitalists, controls as much
as 40 percent of the turnover in goods and services in the economy.50 In more affluent regions, global space is also being
remastered, as transnational enterprises try to instrumentalize relatively strong states to serve their
postnational interests while the wealthy in general try to downsize the state to a self-serving
minimalist functionalism. The European Union envisions Europe largely within the terms of a corporatist vision of the
Continent as an integrated environment for capital accumulation. In the United States, transnational corporations and their
ideologues have successfully rezoned the territory of the United States into a North American
free trade region that presently includes Canada and Mexico and will soon incorporate Chile. New
transnational corporatist spaces are envisioned for the future. A free trade zone from Alaska to Argentina is promised by the year
2005, while an enormous Pacific Basin free trade zone is projected for the year 2020. Whatever the nature of new writings of
global space by intellectuals of statecraft, all must now take account of the dynamics of informationalization and
globalization, of integration and disintegration, in a world where the modern sociospatial triad of the interstate system is in
crisis. As already noted, the meaning of state sovereignty is questionable when even the most powerful of states depend upon the
goodwill of private financial markets for their economic health and security. Similarly, transcontinental missiles, global satellite
television, the Internet, and global warming have rendered territorial integrity a problematic notion. Statist notions of community
are also straining, as nominally national but functionally transnational classes use the state to serve their global
interests and attempt to secede from common national space by barricading themselves off from
the rest in gated enclaves of privilege and affluence. Although somewhat simplistic, the notion of the world
being increasingly marked by wild zones of poverty and violence, on the one hand, and tame
zones of wealth and privatized security, on the other hand, is not that farfetched when one considers
Washington, D.C., the former capital of the free world and current exemplary of a starkly bifur cated
world of haves and have nots.51 The distance between the crack houses and ghettos of the central
city, its wild zones of urban poverty and violence, and the master-planned elegance and simulated
historic charm of edge cities like Reston, Virginia, the districts outlying postsuburban tame zone where traditional
governmental functions like security and zoning are privatized, is only a matter of a few miles, but it is a distance that
perhaps foreshadows the spatial structure of twenty-first century transnational corporatist
capitalism.52 Managing the wild zones of the globe and protecting the security of its tame zones will
certainly exercise the minds of the geopoliticians of the future, geopoliticians who will invariably construct their
mappings of global space from 198 VISIONS AND VERTIGOthe standpoint of tame regions and with the
agenda of protecting the privileges of the affluent and tame against those who appear to threaten their
spatial security and quality of life. Indeed, the recent greening of governmentality marked by the discourse of politicians like Al
Gore, the rising influence of think tanks like the aptly named World Watch Institutea technocratic institution devoted to
monitoring the state of the worlds environmentand the emergence of a new congealment of geo-power called environmental
security can be interpreted as a response to the problems that decades of environmental degradation are posing for the rich and
powerful, planetary-wide dilemmas involving questions of production, technology, sustainable development, and consumerism that
the rich can no longer afford to ignore.53 Even in their relatively immunized tame zones, the worlds richest peoples and ruling
classes will be affected. Thus questions of ozone depletion, rainforest cover, biodiversity, global warming, and production using
environmentally hazardous materials are the subject of new environmentalist mappings of the global, contemporary acts of geo-
power that triangulate global space around the fears and fantasies of the already affluent.





1NC VENEZUELA

The 1acs obsession with capitalist efficiency conceives of our geographic
relationship with venezuela in terms of efficiency while ignoring true barriers to
relations this ensures that the plan further damages Venezuelan relations while
creating existential scenarios to justify it
Bonfili, 10 (Christian Bonfili December 21, 2010 41: 669 Security
Dialogue The United States and Venezuela: The Social
Construction of Interdependent Rivalry BRW)
The present article is framed by a constructivist perspective, as it combines Alexander Wendts social constructivist theory with the
Copenhagen School of security studies. Social constructivism offers a rather sophisticated theoretical apparatus that illuminates the
foundations of state agency in world politics and, most notably, the mutual constitutiveness of social structures and agents. The
work of the Copenhagen School enhances social constructivisms explanatory power, as it provides important insights into the
fundamentally relational character of security, as well as how actors may form a distinctive scheme of interaction as a result of a
particular distribution of power and historical relations of amity or enmity. Insofar as USVenezuelan relations
presuppose the collective construction of threat perceptions that are translated into
extraordinary measures, bilateral interaction reveals the weight of intersubjective understandings. This assumption
allows the chosen theoretical perspective to offer a comparatively better account of the
emergence of conflict in bilateral relations vis--vis liberal and neorealist approaches. The latter are
unable to explain why conflict (namely, security dynamics based on rivalry) has evolved in the first place. Liberal-based
explanations are unable to assess this phenomenon against the background of a longstanding
and institutionalized cooperation in the economic/energy sector. For instance, interdependence
literature assumes that economic cooperation leads to peace, because this is the functionally
efficient policy for states that share a common goal in the maximization of capitalist profits
(Sterling-Folker, 2009: 104109). Yet, such an assumption fails to explain why in the case of the US
Venezuela relationship concerns over less rational aspects such as the autonomy, ideology, and
identity of the state have come to be sufficiently powerful as to co-exist with rational selfinterest
in capitalist profit. For its part, neorealism is unable to explain the extent to which such concerns have framed the appraisal
of external threats and informed foreign policy choices in recent times. The fact that neorealism is a systemic theory not only means
that foreign policy is not its focus of analysis but also, more importantly, that the ultimate cause of state behavior
within its framework is not to be found at the level of the unit but, rather, at the level of the
international structure. Accordingly, the unaffected character of structural variables before and after the Chvez and Bush
administrations emphasizes the need for alternative explanations. These are to be found, for instance, in looking at how certain
understandings about existential issues, and the resulting process of constructing threat perceptions, played a
decisive role in prompting the reassessment of Venezuelas foreign strategy and goals following
the attempted coup of 2002. Sectors The study of bilateral relations is based on a sectoral approach in an attempt to
disaggregate the whole structure of interaction. Though they are inseparable parts of a single social reality, sectors reflect distinct
patterns of behavior and ideational constellations that sustain co-existence between rivalry and interdependence. Geographic
Proximity and Interdependence These concepts refer to the incidence of territoriality in security dynamics,
as well as understandings about costs and vulnerabilities derived from interdependence. The logic of territoriality is strong in the
realm of security, because physical adjacency tends to increase security interaction among states, which means that threats travel
more easily over short distances rather than over long ones (Buzan & Wver, 2003: 12). In the context of the US
Venezuelan energy relationship, proximity is reflected mostly in terms of the density of the two
countries transactions. The question of geographic adjacency draws attention, in turn, to the
interrelated aspect of interdependence. This notion not only refers to the density of interactions
(mutual dependence on a m a terial basis), but also to the particular way actors major perceptions and
concerns are linked (mutual dependence on an intersubjective basis). Politicization and Securitization Politicization is
here understood as the process whereby certain issues become part of the political agenda
because they are now taken into consideration, debated, or assessed, yet without attempts being
made to justify their urgency or existential nature. By contrast, securitization refers to the praxis by which agents
construct an existential threat to an entity or shared value (referent object) conceived of as being worthy of defense by
means of an extraordinary response that, in turn, is legitimized by a significant audience. In this way, securitization entails a
specific discursive structure the security speech act in virtue of which an issue is dramatized and
designated as a matter of supreme priority (Buzan, Wver & de Wilde, 1998: 26). Securitization relies heavily on the
social milieu within which it operates, as the latter provides the content for, and contributes to, the required acceptance by the
audience. For the securitizing actors claim to be accepted, securitization has to be part of a discursive, socially
constituted, intersubjective realm (Buzan, Wver & de Wilde, 1998: 31). Role Identity The concept of role identity
refers to the position an actor fills within a social structure and the actors adherence to social norms when interacting with others
who display counter-identities (Wendt, 1999: 226). The social character of role identities stems from the fact that the agents
position presupposes certain expectations, on the basis of values and norms that are shared by members of a social structure. The
concept of role identity provides a more differentiated classification of patterns of behavior vis--vis the simple dyad of enemy or
friend offered by the Copenhagen School. For instance, it allows for a specific kind of identity in the economic/energy sector,
namely, that of a partner in a partnership internalized by interest, which can be regarded as a middle position between those of
friend and enemy. The Economic/Energy Sector: Partnership Based on Interest USVenezuelan energy
interdependence is at base reflected in the rate of transactions, degree of interconnection, and
investment ties between the two countries oil sectors. Geographic proximity is largely
responsible for this, as Venezuelan oil is only about 5 days away from the US Gulf Coast by tanker, compared to about 30 or
40 days for oil from the Middle East. 2 Conventional and heavy crude oil from state-owned oil and gas company Petrleos de
Venezuela Sociedad Annima (PDVSA) reaches US consumers via the interconnection between PDVSAs subsidiary CITGO
Petroleum Corporation and the downstream portion of the US system. Indeed, a significant level of energy integration is illustrated
by the fact that Venezuela was the fourth-largest exporter of crude oil and the third-largest exporter of petroleum products to the
United States from 2002 through 2009. 3 In addition, most Venezuelan oil is a heavy, sour type of crude oil, which means that only a
small number of US refineries can process it. In the event of potential disruptions to this supply, these same refineries would face
significant difficulty in finding replacements, as they are specifically configured to run most economically with Venezuelan oil (Shore
& Hackworth, 2003).

2NC VENEZUELA

Especially true in Venezuelaeconomic engagement is a ploy to secure the
geopolitical interests of American Empire
Liyanage 13 (Sumanasiri, I teach Political Economy at the University of Peradeniya, Sri Lanka. My research interests
include Marxism, Nationalism, globalization and its impact, Indian Ocean Region. Hugo Chvez: A Leader Who Challenged Neo-
Liberalism And Washington, Colombo Telegraph, March 7
th
, http://www.colombotelegraph.com/index.php/hugo-chavez-a-leader-
who-challenged-neo-liberalism-and-washington/ //[SG])
It is interesting to note that Chavezs foreign policy was consistent with his domestic policies and was based on the same principles.
He knew very well that the local big business, media giants and his political opponents were not working in isolation. He
observed that they were backed and discreetly supported by the US imperialism. In my view, there were two principal pillars in his
foreign policy, namely (1) unconditional opposition to US imperialism and aggression; (2) the formation and strengthening of the
united front of Latin American countries. He characterized the US as No 1 enemy of the poor and marginalized people in the world.
It was no secret for him that the so-called campaigns for democracy and human rights had been now reduced
into techniques of governmentality deployed by the US imperialists and their lackeys. This perspective led to
understand the complexity of the new situation in the Arab World. Hence Chavez opposed the US intervention in toto
wherever and whenever it was in action. When Obama responding to Chavezs death informed that the US can now
have new kind of engagement with Venezuela, he tried to single out Chavez from the movement he built since 1994.
The US imperialists might have thought that the main obstacle for their aggressive policy in Latin and
Central America was now removed.







INTERNALS

BIOPOLITICS

Territory is dynamic and intertwined with biopolitics questions of borders,
defense, and resource extraction highlight the convergence of governmentalities
Elden 13 (Stuart, Ph.D. in Political Theory from Brunel University,
Professor of Geography at Durham University, "How Should We
Do the History of Territory?", Territory, Politics, Governance,
1:1, 5-20, 24 January 2013, slim_)
Territory then should not be understood as the static backdrop or container of political actions.
Nor is it the passive object of political struggle. It is something shaped by, and a shaper of, continual
processes of transformation, regulation and governance. Questions of division, bordering,
contestation and conquest, ownership and extraction of resources, colonisation, measurement
and quantication, threat and defense all have territorial elements; all impact on the
understanding and practice of territory. The relation between territory and population is complicated and inherently
intertwined. Populations are dened, in part, by their location, and territories, in part, by their
inhabitants. Territory and population emerge at a similar historical moment as new ways of rendering,
understanding and governing the people and land. Both are crucial political questionsbiopolitics and
geopolitics exist, not in tension or as alternatives, but as entirely implicated in each other,
intertwined in complicated and multiple ways. To control territory requires the subjugation of
the people; to govern the population requires command of the land. Geographers who have discussed
the question of the population have long understood the spatial aspects of this question (for example, HANNAH,
2000, 2010; LEGG, 2007). They have, if you will, provided a geopolitical emphasis to questions of biopolitics. Drawing on Foucault,
we can think the question of territory with due attention to the populations within and across its borders; to provide a biopolitical
emphasis to questions of geopolitics. InThe Birth of Biopolitics, Foucault claries his relation to that of state theorists. He criticises
those who thought his lack of a theory of the state meant he cancelledthe presence and the effect of state mechanisms (2004b, pp.
7879, 2008, p. 77). Indeed, he claims that the problem of bringing under state control, of statication
[tatisation] is at the heart of the questions I have tried to address (2004b, p. 79, 2008, p. 77). But this does not mean that he starts
from the state in and for itself, as a political universal, or speaks of the essence of the state (2004b, p. 79, 2008, p. 77). He declared
that he had avoided a theory of the state as one must forgo an indigestible meal (2004b, p. 78, 2008, p. 77; also see LEMKE, 2007;
more generally GORDON, 1991; LEMKE, 1997; JESSOP, 2007). In the previous years course, he had wondered what if the state
were nothing more than a way of governing? What if the state were nothing more than a type of governmentality? (2004a, p. 253,
2007a, p. 248). Now he eshes this out in detail, suggesting that: The State is not a universal; the State is not in itself an autonomous
source of power. The State is nothing else but the effect, the prole, the mobile shape of a perpetual statication [tatisation] or
statications, in the sense of incessant transactions which modify, or move, or drastically change, or insidiously shift sources of
nance, modes of investment, decision-making centres, forms and types of control, relationships between local powers, the central
authority, and so onThe state is nothing else but the mobile effect of a regime of multiple govermentalities. (2004b, p. 79, 2008, p.
77) If you replace the state with territory, and statication with territorialisation,
understanding territory as a process rather than a product, then you have something quite close
to what is being claimed here. Territory is not a universal; Territory is not in itself an autonomous
source of power. Territory is nothing else but the effect, the prole, the mobile shape of a perpetual
territorialisation or territorialisations Territory is nothing else but the mobile effect of
a regime of multiple govermentalities . (after 2004b, p. 79, 2008, p. 77)

Geographical divisions are a violent form of biopolitical discipline used to violently
exclude difference
Sletto, 09 (Bjorn Sletto - , March 17, 2009, `Indigenous people don't have boundaries': reborderings, fire management, and
productions of authenticities in indigenous landscapes, http://cgj.sagepub.com.proxy.lib.umich.edu/content/16/2/253.full.pdf)
MD
State projects of boundary-making are thus key to the cultural production of difference.
In post-modern geopolitics, boundaries are assumed to be constructed through social
processes that are contingent on narratives of nation, region, and identity.13 Thus from a
Foucaultian perspective, social boundary-making is intimately implicated in
power/knowledge and constitutes a form of disciplining, since it effects the removal
of people into a space deemed proper for them: In the first instance, discipline proceeds from the distribution of
individuals in space each individual has his own space, and place its own individual.14 That is, powerful institutions
arrange people in space in order to ensure the efficient surveillance of individual conduct.15

DICHOTOMIES
Spatial delineations differentiate populations and republics
Elden 13 (Stuart, Ph.D. in Political Theory from Brunel University,
Professor of Geography at Durham University, "How Should We
Do the History of Territory?", Territory, Politics, Governance,
1:1, 5-20, 24 January 2013, slim_)
Foucault draws upon Giovanni Antonio Palazzos denition of the state, where four determinations
are given. The rst, Palazzo suggests, is that a state is a domain [domaine], dominium, where the Italian is dominio. The
second is thatit is a jurisdiction, a set [ensemble] of laws, rules, and customs (1604, pp. 1011; quoted and
translated in FOUCAULT, 2004a, p. 262, 2007a, p. 256). But when Foucault notes that a republic is a state in the same four senses,
and restates them, the denitions become a republic is rst of all a domain, a territory. It is then a milieu
[milieu] of jurisdiction, a set of laws, rules, and customs (2004a, p. 262, 2007a, p. 256). This is revealing
because of Foucaults slippage between domain and territory, which was a relation he had claimed was not there
in Botero. It is also interesting because Foucaults way of understanding the relation of jurisdiction to the state is to
invoke another spatial term: it is a milieu of jurisdiction. While he then repeats the denition of it as a set of
laws, etc., the question of where the laws apply is obviously crucial. This question of the spatial
determination of another category is important in a later lecture when he raises a series of questions
about politics, including what is a territory? What are the inhabitants of this territory? (2004a, p.
294, 2007a, p. 286). That is, even if we accept the claim that the inhabitants, i.e. a population, become
the object of government, what sets them apart from other people, other populations, i s
a spatial determination or limit.

Geographic definitions are subjective population and governance characterize
the role of the state
Elden 13 (Stuart, Ph.D. in Political Theory from Brunel University,
Professor of Geography at Durham University, "How Should We
Do the History of Territory?", Territory, Politics, Governance,
1:1, 5-20, 24 January 2013, slim_)
It is well known that Foucault considers this relation between sovereignty and what he labels territory
to be distinctively changed by developments in government, and in particular by the emergence of the
category of population. As he says in a 1977 interview, the role of the state in relation to the people has moved
from a territorial pact where it is the provider of territory or the guarantor of peace within
borders to a pact of population, where people will be protected from uncertainty, accident,
damage, risk, illness, lack of work, tidal wave and delinquency (1994, III, p. 385). Foucault suggests that after Machiavelli the key
problem is no longer that of xing and demarcating [xer et marquer] the territory, but a range of other questions. No longer the
safety [sret] of the Prince and his territory, but the security of the population and, consequently, of those who govern it (2004a, p.
67, 2007a, p. 65). His notion of population is, in one sense, a broadening of the analysis he made in The Order of Things concerning
the category of man. Here the three domains of knowledge in The Order of Thingsare explicitly politicised (2004a, pp. 7881,
2007a, pp. 7679). In the earlier work, Foucault had traced the shifts from natural history to biology; from analysis of wealth to
economics; and from general grammar to linguistics (1966, 1970). He now situates these in a broader, and more political, setting.
For these transitions, if we look for the operator that upset all these systems of knowledge, and
directed knowledge to the sciences of life, of labour and production, and of language, then we
should look to population (2004a, p. 80, 2007a, p. 78). He, therefore, suggests that the earlier theme of man
and the human sciences should be understood in terms of how on the basis of the constitution
of the population as the correlate of techniques of power a whole series of objects for possible
forms of knowledge were made visible, and reiterates that this was on the basis of the emergence of population as the
correlate of power and the object of knowledge(2004a, pp. 8081, 2008, p. 79). He therefore concludes that man is to
population what the subject of law [droit] was to the sovereign (2004a, p. 81, 2007a, p. 79). As a result of
this development, Foucault claims that the object of government is transformed, as well as the technique of rule. One never
governs a state, a territory, or a political structure. Those whom one governs are people,
individuals, or groups (2004a, p. 126, 2007a, p. 122). A state or a territory might be ruled, but not governed. One of
Foucaults examples is Guillaume de La Perrires Miroir Politique (1555), where he quotes the denition of government as the right
disposition of things arranged so as to lead to a suitable end. He notes that the denition of government does not
refer to territory in any way: one governs things (2004a, p. 96, 2007a, p. 99). This is a complex of men and things,
where government is not related to the territory, though Foucault does clarify to suggest that the qualities of territory might be
important, even if territory in itself is not the object (2004a, p. 96, 2007a, p. 99). While the issue of the qualities of territory is
important, the key issue is population and its various attributes. Another example is his discussion of Giovanni BoterosReason of
Statefrom 1589. Foucault notes that Boteros work suggests state is a stable dominion [dominio fermostrong, rm rule] over
people, which he translates as strong domination [ferme domination] (1596a/1956, Chapter I, 1). Foucault stresses that there is
no territorial denition of the state, it is not a territory, it is not a province or a kingdom
[royaume], it is only people and a strong domination [domination] (2004a, p. 243, 2007a, pp. 237238). In the
Hrodote interview, remember, Foucault had said that domain [domaine] is a juridico-political notion (1994, III, p. 32, 2007b, pp.
176177), explicitly pushing the spatial determination of the term behind the legal. In order to trace the roots of this idea, Foucault
looks at the Christian notion of the pastoral and the biblical idea of the ock: transient, not xed in place or population, but led by a
strong individual (see 1990, pp. 6162, 1994, III, pp. 561562).

MORALISM

Economic engagement frames the US as an uncontested moral center with an
obligation to the less advanced
Shapiro 4 (Michael, Professor of Political Science @ U Hawaii, Methods and Nations, 22-23 //[SG])
Appropriately entitled Sovereign Virtue, Dworkins investigation reaffirms the conventional jurisdictions: the
legal/spatial boundaries of the state and the privileged political discourses that the boundaries entail.
His geophilosophy has, at its center, what he calls the prosperous democracies: which, he states, seem to be able to provide a
decent minimal life for everyone06 As is the case with the geophilosophy on which empiricist comparative politics has been
traditionally predicated, Dworkin fails to discern the unprosperous worlds sequestered within the prosperous onesfor example,
an unregistered alien labor force living a Third World existence within the First World and a surviving Fourth World of
First World Nations that have been transformed into domestic burdens. Ignoring those who do not seem to share,
even minimally, in the prosperity of the prosperous democracies (one can add here the partially employed, the unemployed, those
out in the streets, the thousands without medical benefits), Dworkin seems also unaware that prosperity is a
relationship; among other things, capital flows, which are largely provoked by the prosperous democracies, affect those
outside as well, as capital rearranges identity spaces, opportunity for kinds of work, and land use patterns all over the globe.
Dworkins anaemic model of global political economy, his limited imagination of the forces affecting the lives of the unprosperous, is
doubtless connected with his anaemic version of political life in general. Biopolitically, life for Dworkin features an ethical
individual07 who deserves to be accorded equal opportunity, if not an equal measure of success or pleasure. Where is Dworkins
life deployed? His political lives are constituted as a group of autonomous individuals, enclosed within
the boundaries of static territories. By contrast, when sovereignty is seen as a dynamic process, life becomes unstuck; it
is subject to the vicissitudes of sovereignty-related enactments (an issue to which I turn in chapter 6). Dworkins adherence to a
static geophilosophy, a world of state-enclosed citizen lives, is an affirmation of a static and uncontested territoriality within
which the virtue of fairness is to be deployed. If instead one recognizes sovereignty as episodic, as moments of violent
enforcement or legal affirmation and counter-sovereign episodes of resistance, political life becomes an unstable set of events.
However, before treating the eventualities of politics, which are accessible within the frame of Rancieres notion that the political is
a rare enactment, Dworkins geophilosophy of equality requires further critical attention.

PATRIARCHY

These policies carry with them patriarchal underpinnings the practice of border
security victimizes women and children creating a slave-master relationship that
ensures exclusion
Gill 2010 (Nick. "New state-theoretic approaches to asylum and refugee geographies." Progress in Human Geography 34.5
(2010): 626-645. http://phg.sagepub.com/content/34/5/626.short BRW)
Another difficulty of an essentialized state concept, as set out in the previous section, is the tendency to see the state as the driver of
change, and to overlook the ways in which social factions might be driving legal changes through the state. Giving attention to the
influence not only of state over society but also of society over the state has allowed some geographers to examine in detail
the way in which states, concerned with the regulation of migration, do so subject to the social biases
and dispositions of their national and cultural contexts. In particular, the gendered impact of state interventions in the
migration arena has underscored the degree to which states do not act in a social vacuum, but
instead behave as both conduits and promulgators of social attitudes and biases (Giles and Hyndman,
2004). Commenting on the gender bias of the Canadian state in the migration sphere, Giles and Hyndman (2004: 304) argue that
[M]ost refugee women and children applicants [are] represented as victims or recipients
of humanitarian aid or welfare which tends to promulgate a condescending, needy image of
female asylum seekers and refugees while simultaneously casting the state as a provider and
protector for these women. The radical rejection of a clear distinction between state and society
by asylum and refugee scholars allows attention to also focus upon the strategic absence of the
state as an explicit strategy of refugee control and asylum-seeker exclusion (Hyndman and Mountz,
2007).

THREAT CON

Hegemonic representations of the US feed cyclic threat construction
Slater 97 (David, Ph.D from London School of Economics and
Professor Emeritus of Geography at Loughborough University,
Geopolitical imaginations across the North-South divide: issues
of difference, development and power, Political Geography Vol.
16 Issue 8, November 1997, pp. 631-653, Muse, slim_)
The theory of modernization evolved in a world marked by three major geopolitical conditions: (i) the
emergence of the United States as a global power, hegemonic in the West: (ii) the evolution of a Cold War
and Super-Power rivalry; and (iii) a growing political turbulence in the regions of the Third World,
with decolonization struggles, liberation wars and revolutionary upheavals giving a new ethos to the era.
In particular, in the 1960s with the Cuban Missile Crisis, the Vietnam War and interventions in countries as
diverse as the Dominican Republic and Indonesia, political instability abroad exerted a growing influence on the trajectory
of theoretical enquiry and public policy. Furthermore, turmoil abroad was echoed in conflicts within the United States itself, as
protests against the Vietnam War and internal social injustices, especially as reflected in practices of racial discrimination, grew
in intensity and impact. Overall, the atmosphere of external and internal instability and conflict
affected the ways in which modernization theory was itself constructed. Thus, whilst a first phase dating from the late 1950s was
characterized by a certain optimism, and a steady belief in the real possibilities of successfully diffusing
modernization to the traditional societies of the periphery, of implanting the achieve- ments of Western
development, the growing instabilities, turbulences and conflicts of the 196Os, especially as they became associated
with the emergence of independent political forces within many Third World societies, came to
undermine the sanguine orientations of that initial phase. Increasingly the international scene came to be depicted in
terms of chronic threat, and the shadow of breakdown.





IMPACTS

1NC
IMPERIALISM

This construction of Latin America makes imperialistic violence, war and
destruction inevitable Latin America becomes a playground for neocons to
execute violence
Grandin 06 (Empire's Workshop: Latin America, the United States, and the Rise of the New Imperialism, Greg Grandin,
Macmillan, May 2, 2006 BRW)
The ARGENTINE WRITER Jorge Luis Borges once remarked that the lack of camels in the Koran proves its Middle Eastern
provenance: only a native author, he explained, could have so taken the animal for granted as not to mention it. Perhaps a similar
familiarity explains the absence of Latin America in recent discussions about the United States and its empire. Though Latin
America has played an indispensable role in the rise of the United States to global power, it
elicits little curiosity from its neighbor to the north. "Latin America doesn't matter, Richard Nixon advised a
young Donald Rumsfeld, who was casting about for career opportunities. Long as weve been in it, people dont give one damn
about Latin America.' Likewise today. In their search for historical precedents for our current imperial moment, intellectuals
invoke postwar reconstructions of Germany and japan, ancient Rome and nineteenth-century
Britain but consistently ignore the one place where the United States has projected its influence
for more than two centuries. "People dont give one shit" about the place, Nixon said.: Vi/ere it not
for Borgess insight, this studied indifference to Latin America would seem ironic, for the region has long served as a workshop of
empire, the place where the United States elaborated tactics of extraterritorial administration and acquired its conception of itself as
an empire like no other before it. The Western hemisphere was to be the staging ground for a new empire for liberty," a phrase used
by Thomas Jefferson specifically in reference to Spanish Florida and Cuba. Unlike European empires, ours was supposed to entail a
concert of equal, sovereign democratic American republics, with shared interests and values, led but not dominated by the United
Statesa conception of empire that remains Washingtons guiding vision. The same direction of influence is evident in any number
of examples. The United Statess engagement with the developing world after World War II, for
instance, is often viewed as an extension of its postwar policies in Europe and japan, yet that view has it exactly backwards.
Washingtons first attempts, in fact, to restructure another countrys economy took place in the developing
worldin Mexico in the years after the American Civil War and in Cuba following the Spanish-American War. We should do for
Europe on a large seale, remarked the U.S. ambassador to England in 1914, "essentially what we did for Cuba on a small scale and
thereby usher in a new era of human history. Likewise, most discussions of George W. Bushs foreign policy focus
on the supposed innovation of a small group of neoconservative intellectuals in asserting the
right to unilateral preemptive military action both to defend national security and to advance
American ideals. But neither the neocons dire view of a crisis-ridden world that justifies the use
of unilateral and brutal American military power nor their utopian vision of the same world
made whole and happy by that power is new. Both have been fully in operation in Washingtons approach to Latin
America for over a century. The history of the United States in Latin America is cluttered with
preemptive" interventions that even the most stalwart champions of U.S. hegemony have
trouble defending. From the mid-nineteenth to the early twentieth century, the U.S. military sharpened its
lighting skills and developed its modernday organizational structure largely in constant conflict
with Latin Americain its drive west when it occupied Mexico in the midnineteenth century aml took more than half of that
countrys national territory. And in its push south: by 1930, Washington had sent gunboats into Latin American ports over six
thousand times, invaded Cuba, Mexico (again), Guatemala, and Honduras, fought protracted guerrilla wars in the Dominican
Republic, Nicaragua, and Haiti, annexed Puerto Rico, and taken a piece of Colombia to create both the Panamanian nation and the
Panama Canal. For their part, American corporations and financial houses came to dominate the
economies of Mexico, the Caribbean, and Central America, as well as large parts of South
America, apprenticing themselves in overseas expansion before they headed elsewhere, to Asia,
Africa, and Europe. Yet Latin America did more than serve as a staging ground for the United Statess
early push toward empire. The region provided a school where foreign policy officials and
intellectuals could learn to apply what political scientists like to call soft powerthat is, the
spread of Americas authority through nonnilitary means, through commerce, cultural exchange,
and multilateral cooperation} At first, the United States proved a reluctant student. It took decades of mounting Latin
American anti-imperialist resistance, including armed resistance, to force Washington to abandon its militarism. But abandon it it
finally did, at least for a short time. In the early 1930s, Franklin D. Roosevelt promised that henceforth the United States would be a
"good neighbor," that it would recognize the absolute sovereignty of individual nations, renounce its right to engage in unilateral
interventions, and make concessions to economic nationalists. Rather than weaken U.S. influence in the Western
Hemisphere, this newfound moderation in fact institutionalized Washingtons authority,
drawing Latin .American republics tighter into its political, economic, and cultural orbit through
a series of multilateral treaties and regional organizations. The Good Neighbor policy was the model for the
European and Asian alliance system, providing a blueprint for Americas empire by invitation, as one historian famously described
Washingtons rise to unprecedented heights of world power} But even as Washington was working out the contours of its kinder,
gentler empire in postwar Western Europe and japan, back in the birthplace of American soft power it was rearming. Latin
America has once again became a school where the United States studied how to
execute imperial violence through proxies. After World War II, in the name of containing Communism,
the United States, mostly through the actions of local allies, executed or encouraged coups in, among other places, Guatemala,
Brazil, Chile, Uruguay, and Argentina and patronized a brutal mercenary war in Nicaragua. Latin America became a
laboratory for counter insurgency, as military officials and covert operators applied insights learned in the re-gion to
Southeast Asia, Africa, and the Middle East. By the end of the Cold War, Latin American security forces trained, funded, equipped,
and incited by Washington had executed a reign of bloody terrorhundreds of thousands killed, an
equal number tortured, millions driven into exilefrom which the region has yet to fully
recover. This reign of terror has had consequences more far-reaching than the damage done to
Latin America itself, for it was this rehabilitation of hard power that directly influenced
Americas latest episode of imperial overreach in the wake of 9/1 1. It is often noted in passing
that a number of the current administrations officials, advisers, and hangers-on are veterans of Ronald
Reagans Central American policy in the 1980s, which included the patronage of anti-Communist governments in El Salvador and
Guatemala and anti-Communist insurgents in Nicaragua. The list includes Elliott Abrams, Bushs current deputy national security
adviser in charge of promoting democracy throughout the world; john Negroponte, former U.N. ambassador, envoy to Iraq, and now
intelligence czar; Otto Reich, secretary of state for the Western Hemisphere during Bushs first term; and Robert Kagan, an
ardent advocate of U.S. global hegemony. john Poindexter, convicted of lying to Congress, conspiracy, and destroying
evidence in the IranContra scandal during his tenure as Reagans national security adviser, was appointed by Rumsfeld to oversee
the Pentagons stillborn Total Information Awareness program. john Bolton, ambassador to the United Nations and an arch-
unilateralist, served as Reagans point man in the justice Department to stonewall investigations into Iran-Contra.; Yet the links
between the current Bush administrations revolution in foreign policy and Reagans hard line in Central America are even more
profound than the simple recycling of personnel. It was Central America, and Latin America more broadly,
where an insurgent New Right first coalesced, as conservative activists used the region to
respond to the crisis of the 1970s, a crisis provoked not only by Americas defeat in Vietnam but
by a deep economic recession and a culture of skeptical antimilitarism and political dissent that
spread in the wars wake. Indeed, Reagans Central American wars can best be understood as a dress rehearsal for what is
going on now in the Middle East. It was in these wars where the coalition made up of neoconservatives,
Christian evangelicals, free marketers, and nationalists that today stands behind George W. Bushs
expansive foreign policy first came together. There they had near free rein to bring the full power of the United
States against a much weaker enemy in order to exorcise the ghost of Vietnamand, in so doing, begin the transformation of
Americas foreign policy and domestic culture. A critical element of that transformation entailed shifting the rationale of American
diplomacy away from containment to rollback, from one primarily justified in terms of national defense to one charged with
advancing what Bush likes to call a global democratic revolution. The domestic fight over how to respond to
revolutionary nationalism in Central America allowed conservative ideologues to remoralize
both American diplomacy and capitalism, to counteract the cynicism that had seeped into both
popular culture and the political establishment regarding the deployment of U.S. power in the
world. Thus they pushed the Republican Party away from its foreign policy pragmatism to the idealism that now defines the war
on terror as a world crusade of free-market nation building. At the same time, the conflicts in Nicaragua, El Salvador, and
Guatemala allowed New Right militarists to find ways to bypass the restrictions enacted by Congress and the courts in the wake of
Vietnam that limited the executive branchs ability to fight wars, conduct covert operations, and carry out domestic surveillance of
political activists. The Reagan White House perfected new techniques to manipulate the media, Congress, and public opinion while
at the same time re empowering domestic law enforcement agencies to monitor and harass political dissidents. These techniques, as
we shall see, prefigured initiatives now found in the PR campaign to build support for the war in Iraq and in the Patriot Act,
reinvigorating the national security state in ways that resonate to this day. The Central American wars also provided the New
Christian Right its first extensive experience in foreign affairs, as the White House mobilized evangelical activists in order to
neutralize domestic opponents of a belligerent foreign policy. It was here where New Right Christian theologians
first joined with secular nationalists to elaborate an ethical justification for a rejuvenated
militarism. In other words, it was in Central America where the Republican Party first combined the
three elements that give todays imperialism its moral force: punitive idealism, free-market
absolutism, and right-wing Christian mobilization. The first justified a belligerent diplomacy not just for the sake
of national security but to advance freedom. The second sanctified property rights and the unencumbered free market as the moral
core of the freedom it was Americas duty to export. The third backed up these ideals with social power, as the Republican Party
learned how to channel the passions of its evangelical base into the international arena. 'lb focus, therefore, exclusively on
neoconservative intellectuals, as much of the commentary attempting to identify the origins of the new imperialism does, deflects
attention away from the long history of American expansion. The intellectual architects of the Bush Doctrine are but part of a larger
resurgence of nationalist militarism, serving as the ideologues of an American revanchism fired by a lethal combination of
humiliation in Vietnam and vindication in the Cold War, of which Central America was the tragic endgame.

GEOPOWER

Geopower is at the root of war makes case impacts inevitable
Tuathail 96[Gearoid Professor, Government and International Afffairs, School of Public and International Affairs, Virginia
Tech Critical Geopolitics: the politics of writing global space Page/s 1-2 GooglePrint]
Geography is about power. Although often assumed to be innocent, the geography of the world is not a
product of nature but a product of histories of struggle between competing authorities over the
power to organize, occupy, and administer space. Imperial systems throughout history, from classical Greece and
Rome to China and the Arab world, exercised their power through their ability to impose order and meaning upon space. In
sixteenth-century Europe, the centralizing states of the "new monarchs" began organizing space around an intensified principle of
royal absolutism. In regions both within and beyond the nominal domain of the Crown, the power of royal authority over space was
extended and deepened by newly powerful court bureaucracies and armies. The results in many instances were often violent, as the
jurisdictional ambitions of royal authority met the determined resistance of certain local and regional lords. Within the context of
this struggle, the cartographic and other descriptive forms of knowledge that took the name "geography" in the earls modern period
and that were written in the name of the sovereign could hardly be anything else but political. To the opponents of the expansionist
court, "geography" was a foreign imposition, a form of knowledge conceived in imperial capitals and dedicated to the
territorialization of space along lines established by royal authority. Geography was not something already possessed by the earth
but an active writing of the earth by an expanding, centralizing imperial state. It was not a noun
but a verb, a geo-graphing, an earth-writing by ambitious endocolonizing and exocolonizing states who
sought to seize space and organize it to fit their own cultural visions and material interests. More
than five hundred years later, this struggle between centralizing states and authoritative centers, on the one hand, and
rebellious margins and dissident cultures, on the other hand, is still with us. While almost all of the land of the earth has now,
been territorialized by states, the processes by which this disciplining of space by modern states occurs remain highly contested.
From Chechnya to Chiapas and from Rondonia to Kurdistan and Fast Timor, the jurisdictions of centralized nation-states strive to
eliminate the Contradictions of marginalized peoples and nations. Idealized maps from the center clash with the
lived geographies of the margin, with the controlling cartographic visions of the former
frequently inducing cultural conflict, war, and displacement. Indeed, the rise in the absolute
numbers of displaced peoples in the past twenty-five years is testimony to the persistence of
struggles over space and place. In 1993 the United Nations High Commissioner for Refugees estimated that roughly 1
in every 130 people on earth has been forced into flight because of war and state persecution. In 1970
there were 2.5 million refugees in the world; today that figure is well over 18.2 million. In addition an estimated 24 million people
are internally displaced within their own states because of conflict. More recently, genocide in Rwanda left over 500,000 murdered
and produced an unprecedented exodus of refugees from that state into surrounding states. Refugees continue to be
generated by "ethnic cleansing" campaigns in the Balkans; economic collapse in Cuba; ethnic
wars in the Caucasus; state repression in Guatemala, Turkey, Indonesia, Iraq, and Sudan; and
xenophobic terror in many other states. Struggles over the ownership, administration, and
mastery of space are an inescapable part of the dynamic of contemporary global politics.

WAR


These processes of geographical configuration enhance a social fabric of violence
Gregory, 11 professor of geography at U British Columbia (Derek Gregory, July 2011, The Geographical Journal, The
everywhere war)//CC
For many, particularly in the United States, 9/11 was a moment when the world turned; for others,
particularly outside the United States, it was a climactic summation of a longer history of American imperialism in general and its meddling in the
Middle East in particular. Either way, it is not surprising that many commentators should have emphasised the temporality of the military violence that
followed in the wake of the terrorist attacks on the World Trade Center and the Pentagon on that bright September morning: the war on
terror that became the long war. For the RETORT collective, the invasions of Afghanistan and Iraq
marked the elevation into a state of permanent war of a long and consistent pattern of
military expansionism in the service of empire (RETORT 2005, 80). Keen (2006) wrote of endless war,
Duffield (2007) of unending war and Filkins (2008) of the forever war. The sense of permanence
endures, and yet Engelhardt (2010, 23) ruefully notes that it remains difficult for Americans to understand that
Washington is a war capital, that the United States is a war state, that it garrisons much of the
planet, and that the norm for us is to be at war somewhere at any moment. Bacevich (2010, 225) traces this
state of affairs to what he calls the Washington rules that long pre-date 9/11. These are the conviction that the obligations
of leadership require the United States to maintain a global military presence, configure its armed forces
for power projection, and employ them to impose changes abroad, which he argues have formed the enduring leitmotif of US national security policy
for the last 60 years and propelled the United States into a condition approximating perpetual war. Each of these temporal formulations implies
spatial formations. For RETORT (2005, 103) military neoliberalism is the true globalization of our time. The
planetary garrison that projects US military power is divided into six geographically defined
unified combatant commands like US Central Command, CENTCOM whose Areas of Responsibility cover
every region on earth and which operate through a global network of bases. If you think this unremarkable,
ask yourself Bacevichs question: how would the United States react if China were to mirror these moves?
Think, too, of the zones in which the shadow of US military violence still falls: not just
Afghanistan and Iraq, but also Iran, Libya, Pakistan, Somalia, Yemen. Then think of the zones
where the rhetoric of the war on terror has been used by other states to legitimise repression:
Chechnya, Libya, Palestine, the Philippines, Sri Lanka. And then think of the cities that have become
displacements of the space of war, punctuation points in what Sassen (2010, 37) calls a new kind of
multi-sited war: Casablanca, Lahore, London, Madrid, Moscow, Mumbai. All these lists are incomplete, but even in this truncated
form they suggest the need to analyse not only the forever war but also what we might call the
everywhere war. This is at once a conceptual and a material project whose scope can be indexed
by three geo-graphs that trace a movement from the abstract to the concrete: Foucaults (19756)
prescient suggestion that war has become the pervasive matrix within which social life
is constituted; the replacement of the concept of the battlefield in US military doctrine by the
multi-scalar, multi-dimensional battlespace with no front or back and where everything
becomes a site of permanent war (Graham 2009, 389; 2010, 31); and the assault on the global borderlands
where the United States and its allies now conduct their military operations. The first two are never far from
the surface of this essay, but it is the third that is my primary focus. Duffield (2001, 309) once described the borderlands as an imagined geographical
space where, in the eyes of metropolitan actors and agencies, the characteristics of brutality, excess and breakdown predominate. There, in the
wild zones of the global South, wars are supposed to occur through greed and sectarian gain,
social fabric is destroyed and developmental gains reversed, non-combatants killed,
humanitarian assistance abused and all civility abandoned. This imaginative geography folds in
and out of the rhetorical distinction between our wars wars conducted by advanced militaries
that are supposed to be surgical, sensitive and scrupulous and their wars. In reality, however, the
boundaries are blurred and each bleeds into its other (Gregory 2010). Thus the US-led invasion of Afghanistan in October
2001 combined a longdistance, high-altitude war from the air with a ground war spearheaded by the warlords and militias of the Northern Alliance
operating with US infantry and Special Forces; counterinsurgency in Afghanistan and Iraq has involved the co-option of ragtag militias to supplement
US military operations; and in Afghanistan the US Army pays off warlords and ultimately perhaps even the Taliban to ensure that its overland supply
chain is protected from attack (Report of the Majority Staff 2010). In mapping these borderlands which are also
shadowlands, spaces that enter European and American imaginaries in phantasmatic form,
barely known but vividly imagined we jibe against the limits of cartographic and so of
geopolitical reason. From Ratzels view of der Krieg als Schule des Raumes to Lacostes stinging denunciation la gographie, a sert,
dabord, faire la guerre the deadly liaison between modern war and modern geography has been conducted in resolutely territorial terms. To be
sure, the genealogy of territory has multiple valences, and Ratzels Raum is not Lacostes espace, but a critical analysis of the everywhere war requires
cartographic reason to be supplemented by other, more labile spatialities. This is not only a matter of transcending the geopolitical, connecting it to the
biopolitical and the geo-economic, but also of tracking space as a doing, precarious, partially open and never complete. It is in something of this spirit
that Bauman (2002, 83) identifies the planetary frontierlands as staging grounds of todays wars, where efforts to pin the divisions and mutual
enmities to the ground seldom bring results. In the course of interminable frontierland warfare, so he argues, trenches are seldom dug, adversaries
are constantly on the move and have become for all intents and purposes extraterritorial. I am not sure about the last (Bauman is evidently thinking
of al Qaeda, which is scarcely the summation of late modern war), but this is an arresting if impressionistic canvas and the fluidity conveyed by
Baumans broad brush-strokes needs to be fleshed out. After the US-led invasion of Iraq it was commonplace to distinguish the Green Zone and its
satellites (the US political-military bastion in Baghdad and its penumbra of Forward Operating Bases) from the red zone that was everywhere else. But
this categorical division is misleading. The colours seeped into and swirled around one another, so that
occupied Iraq became not so much a patchwork of green zones and red zones as a thoroughly
militarised landscape saturated in varying intensities of brown (khaki): intensities because within
this warscape military and paramilitary violence could descend at any moment without warning,
and within it precarious local orders were constantly forming and re-forming. I think this is what
Anderson (2011) means when he describes insurgencies oscillating between extended periods of
absence as a function of their dispersion and moments of disruptive, punctual presence, but
these variable intensities entrain all sides in todays wars amongst the people and most of all
those caught in the middle. This is to emphasise the emergent, event-ful quality of
contemporary violence, what Gros (2010, 260) sees as moments of pure laceration that puncture
the everyday, as a diffuse and dispersed state of violence replaces the usual configurations of
war. Violence can erupt on a commuter train in Madrid, a house in Gaza City, a poppy field in
Helmand or a street in Ciudad Juarez: such is the contrapuntal geography of the everywhere
war. It is also to claim that, as cartographic reason falters and military violence is loosed from its
frames, the conventional ties between war and geography have come undone: that, as Mnkler (2005, 3)
has it, war has lost its well-defined contours. In what follows, I propose to take Mnkler at his word and consider three borderlands
beyond Afghanistan and Iraq that illuminate some of the ways in which, since 9/11, late modern war is being transformed
by the slippery spaces within which and through which it is conducted. I focus in turn on Af-Pak, Amexica
and cyberspace, partly because these concrete instances remind us that the everywhere war is also always somewhere (Sparke 2007, 117), and partly
because they bring into view features of a distinctly if not uniquely American way of war.

2NC
2NC IMPERIALISM

Geopolitical representations of inferiority justify imperialist expansionism in the
name of democracy
Slater 97 (David, Ph.D from London School of Economics and
Professor Emeritus of Geography at Loughborough University,
Geopolitical imaginations across the North-South divide: issues
of difference, development and power, Political Geography Vol.
16 Issue 8, November 1997, pp. 631-653, Muse, slim_)
An influential vision of the need to spread the benefits of Western industrial progress and modern development
was expressed by President Truman in his Point Four Program of 1949. In his strategy for peace and freedom, he argued
that we must embark on a bold new program for making the benefits of our scientific advances and industrial progress available for
the improvement and growth of undeveloped areas; more than half the people of the world are living in conditions approaching
misery, and their poverty, he went on is a handicap and a threat both to them and to more prosperous areas. What is to be
envisaged therefore is a program of development based on concepts of democratic fair-dealing. Democracy was seen as a
vitalizing force capable of stirring people into action against their human oppressors and their
ancient enemies of hunger, misery and despair.33 The connection made between poverty and
threat, and the emphasis on democracy as a force against human oppression, capture an
underlying and motivating concern about communism. The primary Second World power, the Soviet Union, the
heartland of communism, was of course seen as a threat that had to be contained. In the late 1940s and subsequently, the Soviet
Union was frequently perceived with a certain duality-a respect for its industrial achievements-as
George Kennan expressed it in his I947 speech on containment, Soviet economic development. can list certain favourable
achievements-and an implacable opposition to its form of government, defined as totalitarian-again in Kennans
words, the Soviet leaders, taking advantage of the contribution of modern technique to the arts of despotism, have solved the
question of obedience within the confines of their power (see Brockway, 1957: 152-153). The view of the Soviet Union as
being geopolitically expansionist, and autocratic, but also capable of industrial, scientific and technological
development, contrasted markedly with the vision of Third World societies as being backward and
in need of tutelage. Symptomat- ically, in the arena of political change, Third World rebellions or post-
revolutionary regimes were frequently interpreted as being the subject of manipulation by
external communist forces, as was, for example, suggested in the Cuban and Nicaraguan casesj (see Figs. 3 and 4).

2NC RESOURCE WARS

This epistemology always results in exeptionalist policies that exploit foreign
countries and result in conflict
Klare 01 (Micheal T. Klare October 2001 - New Geography of Conflict, The [comments] Foreign Affairs, Vol. 80, Issue 3
(May/June 2001), pp. 49-61 Klare, Michael T. 80 Foreign Aff. 49 (2001)
http://heinonline.org.proxy.lib.umich.edu/HOL/Page?handle=hein.journals/fora80&div=44&collection=journals&set_as
_cursor=8&men_tab=srchresults&terms=venezuela| critical|geography&type=matchall BRW)
IN OCTOBER 1999, in a rare alteration of U.S. military geography, the Department of Defense reassigned senior command
authority over American forces in Central Asia from the Pacific Command to the Central Command. This decision produced no
headlines or other signs of interest in the United States but nevertheless represented a significant shift in American
strategic thinking. Central Asia had once been viewed as a peripheral concern, a remote edge of the Pacific Command's main
areas of responsibility (China, Japan, and the Korean Peninsula). But the region, which stretches from the Ural Mountains to China's
western border, has now become a major strategic prize, because of the vast reserves of oil and natural gas thought to lie under and
around the Caspian Sea. Since the Central Command already controls the U.S. forces in the Persian Gulf region, its assumption
of control over Central Asia means that this area will now receive close attention from the people whose primary task is to
protect the flow of oil to the United States and its allies. The new prominence of Central Asia and its potential oil riches is
but one sign of a larger transformation of U.S. strategic thinking. During the Cold War, the areas of greatest
concern to military planners were those of confrontation between the U.S. and the Soviet blocs: central and southeastern Europe and
the Far East. Since the end of the Cold War, however, these areas have lost much strategic significance for the United States (except,
perhaps, for the demilitarized zone between North and South Korea), while other regions-the Persian Gulf, the Caspian Sea basin,
and the South China Seaare receiving increased attention from the Pentagon. Behind this shift in strategic geography
is a new emphasis on the protection of supplies of vital resources, especially oil and natural gas.
Whereas Cold War-era divisions were created and alliances formed along ideological lines, economic competition now
drives international relations-and competition over access to these vital economic assets has
intensified accordingly. Because an interruption in the supply of natural resources would portend severe economic
consequences, the major importing countries now consider the protection of this flow a significant national concern. In addition,
with global energy consumption rising by an estimated two percent annually, competition for
access to large energy reserves will only grow more intense in the years to come. Accordingly, security
officials have begun to pay much greater attention to problems arising from intensified competition over access to critical materials-
especially those such as oil that often lie in contested or politically unstable areas. As the National Security Council observed in the
White House's 1999 annual report on U.S. security policy, "the United States will continue to have a vital interest in ensuring access
to foreign oil supplies." Therefore, the report concluded, "we must continue to be mindful of the need for regional stability and
security in key producing areas to ensure our access to, and the free flow of, these resources." CONCERN OVER access to
global resources has, of course, long been an important theme in U.S. security policy. In the 189os, for
example, the nation's preeminent naval strategist, Captain Alfred Thayer Mahan, won widespread support for his argument that the
United States required a large and capable navy in order to bolster its status as a global trading power. This perspective also shaped
the geopolitical thinking of Presidents Theodore Roosevelt and Franklin Delano Roosevelt. During the Cold War, however,
resource concerns were often subordinated to the political and ideological dimensions of the
U.S.-Soviet rivalry. It is only now, with the Cold War safely over, that securing access to vital materials
has again assumed a central position in American security planning. Evidence of this revival of interest in
resources was especially plentiful during last year's global shortage of petroleum and natural gas. President Bill Clinton flew to
Africa in August 2000 with the hope of obtaining additional oil from Nigeria--currently one of America's leading suppliers-and
prodded the Caspian states to accelerate the construction of new pipelines to Europe and the Mediterranean. Meanwhile, then
Governor George W. Bush used the presidential campaign debates to call for oil and gas exploration in U.S.
wilderness areas in order to reduce the nation's dependence on foreign supplies. Once elected, he made
one of his earliest foreign policy initiatives a meeting with Mexican President Vicente Fox to discuss proposals for increasing the flow
of energy from Mexico to the United States. A similar focus on the acquisition or protection of energy supplies is evident in the
strategic thinking of other powers. Large energy importers, such as China, Japan, and the major European powers, have
made ensuring the stability of their supplies a top priority. Russia is placing greater foreign policy emphasis on energy-producing
areas of Central Asia. Although Moscow continues to worry about developments on its western frontiers in the areas facing NATO, it
has devoted considerable resources to strengthening its military presence in the south, in the Caucasus (including Chechnya and
Dagestan), and among the former Soviet Central Asian republics. Similarly, the Chinese military has shifted its concentration from
the northern border with Russia to Xinjiang in the west (a potential source of oil) and to offshore areas of the East and South China
Seas. Japan has followed China to these seas and has boosted its own ability to operate there, procuring and deploying new warships
and a fleet of missile-armed P-3C Orion patrol planes. Securing access to sufficient supplies of oil and gas is also a great concern of
the newly industrializing nations of the developing world, such as Brazil, Israel, Malaysia, Thailand, and Turkey-many of which are
expected to double or triple their energy consumption rates over the next 20 years. Although obtaining sufficient supplies of energy
is becoming the foremost resource priority for some states, the pursuit of adequate water will be the central focus for others. Water
supplies are already insufficient in many parts of the Middle East and Southwest Asia; continued population growth and the
increased likelihood of drought from global warming will likely create similar scarcities elsewhere. To further complicate the
problem, water supplies do not obey political boundaries, and so many of the countries in About four-fifths of the these regions must
share a limited number of major water sources. With all the states that world's known petroleum touch these waters seeking to
increase their reserves lie in politically allotted supplies, the danger of conflict over unstable or contested competition for these
shared supplies will inevitably grow. areas. In other parts of the world, localized conflicts have broken out for control of valuable
timber and minerals. Typically, these conflicts entail a struggle between competing elites or tribes over the income derived from
commodity exports. In Angola and Sierra Leone, for instance, rival groups are battling for control of lucrative diamond fields; in the
Democratic Republic of the Congo (D.R.C.), the conflict concerns copper as well as diamonds; and in parts of Southeast Asia, various
groups are fighting over valuable stands of timber. The recent bloodshed on Borneo arose from clashes between the indigenous
Dayak, who have long occupied Borneo's extensive forests, and settlers from Java and Madura who were brought in by the
Indonesian government to harvest all this timber. Although not a direct threat to the security of the major powers, these conflicts
can lead to the deployment of U.N. peacekeeping forces-as in Sierra Leone-and thus impose significant demands on the world's
capacity to manage ethnic and regional violence. All of these phenomena-increased competition over access to major sources of oil
and gas, growing friction over the allocation of shared water supplies, and internal warfare over valuable export commodities-have
produced a new geography of conflict, a reconfigured cartography in which resource flows rather than political and ideological
divisions constitute the major fault lines. Just as a map showing the world's tectonic faults is a useful guide to likely earthquake
zones, viewing the international system in terms of unsettled resource deposits-contested oil and gas fields, shared water systems,
embattled diamond mines-provides a guide to likely conflict zones in the twenty-first century. POLITICAL ANALYSTS
have yet to devise a model that accurately represents the global power dynamic of the post-Cold
War world. A comprehensive and predictive explanation of this dynamic must account for a variety of shifts in power politics
and conflict zones. The bipolar face-off of the Cold War has been reconfigured to leave one global superpower--the United States-
facing a group of smaller power centers, from western Europe to Russia, China, and Japan. In the early 199os, violence in the former
Yugoslavia, Kashmir, and Central Africa made the world community concentrate on preventing ethnic and intercommunal conflict,
but this focus on ethnicity could not predict or address the violence in Africa over control of diamond fields, copper mines, and
farmlands. Economic globalization is turning some poor areas into centers of prosperity and growth but leaving
others behind in abject poverty, sparking conflicts that have more to do with resources than with
nationalism. In short, contemporary world affairs defy exclusively political, security-related, and economic definitions. A better
analysis of stresses in the new international system, and a better predictor of conflict, would view international relations through the
lens of the world's contested resources and focus on those areas where conflict is likely to erupt over access to or the possession of
vital materials.

2NC HUMAN TRAFFICKING

The militarization of the U.S.-Mexico is the root cause of traffickings sexualized
violence
Falcon 01 (Sylvanna, Professor of Latin America Studies at the University of California Santa Cruz, Rape as a Weapon of
War: Advancing Human Rights for Women at the U.S.-Mexico Border, Social Justice, Vol. 28, No. 2, pg. 31-32, accessed via
JSTORE, [SG])
From the Conquest to the present, women have been targeted in gender specific ways during
militarized conflict. In every militarized conflict, women are systematically raped or sexually
assaulted. Some feminist scholars and advocates contend that rape is not about sex, but rather about power and the
dehumanization of women (Woodhull, 1988). By international standards, rape is a war crime, a form of
torture, and a link to genocide. Tadeusz Mazowiecki, the former Special Rapporteur for the United Nations (U.N.)
Commission on Human Rights, released a document on the former Yugoslavia that classified rape as "an abuse of power and
control in which the rapist seeks to humiliate, shame, embarrass, degrade, and terrify the victim. The primary objective is
to exercise power and control over another person" (U.N. Economic and Social Council, 1993a: 71). In this
article, I argue that rape is one outcome of militarization along the U.S.-Mexico border. I examine
specific cases of militarized border rape using data from nongovernmental organizations, government committees, and U.S.
newspapers.1 I also analyze the factors that facilitate militarized border rape and emphasize the need to advance human rights for
women in the border region. Each of the women in the case studies took some form of action against the Immigration
and Naturalization Service (LNS). Some even used an advocate to move their cases forward through an investigation. All of the
cases involved INS officials or Border Patrol agents. Though the cases highlighted do not include U.S. military or paramilitary
forces, the influence of military culture on Border Patrol agents has affected that agency. Rape is a weapon of war and
militarization at the border indicates that a form of war exists. Data indicate that some men have reported being raped at the
border (Amnesty International, 1998), but most rapes violate women, whether at the border or throughout the world.
Motivations for raping women differ in a war-torn country from those committed along the
U.S.-Mexico border. However, the outcome remains the same: the systematic degradation of
women. National concern over the border has led to broad public support for militaristic tactics in this region. The
militarization of the U.S.-Mexico border involves two key elements: the introduction and
integration of military units in the border region (the War on Drugs is the primary motivator for involving
military units) and the modification of the Border Patrol to resemble the military via its
equipment, structure, and tactics. Cynthia Enloe (2000: 3) contends that militarization involves cultural,
institutional, ideological, and economic transformations. The INS has undergone these transformations. For example, transferring
the INS from the Department of Labor to the jurisdiction of the Department of Justice (DOJ) in 1940 resulted in institutional and
ideological shifts (Dunn, 1996: 13). Various Department of Defense (DOD) Authorization Acts loosened the restrictions placed on
the military's domestic enforcement roles. The DOD Authorization Act of 1982 started the process of altering a 100-year-old statute
that prohibited cooperation between the army and civilian law enforcement. This had a major impact on the role of
the military in domestic affairs and encouraged an alliance between civilian law enforcement
and the military. Other DOD Authori? zation Acts advanced and expanded this cooperation. In addition, other
national actions, such as Operation Alliance and Joint Task Force 6 advanced the militarization of the border, especially after 1986
when President Reagan declared drug trafficking to be a national security threat (Ibid.). Militarized antidrug strategies
influence the policies for undocumented border crossers who are not involved in drug
trafficking. For example, Operation Hold the Line and Operation Gatekeeper focus on the points of entry frequented
by undocumented people in El Paso, Texas, and San Diego, California. Both of these border enforcement efforts contain
militaristic characteristics. Timothy Dunn employs low-intensity conflict (LIC) military doctrine to contextualize the militarization
of the U.S.-Mexico border. LIC doctrine advo? cates "unconventional, multifaceted, and relatively subtle forms of
militarization" and emphasizes "controlling targeted civilian populations." LIC doctrine, con? structed by the U.S. military-
security establishment to target Third World upris? ings and revolutions, particularly in Central America, contains the following
three aspects: (1) an emphasis on the internal defense of a nation, (2) an emphasis on controlling targeted civilian populations
rather than territory, and (3) the assumption by the military of police-like and other unconventional, typically nonmilitary roles,
along with the adoption by the police of military characteristics (Dunn, 1996: 21).


2NC VTL

Attempts to organize space in a specific way in order to fit western centric
interests results in a devaluation of others
Gregory, 5 professor of geography at U British Columbia (Derek Geography, 2005, Progressive Hum Geography, The role of
geography in public debate, 2005 29: 165)//CC
So, to return to the original question: what kind of geography for what kind of public debate? In The colonial present (Gregory, 2004a) I identified
three imaginative geographies that continue to be central to the 'war on terror'. War is not the only possible response to
terrorism, and neither is it the most effective, and many geopolitical and geoeconomic
calculations enter into the decision to resort to spectacular and sustained military violence. But
war also requires a cultural mobilization - the inculcation ofa sense of common purpose and
public conviction that identifies an Enemy and legitimizes the loss of life (on both sides).
Imaginative geographies are powerful rhetorical weapons precisely because they
fold difference into distance. Yet they do not only produce a series of performative spacings
between 'us and 'them', 'white on 'black: their topologies also produce a vast grey zone in which
indifference is folded into indistinction. I want to sketch the imaginative geographies that were
mobilized to convert the 'war on terror into a mission civilisatrice, in each case describing them in active terms as
spatial strategies, in order to identify in minimalist form three countergeographies that might produce counterpublics informed by other, less
destructive maps of meaning (Gregory, 2004a; 2004b). First, 'locating' mobilized a technical or technocultural register
in which opponents were routinely reduced to mute objects in a purely visual field - letters on a
map, coordinates on a grid - that produced an abstraction ofother people as 'the Other'. By this means,
American bombs and missiles rained down on K-A-B-U-L not on the eviscerated city of Kabul; Israeli troops turned their guns on
Palestinian 'targets' not on Palestinian men, women and children; American firepower
destroyed Baghdad buildings and degraded the Iraqi military machine but somehow never killed
Iraqi people who were effaced from the scene. Against these reductions that hollow out places, both figuratively and physically, I urge the
continued development of contextual geographies that affirm the materiality and corporeality ofplaces and attend to the voices (and the silences) of
those who inhabit them. Before the US invasion of Iraq, Baghdad was presented to an American audience
as a city of targets: those blank circles that pockmarked satellite photographs in newscasts, newspapers, and websites. It was only when
Saddam's statue was toppled that it was allowed to appear as a city ofneighborhoods, inhabited not by tyrants, torturers, and terrorists, as you might
have expected, but by doctors, engineers, shopkeepers: people very much like you and me (Gregory, 2004a: 213-15). I am not surprised by the reversal,
but think what could have happened had that countergeography been affirmed before the invasion. How might the public have viewed the war then?
For this very reason, the production of such a contextual geography faces obstacles that are more than intellectual. Within the United States,
criticism of the 'war on terror and the racisms that underwrite so much of its violence has
spawned a series of willfully ignorant attacks on area studies, and in particular Middle Eastern
Studies (see Prashad, 2003; Lockman, 2004; Heydemann, 2 0 04; Turse, 2 0 04). The silence of our own associations has been shocking, but I hope
that geographers everywhere will come to the aid of those scholars - all ofthem geographers-with-little-gs in one way or another -who have invested so
much of their lives in studying the languages and landscapes, cultures and histories of other people and other places. For their offence should be our
offence too: to refuse the brutal reduction ofother places and other people to counters in a calculus of self-interest and opportunism, and instead to
affirm the importance of a careful geography of engagement and understanding. Again, this is not to blunt criticism, merely to establish the ground for
a critique that is open, dialogical and informed. The second spatial strategy was 'opposing' or 'inverting', which
mobilized a largely cultural register in which antagonism was reduced to a conflict between a
unitary and universal Civilization (epitomized by the United States) and multiple, swarming barbarisms that
were its negation and nemesis. America, with its proxies and allies, was thus called to take up
arms against the gathering forces of darkness, of Evil incarnate. Osama bin Laden and Saddam
Hussein became Doppelgdnger, inversions of the face of Goodness reflected in the White House
mirror, with fateful consequences for the people ofAfghanistan and Iraq, while fundamentalists on both the
Christian and the Zionist Right saw the redemption of the biblical Land of Israel and the dispossession of the Palestinians as fulfilling God's ultimate
purpose. All were barbarians to be summarily dispatched. Against this dismal logic of
exclusion and estrangement, I urge the construction of con trapuntal geographies - the term is derived
from Said (1993: 67) - that explore the webs ofconnection and affiliation, the myriad ways in which we are all involved in the lives (and deaths) of
millions of unknown others. The reason I am still a geographer is that I want my students to know about - to care about - the lives of distant strangers,
people whom they do not know but without whom their own lives would be impossible (cf Ignatieff, 1984; Corbridge, 1993). The complex topologies of
the commodity chain show that contrapuntal geographies are unlikely to be simple or transparent - as Harvey
once remarked, you cannot see the fingerprints of exploitation on the grapes in the supermarket - and their shape-shifting duplicity
is conveyed by a series of imaginative geographies, in part constructed through advertising
campaigns and in part assimilated as part of a taken-for-granted vernacular (Castree, 2001). In a haunting
novel written before 9/11, Giles Foden captured the contrapuntal geographies that are written in the political register when he described 'the endless
etcetera of events which led from dead Russians in Afghanistan, via this, that and the other, through dead Africans and Americans in Nairobi and Dar,
to the bombardment of a country with the highest levels of malnutrition ever recorded' (Foden, 2002: 335-36). There will always be those with reasons
to erase connections like these, of course, and there were attempts to snap the webs that tied successive US administrations, via the CIA, the activities
of multinational corporations, and the business ties ofthe Bush family, through the foreign and domestic policies of Saudi Arabia and Pakistan, to the
Soviet occupation of Afghanistan, the arming and financing of the mujaheddin, and the rise of al-Qaeda and the Taliban. These were remarkably
successful; the public reaction to 9/11 showed how easily the capacity for distraction is aggravated
by grief and anger. It soon became clear that any acceptable answer to the repeated question
'Why do they hate us? - which was itself a rhetorical device of considerable power - was to be
found uniquely among 'them', the sort of people 'they' are, and not among 'us'. This is an
effective way of producing identity and identification, but its excision of the contrapuntal
geographies in play provides neither a rigorous analysis of political violence nor an effective
response to it. The third spatial strategy, 'excepting', mobilized a political-juridical register, in
which not only armed opponents - al-Qaeda terrorists, Taliban troops, Palestinian fighters, Iraqi
soldiers - but also civilians and refugees were reduced to the status of what Giorgio Agamben identifies as
homines sacri. Their lives did not matter. The sovereign powers of the American, British and Israeli states disavowed or
suspended international law so that men, women, and children were made outcasts, placed beyond the pale and beyond the protections and affordances
of the Modern. The deaths of American, British, and Israeli citizens mattered (unless ofcourse they were killed opposing or witnessing the bloody wars
in Afghanistan, Palestine, or Iraq: think of Rachel Corrie, Tom Hundall, and the journalists who were targeted during the Iraq war). In this grisly
colonial calculus the deaths ofAfghans, Palestinians, and Iraqis were rendered not only
uncountable - the US-led coalitions constantly proclaimed they did not do body counts, and the
murders of Palestinians in the occupied territories rarely provoke a tremor of concern in the
mainstream media- but also unaccountable. They were trapped in a zone ofindistinction,
where those who observed them were utterly indifferent to their fate. Against this, I urge the elaboration of
cosmopolitan geographies. Cosmopolitanism is a vexed term, and I cannot do justice to its complexitie s here (cf Harvey, 2 0 0 0), but one starting point
is Susan Sontags call for an understanding 'that human beings everywhere do terrible things to one other' (Sontag, 2003). In other words, without
dissolving the specfic injuries and horrors of 9/1 1, or suicide bombings or military violence
elsewhere in the world, we need to struggle against seeing them as special. Sontag argues
that the pain of others is not somehow less than our own, and that it is only through this regard -
rather than the detached gaze or glance - that it is possible to retain our own humanity.13 That is another awkward term, of course, and Agamben is
properly critical of the modern anthropological machine that produces our seemingly commonsensical idea of 'the human', but this should direct our
attention to its silent production of all those excluded 'others'- the barbarians, the savages, the monsters - who are to have no claim on our sympathies,
what Paul Gilroy (2003) calls 'all the other shadowy "third things" lodged between animal and human [that] can only be held accountable under special
emergency rules and fierce martial laws' (see also Agamben, 2004; Gregory, 2004c). Against this colonial economy of meaning,
as part of what we might think of as a 'geographical compact', we must hold ourselves to the
same laws and standards as we do others, and extend to them the same rights and affordances
that we extend to ourselves. How else can we turn the logics of effacement and estrangement
into an ability to face the strange that inheres within - and makes possible - 'our' selves and 'our'
spaces? It is this unsettling, haunting, demanding hospitality that provides the very horizon of
the political (Derrida and Dufourmantelle, 2000; cf Amin, 2003). Finally, all three of these countergeographies
require, in addition and as an essential moment in their production, collaborative geographies:
critical work done not merely for the people we write about but with them. It is only by this
means that we will ever be fully engaged in both public debate and a genuinely democratic
politics, and it is only by this means that 'progress in human geography' can yield to a truly
progressive human geography.


2NC THREAT CON

Military geography are sanctioned by discourse of threatthis causes collusion
between knowledge and power at the expense of lived experience
Morrissey, 11 professor of geography at the National University of Ireland, 2011, Antipode, Vol. 43 No. 2 pp. 435-470)//CC
Collusion between knowledge and power must be forcefully exposed, as must the
purposes to which bureaucracy bends knowledges specialization. When institutional (academic)
knowledge sets itself up above lived experience...catastrophe is in the offing. Catastrophe is indeed already
upon us (Lefebvre 1991:415). Henri Lefebvre may have been writing in 1974 but his perceptive thoughts are perhaps as vital today as ever. The
specialized knowledges of the militarystrategic studies complex have long been patronized,
prioritized and actioned by the US military. The cosy collusion between the Pentagon and
militarystrategic studies has been instrumental in the contemporary production of military
space. Reductive scriptings of national security, abstracted geopolitical visions and dreams of empire
have collectively served to occlude geographies of the lived experience (Chandrasekaran 2006;
Packer 2005). As Bradley Klein (1994:3) reminds us, questions of war and peace are too important to leave to students [and practitioners] of Strategic
Studies. Strategic studies knowledges have long been above lived experience, yet their power has been
instrumental in unleashing catastrophe, terror and abject misery for the very people whose lives
they are above. But clearly there is catastrophe for us too: the catastrophe of being
overwhelmed by the collusion of power and knowledge, the catastrophe of the militant and
deeply unequal world in which we live and the catastrophe of inactionpolitically, discursively
and otherwise. But of course there has been action, with some of the most significant resistance taking place outside the academy,
such as that seen in the unprecedented global protests against the Iraq War in February and March 2003,
and continued anti-war activism worldwide since then. Geographers and other academics have of course
been variously actively involved. Within the academy, geographers have illuminated key aspects of the US-
led war against militant Islam, including its place-making strategies, its territorial responses
to terrorist attacks and its exceptional legal and biopolitical geographies (Coleman 2003; Elden 2007; Morrissey 2011;
Reid-Henry 2007). Others have revealed the imperial historical geographies of contemporary geopolitics, and signalled its geoeconomic underpinnings
(Cowen and Smith 2009; Harvey 2003; Kearns 2006; Smith 2003a). In addition, geographers have depicted the violent
geographies of recent western military interventions (Dalby 2006; Flint 2005; Graham 2005; Gregory and Pred 2007).
And focus has been placed too on the state discourses of military power and broader imaginative
and affective geographies legitimating that violence (Bialasiewicz et al 2007; Hannah 2006; O Tuathail 2003;
Woodward 2005). Such counter-geographies are important, yet their disruptive power, as Matthew Sparke notes (2007:347), is perhaps ultimately
practically limited. In spite of the above work, and after a cultural turn in the US military that has produced a powerful
rhetorical effect that justifies more killing to stop the killing (Gregory 2008a:21), reductive vernaculars,
reifying essentialist tropes of terror, threat, correction and security still prevail and discursively underpin the war in Iraq and
broader war on terrorism. The militarystrategic studies complex plays a central role in advancing such discourses, and possesses vital forums through
which to enunciate their endgame: legitimized state violence. I want to conclude more positively, however, by suggesting ways to effectively oppose
them. As an academic working in political geography, a key starting point of resistance for me is the
careful detailing of the largely unseen inner workings of empire in our contemporary world, ultimately in order
to be better able to resist it (which is what this paper has been about). That resistance can manifest itself in
counter-scriptings in a variety of contexts, from lecture halls to town halls, from academic
journals to online blogs. And in a variety of public forums, many geographers have played, and continue to
play, important roles in critiquing the war on terror and advancing more nuanced, reasoned and
humane geographies and histories of Islam and the Middle East (Gregory 2005). Such academic and public
intellectual work can also crucially liaise with, learn from, and be transformed by grassroots
activists in peace and social justice movements throughout the world.44 And linking to their work
in our teaching especially has more power than perhaps we sometimes realise; especially given
the multimedia teaching and learning tools available today.45

2NC ORIENTALISM

Violent geography sustains orientalism.
Morrissey, 11 professor of geography at the National University of Ireland, 2011, Antipode, Vol. 43 No. 2 pp. 435-470)//CC
The reductive imaginative geographies of the militarystrategic studies complex not only
support the operations of US geopolitical and geoeconomic calculation in the Middle East;
they also contribute to a pervasive and predominant cultural discourse on the region that
has all the hallmarks of Orientalism (Gregory 2004; Little 2002; Said 2003; Shapiro 1997). National security
specialist commentaries have long enunciated the threat of Islamic fundamentalism in the
Middle East and linked it to the feared potential of new political and economic orders
emerging in the region (Lewis 1995; Roberts 1995). Since the war on terrorism began, such sentiment has been
relentlessly championed in broader popular media circles; a development that has had
grave consequences. As Stephen Graham (2005:6, 8) notes, the result of the combined vitriol of a whole legion of US
military commentators who enjoy huge coverage, exposure, and influence in the US media is a world in which whole
populations are positioned as unworthy of any political or human rights: 24 In the construction of people as
inhuman terrorist barbarians understanding little but force, and urban places as
animalistic labyrinths or nests demanding massive military assault, Islamic cities, and
their inhabitants, are, in turn, cast out beyond any philosophical, legal, or humanitarian
definitions of humankind or civilisation.

2NC VIOLENCE

The alternative solves violence remapping the world in solidarity with the
persons who inhabit its space can challenge violent discursive practices
Springer, 11 Professor of Geography at Ottag U (Simon Springer, 2011, Political Geography, Violence sites in places?
Cultural neoliberal rationalism, and virulent imaginative geographies, 30)//CC
The movement of neoliberalism towards economic orthodoxy, and its eventual capture of such hegemony,
was not only achieved through dissemination of its class project geographically through shocks or
otherwise, but also by spreading its worldviews across various discursive fields (Plehwe & Walpen, 2006). Through
this merger of discourse and an imperative for spatial diffusion, neoliberalism has constructed virulent imaginative
geographies that appeal to commonsense rhetorics of freedom, peace, and democracy through the
destructive principles of Orientalism, and in particular by proposing a static and isolated place-based culture of violence thesis in the context of the
Other. These representations of space and place are never merely mirrors held up to somehow
reflect or represent the world but instead enter directly into its constitution (and destruction). Images
and words release enormous power, and their dissemination. can have the most acutely material
consequences (Gregory & Pred, 2007: 2). Neoliberalism is a discourse, and words do damage as actors perform their
scripted roles. But neoliberalism is also a practice that has actually existing circumstances (Brenner & Theodore, 2002) where new violences
are created. Thus, the global south has become the theater of a multiplicity of cruel little wars that, rather
than barbaric throwbacks, are linked to the current global logic (Escobar, 2004: 18). Yet there is nothing quintessentially
neoliberal about Orientalism. Its entanglement with the neoliberal doctrine is very much dependent upon the context in which neoliberalization
occurs. Initially conceived during the Enlightenment, and later revived in the postwar era, neoliberalism had a western birth, radiating outwards
across the globe as the sun was setting on Keynesian economics. Orientalism is, however, entangled in the project of imperialism, which is supported
and perhaps even impelled by impressive ideological formations that include notions that certain territories and people require and beseech
domination, as well as forms of knowledge affiliated with domination (Said, 1993: 9). As the latest incarnation of empire (Hardt & Negri, 2000;
Pieterse, 2004), the principles, practices, theories, and attitudes of a particular class-based faction maintaining economic control over various
territories remains intact under neoliberalism and so we should not be too surprised to discover that the pernicious discourses that support such
resurgent imperialism similarly remain unchanged (Hart, 2006). If, as Richard Peet (2000: 1222) argues, economic rationality is a symbolic logic
formed as part of social imaginaries, formed that is in culture, then like the project of colonialism, and indeed in keeping with the Self-expanding logic
of capital and its fundamental drive to capture new sites for (re)production (Harvey, 2005), neoliberalism is intimately bound up in articulating and
valorizing cultural change. Yet in order for such change to be seen as necessary, the irrationality of the Other must be discursively constructed and
imagined. This is precisely where neoliberalism and Orientalism converge. Neoliberalization proceeds as a civilizing enterprise; it is the confirmation
of reason on barbarians who dwell beyond. Reason, like truth, is an effect of power, and its language developed out of the Enlightenment as an
antithetical response to madness, or the outward performances of those seen as having lost what made them human (Foucault, 1965). Reason as such,
triumphs at the expense of the non-conformist, the unusual, the Other. As a consequence, neoliberal ideas are proselytized to rescind the ostensible
irrationality and deviance of the Other. A closely related second reason for evangelism relates to the purported wisdom of neoliberalism, which
repeatedly informs us that we have never had it as good as we do right now, and thus Others are in need of similar salvation. If they are to be ruled,
whether by might or by markets, they must become like us. This theology of neoliberalism maintains a sense of rationalism precisely because it looks
to reason rather than experience as the foundation of certainty in knowledge. As Neil Brenner and Nik Theodore (2002: 353) argue, the manifold
disjunctures that have accompanied the worldwide imposition of neoliberalismebetween ideology and practice; doctrine and reality; vision and
consequenceeare not merely accidental side effects of this disciplinary project. Rather, they are among its most essential features. In other words, the
effects of neoliberalization (poverty, inequality, and mythic violence) are ignored (Springer, 2008), and in their place a commonsense utopianism is
fabricated (Bourdieu, 1998). And so we stand at the end of history (Fukuyama, 1992), or at least so we are told, wherein the monotheistic imperative of
one God gives way to one market and one globe. Yet the certainty of such absolutist spatio-temporality is in every respect chimerical. Space and time
are always becoming, invariably under construction. The future is open, and to suggest otherwise is to conceptualize space as a vast lacuna.
There are always new stories yet to be told, new connections yet to be made, new contestations yet to erupt,
and new imaginings yet to blossom (Massey, 2005). As Said (1993: 7) argued, Just as none of us is outside or
beyond geography, none of us is completely free from the struggle over geography. That struggle
is complex and interesting because it is not only about soldiers and cannons but also about
ideas, about forms, about images and imaginings. This sentiment applies as much to the geographies of neoliberalism as it
does to violent geographies. If so much of the worlds violence is made possible through virulent imaginings, then perhaps the first step towards peace
is a collective imagining of nonviolence. Undoubtedly, this is an exercise made possible though culture via human agency because, [i]f violence has
meaning, then thosemeanings can be challenged (Stanko, 2003: 13). Yet conceiving peace is every bit as much a geographic project. Violence
sits in places in a very material sense, we experience the world though our emplacement in it,
where violence offers no exception to this cardinal rule of embodiment. But there is no predetermined plot to the stories-so-far of
space, the horizons of place are forever mercurial, and geographies can always be re-imagined. Geography is not destiny any more
than culture is, and as such the possibility of violence being bound in place is only accomplished
through the fearful and malicious imaginings of circulating discourses. Put differently, it is the performative
effects ofOrientalism and other forms ofmalevolent knowledge that allow violence to curl up andmake itself comfortable in particular places.What
can emerge from such understandings is a principled refusal to exclude others from the sphere
of the human and an appreciation of how violence compresses the sometimes forbiddingly
abstract spaces of geopolitics and geo-economics into the intimacies of everyday life and the innermost
recesses of the human body (Gregory & Pred, 2007: 6). Violence is not the exclusive preserve of the Other rooted in the supposed
determinism of either biology or culture; it populates the central structures of all societies. The capacity for violence exists within
the entirety of humanity, but so too does its opposite, the rejection of violence. There are choices to be
made each moment of every day, and to imagine peace is to actively refuse the exploitative structures, virulent ideologies, and geographies of death that
cultivate and are sown by violence. This emancipatory potential entails challenging the discourses that
support mythic violence through a critical negation of the circuits it promotes, and nonviolent engagement
in the sites e both material and abstract e that it seeks to subjugate. It requires a deep and committed sense of
Selfreflection to be able to recognize the circuitous pathways of violence when it becomes
banal, systematic, and symbolic. And it involves the articulation of new imaginative geographies rooted not in the architectures of
enmity (Gregory, 2004a), but in the foundations of mutual admiration, respect, and an introspective sense of humility. By doing so, we engage
in a politics that reclaims the somatic as a space to be nurtured, reproduces familiar and not so
familiar geographies through networks of solidarity built on genuine compassion, and rewrites local
constellations of experience with the poetics of peace.


cartographic critique is a prerequisite to understanding violence as more than just
institutional or locational
Springer, 11 Professor of Geography at Ottag U (Simon Springer, 2011, Political Geography, Violence sites in places?
Cultural neoliberal rationalism, and virulent imaginative geographies, 30)//CC
A perennial complication of discussions about human suffering is the awareness of cultural
differences. In the wake of the damage wrought by Samuel Huntington (1993), some might contend
that the concept of culture is beyond reclamation (Mitchell, 1995), especially with respect to discussions of violence. There
is, however, still a great deal of resonance to the concept that can, and perhaps must be salvaged if we are to ever make sense of violence. If
culture is defined as a historically transmitted form of symbolization upon which a social order
is constructed (Geertz, 1973; Peet, 2000), then understanding any act, violent or otherwise, is never
achieved solely in terms of its physicality and invariably includes the meaning it is afforded by
culture (Scheper-Hughes & Bourgois, 2004). An account of the cultural dimensions of violence is perhaps
even vital, as focusing exclusively on the physical aspects of violence transforms the project into
a clinical or literary exercise, which runs the risk of degenerating into a pornography of
violence (Bourgois, 2001) where voyeuristic impulses subvert the larger project of witnessing, critiquing, and writing against violence. While
violence in its most fundamental form entails pain, dismemberment, and death, people do not
engage in or avoid violence simply because of these tangible consequences, nor are these
corporeal outcomes the reason why we attempt to write or talk about violence. Violence as a
mere fact is largely meaningless. It takes on and gathers meaning because of its affective and
cultural content, where violence is felt as meaningful (Nordstrom, 2004). To write poetry after Auschwitz is barbaric, Theodor Adorno (1981:
34) once famously wrote. Confounded by the atrocities that had occurred under the Nazis, he failed to understand how a humanity capable of causing
such catastrophic ruin could then relate such an unfathomable tale. Although struck by the emotional weight of violence, Adorno was wrong, as it is not
poetry that is impossible after Auschwitz, but rather prose: Realistic prose fails, where the poetic evocation of the unbearable atmosphere of a camp
succeeds. That is to say, when Adorno declares poetry impossible (or, rather, barbaric) after Auschwitz, this impossibility is an enabling impossibility:
poetry is always about something that cannot be addressed directly, only alluded to (Zizek, 2008: 4e5). For victims, any retelling of violence is
necessarily riddled with inconsistency and confusion. The inability to convey agony and humiliation with any sense of clarity is part of the trauma of a
violent event. Indeed, physical pain does not simply resist language but actively destroys it, bringing about an immediate reversion to a state anterior
to language, to the sounds and cries a human being makes before language is learned (Scarry, 1985: 4). As such, the chaotic bewilderment of
experiencing violence makes understanding it an unusually mystifying endeavor. Thus, what can we say about violence without being overwhelmed by
its unnerving horror and incapacitated by the fear it instills? How can we represent violence without becoming so removed from and apathetic towards
its magnitude that we no longer feel a sense of anguish or distress? And in what ways can we raise the question of violence in relation to victims,
perpetrators, and even entire cultures, without reducing our accounts to caricature, where violence itself becomes the defining, quintessential feature of
subjectivity? To quote Adorno (1981: 34) once more, Even the most extreme consciousness of doom threatens to degenerate into idle chatter. The
confounding effects of violence ensure that it is a phenomena shot through with a certain perceptual blindness. In his monumental essay Critique of
Violence,Walter Benjamin (1986) exposed our unremitting tendency to obscure violence in its
institutionalized forms, and because of this opacity, our inclination to regard violence
exclusively as something we can see through its direct expression. Yet the structural violence
resulting from our political and economic systems (Farmer, 2004; Galtung, 1969), and the symbolic
violence born of our discourses (Bourdieu 2001; Jiwani, 2006), are something like the dark matter of
physics, [they] may be invisible, but [they have] to be taken into account if one is to make sense
of what might otherwise seem to be irrational explosions of subjective [or direct] violence (Zizek,
2008: 2). These seemingly invisible geographies of violence e including the hidden fist of the market
itself e have both nonillusory effects (Springer, 2008) and pathogenic affects in afflicting human
bodies that create suffering (Farmer, 2003), which can be seen if one cares to look critically enough.
Yet, because of their sheer pervasiveness, systematization, and banality we are all too frequently
blinded from seeing that which is perhaps most obvious. This itself marks an epistemological downward spiral, as the
economic in particular is evermore abstracted and its real world implications are increasingly erased from collective consciousness (Hart, 2008).
The clearest available example of such epistemic violence, Gayatri Spivak (1988: 24e25) contends, is the
remotely orchestrated, farflung, and heterogeneous project to constitute the colonial subject as
Other, and it is here that the relationship between Orientalism and neoliberalism is revealed.
Since Orientalism is a discourse that functions precisely due to its ability to conceal an
underlying symbolic violence (Tuastad, 2003), and because the structural violence of poverty and
inequality that stems from the political economies of neoliberalism is cast as illusory (Springer,
2008), my reflections on neoliberalism, Orientalism, and their resultant imaginative and
material violent geographies are, as presented here, purposefully theoretical. As Derek Gregory (1993: 275)
passionately argues, human geographers have to work with social theory. Empiricism is not an option,
if it ever was, because the facts do not (and never will) speak for themselves, no matter how
closely. we listen. Although the facts of violence can be assembled, tallied, and categorized, the
cultural scope and emotional weight of violence can never be entirely captured through
empirical analysis. After Auschwitz, and now after 9/11, casting a sideways glance at violence through the poetic abstractions of theory must
be considered as an enabling possibility. This is particularly the case with respect to understanding the geographies of violence, as our understandings
of space and place are also largely poetic (Bachelard, 1964; Kong, 2001). Imaginative bindings of space: geography and narrative Despite
the attention space and place receive in contemporary human geography, Massey (2005) has
convincingly argued that there is a prevailing theoretical myopia concerning their
conceptualization. Space and place are typically thought to counterpose, as there exists an
implicit imagination of different theoretical levels: space as the abstract versus the
everydayness of place. Place, however, is not the Other of space, it is not a pure construct of the
local or a bounded realm of the particular in opposition to an overbearing, universal, and
absolute global space (Escobar, 2001). What if, Massey (2005: 6) muses, we refuse this distinction, between place (as meaningful, lived
and everyday) and space (as what? the outside? the abstract? the meaningless?). By enshrining space as universal, theorists
have assumed that places are mere subdivisions of a ubiquitous and homogeneous space that is
dissociated from the bodies that occupy it and from the particularities that these bodies len[d]
to the places they inhabit (Escobar, 2001: 143). Such disregard is peculiar since it is not the
absoluteness of space, but our inescapable immersion in place via embodied perception that is
the ontological priority of our lived experience. Edward Casey (1996: 18) eloquently captures this notion in stating that, To
live is to live locally, and to know is first of all to know the places one is in. The inseparability of space and time entails a further recognition that places
should be thought of as moments, where amalgamations of things, ideas, and memories coalesce out of our embodied experiences and the physical
environments in which they occur to form the contours of place. As such, Massey (2005) encourages us to view space as the simultaneity of stories so
far, and place as collections of these stories, articulations within the wider power-geometries of space. The production of space and
place is accordingly the unremitting and forever unfinished product of competing discourses
over what constitutes them (Lefebvre, 1991). Violence is one of the most profound ongoing stories
influencing the (re)production of space. Similarly, individual and embodied narratives of
violence woven out of a more expansive spatial logic may become acute, forming constellations
that delineate and associate place. Accordingly, it may be useful to begin to think about violent
narratives, not simply as stories about violence, but rather as a spatial metaphor analogous to
violent geographies and in direct reference to Masseys (2005) re-conceptualization of space and
place. Allen Feldman (1991: 1) looks to bodily, spatial, and violent practices as configuring a unified language of material signification, compelling
him to treat the political subject, particularly the body, as the locus of manifold material practices. To Feldman approaching violence from its site of
effect and generation (agency) is to examine where it takes place, thereby embedding violence in the situated practices of agents. Violence is bound up
within the production of social space (Bourdieu, 1989), and because, by virtue of spatiality, social space and somatic place continually predicate each
other, the recognition of violence having a direct bearing on those bodies implies a geography of violence. Foucault (1980: 98) has argued
that individuals are the vehicles of power, not its points of application, and this is precisely
howpower and violence depart, as individuals are at once both the vehicles of violence and its
points of application. In the end, because the body is where all violence finds its influence e be it
direct and thus obvious to the entangled actors, or structural and thus temporally and spatially
diffused before reaching its final destination at and upon the embodied geographies of human
beings e place is the site where violence is most visible and easily discerned. Yet violence is only one facet of
the multiple, variegated, and protean contours of place. So while violence bites down on our lived experiences by affixing itself to our everyday
geographies and by colonizing our bodies, violence itself, much like culture, is by no means restricted to place, nor is place static. Thus, the place-based
dynamics of violence that seemingly make it possible to conceive a culture of violence actually render this notion untenable precisely because of places
relationality and proteanism. The embodied geographies of experience (including violence) that exist in places stretch their accounts out through other
places, linking together a matrix of narratives in forming the mutable landscapes of human existence (Tilley, 1994). This porosity of boundaries is
essential to place, and it reveals how local specificities of culture are comprised by a complex interplay of internal constructions and external exchange.
In the face of such permeability an enculturation of violence is certainly conceivable. All forms of violence are not produced by the frenzied depravity of
savage or pathological minds, but are instead cultural performances whose poetics derive from the sociocultural histories and relational geographies of
the locale (Whitehead, 2004). Violence has a culturally informed logic, and it thereby follows that because culture sits in places (Basso, 1996; Escobar,
2001), so too does violence. Yet the grounds on which some insist on affixing and bounding violence so firmly to particular places in articulating a
culture of violence argument are inherently unstable.1 The shifting, kaleidoscopic nature of space-time demonstrates the sheer impossibility of such
attempts. So while it is important to highlight the emplacement of all cultural practices (including violence), whereby culture is carried into places by
bodies engaged in practices that are at once both encultured and enculturing (Escobar, 2001), it is only through a geographical imagination constructed
on a parochial agenda, rooted in colonial modes of thought, and dislocated from the dynamic material underpinnings of place that a culture itself can be
caricatured as violent. In short, while violence forms a part of any given culture, it is never the sole defining feature.

2NC DEMOCRACY

We solve bottom-up democracyconnecting geography to activism creates a more
healthy expression of everyday power relations which are comparatively more
important to reproducing space
Springer, 11 Lecturer at Department of Geography at the University of Otago (Simon Springer, 2011, Antipode, Public
Space as Emancipation: Meditations on Anarchism, Radical Democracy, Neoliberalism and Violence, Vol. 43 Iss. 2 pp. 525-
562)//CC
Radical democracy is a messy process with an inherent uncertainty reflecting the essential
agonism of open public discussion concerning community principles, and the possibility of
sudden changes, conflicts, and contradictions in collective goals. The spaces of democratic
societies must always be in process, constructions to be maintained and repaired as the
collective interest is defined and contested (Entrikin 2002). This processual nature of public space explains
why it is and must be the subject of continuous contestation, spanning a fluid spectrum between debate, protest, agonism and
at times, lamentably antagonism and violence. Accordingly, it is paramount to view public space as a
medium allowing for the contestation of power, focusing on issues of access ranging from
basic use to more complicated matters, including territoriality and symbolic ownership
(Atkinson 2003). Public space is never a complete project, but is instead both the product and
site of conflict between the competing ideologies of order
(authoritarianism/archy/representation of space) and unscripted interaction
(democracy/anarchy/representational space) (Lefebvre 1991; Mitchell 2003b).10 These competing
approaches do not result in dichotomous public spaces. Rather, emphasis must be placed on the processual
and fluidic character of public space, where any recognizable outcome from either the
ordered or the unscripted is necessarily temporary, that is, a means without end. Although
claiming to advocate democratic public space, Carr et al (1992:xi) exemplify the ordered approach by
suggesting public space is the setting for activities that threaten communities, such as crime and protest. The ability to
protest is what makes public space democratic as it provides those without institutionalized
power the opportunity to challenge the status quo. Crime, for its part, is most often conceived in terms of
property rights, and accordingly the poor and propertyless are repeatedly cast as transgressors of public space. Hee and Ooi
(2003) take a different approach to the ordered view, contending that the public spaces of colonial and post-colonial cities are
constructions of the ruling elite. Certainly, colonial administrators and incumbent regimes enforce their
representations of space, but this ignores the element of contestation and the possible
emergence of representational space. Beijing's Tiananmen Square offers a case in point, as the
people took this controlled space, and, although recaptured by the state, it remains
ideologically contested in the public sphere, continuing to fire the imagination of social
movements in China and beyond (Lees 1994). Thus, the values embedded in public space are
those with which the demos endows it (Goheen 1998), not simply the visualizations and administrations of
reigning elites. States, corporations, and IFIs may challenge collectively endowed values and
espouse the ordered view because they seek to shape public space in ways that limit the
threat of democratic power to dominant socioeconomic interests (Harvey 2000). Although total
control over public space is impossible, they do attempt to regulate it by keeping it relatively
free of passion (Duncan 1996). To remove the passion from public space, corporate or state
planners attempt to create spaces based on a desire for security more than interaction and
for entertainment more than democratic politics (Goss 1996), a process Sorkin (1992) calls the end of
public space. Under the ordered view of public space, premised on a need for surveillance and control
over behavior, representations of space come to dominate representational spaces. The
processes of increasing surveillance, commodification, and private usage are known in the literature as the
disneyfication of space, where the urban future looms as a sanitized, ersatz architecture devoid
of geographic specificity (Lees 1994:446). In this light, the struggle for democracy is inseparable
from public space, as where things are said is at least as important as what is said, when it is
said, how it is said, and who is saying it. Thus, shielding oneself from political provocation is easily achieved
when all the important public gathering places have become highly policed public space, or its corollary, private property
(Mitchell 2003a). Relentlessly confronting the arrogation of public space is imperative, because
the entrenched power of capital can only be repealed through agonism, whereby a multiplicity of
subject positions may be recognized as legitimate claimants to the spaces of the public (Mouffe 2006). When the seemingly
everyday, yet disneyfied performances of capitalism are ignored as normative values, unexceptional practices, and quotidian
sequences they are lent the appearance of insignificance. This is the center of Lefebvre's (1984:24) critique of everyday life,
where such taken for granted succession helps to explain why neoliberalism is often understood as an inevitable, monolithic
force. Such a view ignores how hegemony, understood in the sense advocated by Laclau and Mouffe (2001), is a
discursively constructed strategy, reproduced through everyday practices that are often
oppressive, yet frequently go unnoticed as such. This suggests that neoliberalism proceeds through a
dialectic of coercion and co-optation, which has significant implications for public space. Most often public space is
not the site of momentous clashes between archy and demos, but rather a site of mundanity
and routinized conduct. Consequently, everyday life as it is mediated through the continual
(re)production of space (Lefebvre 1991), is also the terrain in which power is reified, manipulated,
and contested (Cohen and Taylor 1992). It is the everyday forms and uses of public space that inform
those moments when extraordinary contestation becomes manifest. So while public protests
may initially appear limited in scope, they are often expressions of latent dissatisfactions,
which in the current moment, are related primarily to the strains of neoliberalism.

2NC PSYCHOANALYSIS

establishing the US as a safe space is a psychic retreat they infuse the border as a
home space which exacerbates their anxiety and turns the case
Blandy and Sibley, 10 professors at the School of Law of the University of Leeds (Sarah Blandy and David Sibley, 2010,
Social and legal studies, Laws boundaries and the production of space, 19(3))//CC
The psychoanalyst, Melanie Klein, suggested that a strong sense of boundary separ-ating the good and the
bad constituted a defence against psychotic anxieties, anxieties which we all experience to some degree
(Segal, 1992; Steiner, 1987). Modern Kleinians generally agree that we oscillate between psychotic and depressive
anxieties, the latter being associated with a concern for others (initially the mother). Thus, we could
associate psychotic anxiety with a desire to close off and separate from the outside, both socially
and spatially, because society or the space beyond is fantasized (in the conscious mind) and phantasized (in
the unconscious) as containing disturbing or unsettling elements. These may include racialized
groups, homeless people, the mentally ill, nomads, people defined by their sexuality, and so on.
The list varies from place to place and over time but all these groups are defined negatively through simplified,
stereotypical images and they are all potentially transgressive. According to Klein, these groups
represent the bad in ourselves which we project on to others. Kleins thesis does not imply, for example, that
all white people are racist or all heterosexuals are homophobic, but that this primitive splitting of good and bad can
never be entirely removed from the psyche (see e.g. Chapter 6 of Robert Youngs Mental Space (1994), on projective
space and the racial other, and Joel Kovel (1995) on racism and psychoanalysis). The Kleinian analyst John Steiner (1993) has
suggested that some people get stuck in the psychotic mode and inhabit what he terms psychic
retreats. These could be imagined literally as material spaces, such as a cave or a house, or the retreat could
be an organization which provides comfort and a sense of protection, such as the mafia or boy scouts. We
could reasonably add that deep anxiety, manifest in psychic retreats, contributes to the creation of
material spaces which are strongly bounded, as a defence against anxiety. However, we could also
argue, following Freud (1919/ 1985), that the illusion of security created by a strongly bounded space
only reproduces the problem. Distancing from othered groups and a lack of familiarity and appreciation of
the humanity of others, do nothing to dispel negative stereotypes, so the creation of secure or homely
spaces brings with it a sense of potential transgression, the unheim- lich or unsettling. Thus, anxiety
is maintained or even exacerbated through the efforts made to reduce it.


ALTS

FOUCAULT

1NC

The alternative is to reject the 1AC in order to politicize the affirmatives
conception of geography discourse analysis solves
Jones, Jones, and Woods, 04 (Martin Jones* - PhD in Human Geography from the University of Manchester, Rhys
Jones; Professor of Human Geography at the University
of Wales Aberystwyth** - Professor in Human Geography @ the University of Wales Aberystwyth, Michael Woods*** - PhD in
Human Geography from Bristol University; Professor of Human Geography and Director of the Institute of Geography and Earth
Sciences @ the University of Wales Aberystwyth, 2004, AN INTRODUCTION TO POLITICAL GEOGRAPHY Space, place and
politics, http://118.97.161.124/perpus-
fkip/Perpustakaan/Geography/Geografi%20manusia/Pengantar%20Geografi%20Politik.pdf) MD
Far more influential have been two conceptual developments which served to further politicise
the outlook of human geography as a whole. The rst of these was the so-called cultural turn of the late 1980s
and 1990s which promoted a new understanding of culture as the product of discourses through which people signify their identity
and experiences and which are constantly contested and renegotiated (see Jackson 1989; Mitchell 2000). Consequently, issues
of power and resistance were positioned as central to the analysis of cultural geographies,
generating signicant clusters of research on questions of identity and place, including national
identity and citizenship; conflict and contestation between cultural discourses; geographies of
resistance; the role of landscape in conveying and challenging power; and micro-geographies
of politics, including investigation of the body as a site of oppression and resistance (see for example Pile and Keith 1997;
Sharp et al. 2000). These themes are discussed further in Chapters 5, 7 and 8. Moreover, the new cultural geography
drew on the conceptual writings of post-structuralist thinkers such as Michel Foucault, Jacques Derrida,
Gilles Deleuze and Flix Guattari, and postcolonial theorists such as Homi Bhabha, for whom the relation of power
and space was a key concern (see Box 1.3). A number of different strands of post-structuralist thought have been
introduced into political geography, including ideas about difference in research on the cultural politics of identity and the use of
Derridas method of deconstruction in critical geopolitics (see below). However, it is the work of Michel Foucault that has arguably
had the greatest influence in political geography, in particular through the development and application of two key concepts. The
rst of these is discourse, which Foucault redened as referring to the ensemble of social practices through
which the world is made meaningful but which are also dynamic and contested (Box 1.4). In books such as The Order
of Things (1973 [1966]) and The Archaeology of Knowledge (1974 [1969]) Foucault examined the articulation of discursive
practices and thus established precedents as to how discourses might be analysed. These ideas have been fundamental to the
development of geographical work on cultural politics and of critical geopolitics, as well as to the development of
discourse analysis as a methodological approach which is now widely used across political
geography. The second key concept is governmentality, by which Foucault refers to the means by which
government renders society governable. Governmentality is essentially about the use of particular apparatuses of
knowledge and has been employed in recent years in work on the state and citizenship (see Chapter 8). A signicant
aspect of both discourse analysis and governmentality is the potential they allow
for exploration of the incorporation of space itself as a tool in the exercise of
power. Much of Foucaults writing was concerned with power, but he rejected conventional notions of power as a property
that is possessed, focusing instead on how power is exercised and how it circulates through society. Foucault stated that
space is fundamental in any exercise of power (Rabinow 1984: 252), and this principle underlies much of his
work on disciplinary power. His best known illustration of this is his discussion of Jeremy Benthams panopticon (Foucault 1977:
ch. 3). The panopticon was a proposal for an ideal prison, the spatial arrangement of which would effectively force prisoners to
discipline themselves. The panopticon would be built in a circular arrangement with all the cells facing a central observation tower.
The circle meant that prisoners could not see or communicate with each other, but also by means of backlighting from a small
external window it allowed prisoners to be constantly visible via a large internal window from the observation tower, whose own
windows had blinds to prevent prisoners seeing in. The prisoners could not know whether they were being watched at any
particular time, but had to presume that they were under constant surveillance and therefore act within the rules. As Foucault
describes, the major effect of the Panopticon [is] to induce in the inmate a state of conscious and permanent visibility that assures
the automatic functioning of power. So to arrange things that the surveillance is permanent in its effects, even if it is
discontinuous in its action; that the perfection of power should tend to render its actual exercise unnecessary; that this
architectural apparatus should be a machine for creating and sustaining a power relation independent of the person who exercises
it. (Foucault 1979: 201)

2NC
Our analysis is key
Amit, 10 (Dotan Amit University of British Columbia, April 7, 2010, Putting it on the Map: Imperial Gazing and
Cartographic Meaning, https://circle.ubc.ca/bitstream/id/85316/Putting_it_on_the_Map_-
_Imperial_Gazing_and_Cartographic_Meaning.pdf) MD
Elaborating on the significance of citation to geography, Tuathail writes how Cartesian perspectivalism (erroneously)
reinforces the differentiation of the visual (sight) from the textual (cite), as Descartes assumed
a divine congruence between language and the world of transparent objects (sites).18 A traditional
ocularcentric or Cartesian perspectivalism, then, assumes that through sight a subject is able to
discern transparent and inert spatial sites. Put differently, Cartesianism is an ideological lense
that assumes that there is no lense; one thinks I see it, therefore it must be as I see it. Tuathail
proposes, however that the ocularcentric world of sight is a world that is already
infested with textual cites. 19 Though the imperial activity of geo-graphing assumed
otherwise, the maps they produced were constructed from knowledge circumscribed by the
numerous contingencies of knowledge acquisition. The [cartographic texts]did not present
truth, nor [did] the maps constitute panopticons. The [imperial powers] simply believed that they did20 in
accordance with their own citational repetitions. Critical geography , according to Tuathail, must
problematize the relationship between subject, object, and text , orthat between sight,
sites, and cites.21 The following historical examples of imperial cartographic practices in Asia and the Americas will show how
cartographic descriptions grounded in Cartesian perspectivalism could not maintain their
illusion of detachment and neutrality when examined according to the full equation of
sight/sites/cites.

Only an embracement of the geography of resistance through dispossession are we
able to resolve the impacts of the kritik
Sparke 08 (Matthew Sparke published may of 2008 Political geography political geographies of globalization III: resistance
* Department of Geography, Box 353550, University of Washington, Seattle, WA 98105, USA
http://search.proquest.com.proxy.lib.umich.edu/pqrl/docview/230709084/13EE75FEC2A14F99E92/1?accountid=14667 BRW)
Writing on the geography of resistance is especially indicative of the widened field of political geography.
Indeed, breaking the frame of what counts as political might even be argued to be one of the
major theoretical lessons of studying resistance without romance, a point not coincidentally advanced by
Michel Foucault in The history of sexuality (1980). It is worth remembering thus how, after suggesting it is time for scholars
to cut off the head of the king in analysing power, Foucault proceeded to use a series of spatial metaphors to
argue against imagining a single site of resistance opposite and bounded away from a single site of sovereignty: Where there is
power, there is resistance, he said, and yet or rather consequently, this resistance is never in a position of
exteriority in relation to power ... Points of resistance are present everywhere in the power
network. Hence there is no single locus of Great Refusal, no soul of revolt, source of all
rebellions, or pure law of the revolutionary (Foucault, 1980: 9596). Of course, there has been considerable
philosophical debate about whether Foucaults own rather romantic refusal to replay disciplinary power left him unable to critique
forms of violence (such as abusive paedophilia) perpetrated in the name of resistance (Hoy, 2004; on the passionate remaking of
romance by Foucault, Miller, 1993). But for geographers who have sought to trace the lived geographies that
Foucault mapped with metaphor, his arguments against the binary law of the romantic
revolutionary (ie, the oppositional rubric bounding the powerless off from sovereign power) have been profoundly
enabling. Most notably (albeit unlikely to be noted because of its own refusal to subject itself to the demands of academic human
capital metrics that privilege short articles, short editorials and long CVs) Chris Philos insanely big book on the space reserved for
insanity shows how a conceptualization of power as productive, networked and capillary is in turn productive of a thoroughly
nonromantic account of subjectifi cation spaces (Philo, 2004). Tellingly there is no entry on resistance in the index of his tome and
Philo scrupulously avoids depicting the targets of the mad business as souls of revolt, but his concluding hopes point carefully
beyond despair to imagine how the dispossession of the mad through both liberal institutionalization and neoliberal
deinstitutionalization might be avoided by providing a profusion of care spaces that allow people suffering with mental disease to
repossess space and take their place in everyday life as human beings at ease with themselves, their loved ones and their
immediate surroundings (Philo, 2004: 660). Moving from dispossession to repossession (without either
denying hierarchies that subjugate nor asserting the homogeneity of rational choice possessive
individualism) seems in turn to represent a useful model for refl ecting without romance on
political geographies of resistance more globally. Thus, beginning here with representations that examine
resistance as a response to dispossession, and then turning to others addressing the dilemmas of academic engagement with
struggles for repossession, the following two main sections return repeatedly to the ways in which recommendable work on
resistance registers both the hierarchy and heterogeneity of power relations at the very same time In his convincing accounts of the
new imperialism (Harvey, 2003) and neoliberalism (Harvey, 2005), David Harvey argues that both involve and, indeed,
interconnect in expanding accumulation by dispossession. It is a useful phrase that, as Jim Glassman (2006) has described in
further detail, has a long intellectual heritage in Marxist thought: stretching from Marx himself on primitive accumulation (also
Perelman, 2000), to Rosa Luxemburg, to the dependency school theorists, to recent studies by geographers on topics
ranging from the sexual politics of expanded social reproduction (Mitchell et al ., 2004) to the expansive ecological politics
of free-trade agreements (McCarthy, 2004). As Glassman indicates with his fi nal refl ections on how accumulation by
dispossession thereby relates to the complex diversity of new social movements, it is also a hinge category that, by opening the
door to concurrent extra-economic accounts of dispossession, invites the critical
supplementation of Marxian theory (also Sidaway, 2007). In other words, alongside economic exploitation
and all the other dispossessing dynamics unleashed by a social system predicated on the profit
motive, accumulation by dispossession invites critics to examine the role played by racial, sexual
and other social power dynamics in co-determining capitalist dispossession through extra-
economic oppression (Sparke et al ., 2005). Of course, one implication of accumulation by dispossession as a term is that it
is still an accumulative and, as such, economic imperative that remains the primary frame for theorizing dispossession. However, if
we reject such economic apriorism, and if we are adequately attuned to the accumulation of accounts of ongoing
extraeconomic dispossession by those struggling globally for social justice, an altogether more salutary possibility
emerges. In short, we can thereby use the formula to point to ties between very different global
forms of injury and injustice without on the one side falling prey to economic reductionism or on
the other of pretending that the capitalist connections can be ignored. This is an argument that Gillian Hart has fleshed out with
especial ethnographic lan in her elaboration of relational comparisons (Hart, 2006). She argues that: Accumulation
through dispossession may be a useful first step in highlighting the depradations wrought by
neoliberal forms of capital, but it needs to be infused with concrete understandings of specific
histories, memories, meanings of dispossession. To be grasped as an ongoing process,
dispossession also needs to be rendered historically and geographically specific, as well as
interconnected and these specificities and connections can do political as well as analytical work
(Hart, 2006: 988)

Our analysis is key to expose violent geographical discourse
Amit, 10 (Dotan Amit University of British Columbia, April 7, 2010, Putting it on the Map: Imperial Gazing and
Cartographic Meaning, https://circle.ubc.ca/bitstream/id/85316/Putting_it_on_the_Map_-
_Imperial_Gazing_and_Cartographic_Meaning.pdf) MD
Elaborating on the significance of citation to geography, Tuathail writes how Cartesian perspectivalism (erroneously)
reinforces the differentiation of the visual (sight) from the textual (cite), as Descartes assumed
a divine congruence between language and the world of transparent objects (sites).18 A traditional
ocularcentric or Cartesian perspectivalism, then, assumes that through sight a subject is able to
discern transparent and inert spatial sites. Put differently, Cartesianism is an ideological lense
that assumes that there is no lense; one thinks I see it, therefore it must be as I see it. Tuathail
proposes, however that the ocularcentric world of sight is a world that is already
infested with textual cites. 19 Though the imperial activity of geo-graphing assumed
otherwise, the maps they produced were constructed from knowledge circumscribed by the
numerous contingencies of knowledge acquisition. The [cartographic texts]did not present
truth, nor [did] the maps constitute panopticons. The [imperial powers] simply believed that they did20 in
accordance with their own citational repetitions. Critical geography , according to Tuathail, must
problematize the relationship between subject, object, and text , orthat between sight,
sites, and cites.21 The following historical examples of imperial cartographic practices in Asia and the Americas will show how
cartographic descriptions grounded in Cartesian perspectivalism could not maintain their
illusion of detachment and neutrality when examined according to the full equation of
sight/sites/cites.


INTERROGATION


1NC

The alternative is a rejection of the affs spatial relegation in order to open up
debate to discuss the social relationships that form geographical epistemology
Fraser 08 (Fraser, Alistair GEOGRAPHY AND LAND REFORM
Fraser, AlistairGeographical Review98.3 (Jul 2008): 309-321.
ttp://search.proquest.com.proxy.lib.umich.edu/docview/225330
662?accountid=14667 BRW)
The critique advanced by some geographers is that MLAR approaches tend to ignore "the long history of
land acquisition through thievery, personal connections, and domination and overlook the
obstacles to individual well-being caused by overwhelming inequality in access to land" (Wolford
2005,257). MLAR approaches therefore do little to undermine the power of dominant groups or classes; indeed, they tend to benefit
existing elites rather than the poorest or the most land hungry. In Wolford's words, "the neoliberal policy of market-led
agrarian reform privileges the status quo (supporting the owners who defend their right to land
in part because they have land) rather than modifying the inequitable distribution of land" (p.
257). But, as restitution in Levubu, South Africa, also indicates, the notion that neoliberal influences are completely dominant has to
be questioned. In Levubu the state's approach to settling restitution claims has entailed a more interventionist stance that reflects
the state's attempt to become the guardian of land-reform beneficiaries (Fraser 20073); departures from the neoliberal-style MLAR
model can therefore lead to hybrid approaches to land reform. If two areas of interest in the literature are notions of community and
the influence of neoliberalism on land reform, nowhere near as much has been said about the impact on/of land reform of/on
gender relationships. This is definitely not to suggest that gender relationships have been ignored in
geographical studies of land reform. Haripriya Rangan and Mary Gilmartin, for example, use a range of materials to
provide a detailed account and incisive critique of the place of women's rights in South Africa's land reform (2002). They cite in
particular a "Constitution contradiction" that simultaneously accords equal rights to men and women and endorses
traditional customary rule that facilitates discrimination against women in the former homeland
areas. Goebel also addresses the place of women amid land reform in Zimbabwe (2005). A special issue of the Journal of Agrarian
Change (republished as Agrarian Change, Gender and Land Rights [Razavi 2003] ), partly addressed gender relations, though not
with a sufficient focus on their geographical dimensions. Future research would do well to further correct this lacuna. ISSUES OF
CONCERN FOR FUTURE GEOGRAPHICAL WORK ON LAND REFORM Land reform is indeed a "many-splendored thing"
(Wolford 2007). Its study necessarily demands consideration of abstract concepts such as politics and the state, accumulation and
the market, human interactions with nonhuman actors and objects, subject formation, culture, and representation. Attention to all
of these issues demands a type of analysis that places them in their broader geohistorical context. Geographers should be well placed
to achieve effective results in the study of land reform. They should bring to studies of land reform a heightened degree of sensitivity
to space and place-two key concepts in understanding the geohistorical contexts of land-reform efforts. Geographers are
trained to "think geographically," the practice of which demands a particular type of intellectual
rigor stemming from geography's ontological basis and which is markedly different from other
branches of the academy, including those disciplines with longer traditions of contributing to
the literature on land reform. Geographical thinking entails recognizing, acknowledging, and
theorizing unevenness, differences across space, and the range of inter sections and crosscutting
social relationships that connect people and places. Arguably, land reform is best comprehended
when it is viewed geographically. Toward deepening geography's engagement with land reform, therefore, I underpin
the following points, which are intended to generate discussion and/or debate, with the question of whether the
geography-of-land-reform literature has any sort of future.


IMMIGRATION


1NC

The alternative is a rejection of the plan only rejecting the paid politics of border
security can open up room for democratically informed politics
Gill 2010 (Nick. "New state-theoretic approaches to asylum and refugee geographies." Progress in Human Geography 34.5
(2010): 626-645. http://phg.sagepub.com/content/34/5/626.short BRW)
Authors disagree about which interest group has reacted most effectively. For example, Bigo (2002) traces the Europeanization of
asylum policy to a security discourse, claiming that the transformation of security and the consequent focus on
immigrants is directly related to security professionals interests, defined in terms of competition for
budgets and missions (Bigo, 2002: 64). This contrasts with Koslowskis (2001) emphasis upon EU bureaucrats control of the
asylum policy field within Europe, Quassolis (1999) identification of local magistrates as the key players, Favell and Geddes (2000)
identification of NGO and military interest groups, and Guiraudons (2003) discussion of diplomats pivotal role. Freeman, writing
in the American context, has added industrialists and immigrant groups to the categories of state actors competing for the capture of
border policies (Freeman, 2001). What these disagreements reflect, paradoxically, is the consensus that the actors who
populate the state are diverse, competitive and, above all, distinguishable not as state actors
but by their particular social roles (Favell and Geddes, 2000). The category of state actor threatens to
obscure these important distinctions. Related to the potential of a reified state to obscure the political sociology within
states is, third, the risk that an essential state concept might also obscure the agency of individuals working within the state. While
there has long been awareness of the tension between structure and agency in the production of state effects among state theorists,
renewed emphasis is being put upon volitional agency within the state infrastructure by a number of authors (see Mitchell, 1999;
Painter, 2006). Within geography, for example, Jones (2007) appeals for greater attention to be paid to the
personal politics and immediate cultural context of key powerful decision-makers within the
state, drawing upon historical studies around Welsh nationalism and devolution to illustrate the decisive influence that
individuals convictions and social positions can have over nation-state formation. Within poststructural geographies of
the state in particular, a rich vein of research has also opened up concerning the degree to which states rely upon
social reproduction through mundane and repetitive practices in local contexts (Ferguson and Gupta,
2002; Painter, 2006; Mountz, 2007). Again, central to state power in these accounts are the people who enact states and put them to
work. Deciphering how these key state actors view themselves and view the state that they seek to
produce opens up a layer of productive research that refuses to take the everyday, situated state
for granted. A fourth reason to treat accounts of an essentialist state cautiously concerns the contingency of policy genesis and
policy outcomes upon social and cultural circumstances. In the context of immigration policy, political theorists have
argued that Britains parliamentary system transmits pressure from local constituencies to the national level fairly rapidly (Money,
1999). By contrast, neocorporatist institutional structures in Scandinavian countries insulate liberal elites
from the whims of mass publics, depoliticizing the migration issue (Brochmann and Hammar, 1999).
These constitutional differences between countries affect the likelihood of democratically informed
state policies emerging, the details of their content and the likelihood of their success. Societal
histories and circumstances can also affect the ways in which state policies are implemented on the ground: different polities may
share similar ideological stances towards immigration but may carry out radically different
practices in order to pursue these stances. Andreas (1998: 612), for example, contrasts the steel fences and stadium
lights of the Mexican American border with their notable absence from the German-Polish frontier, arguing that the combined
legacy of the countrys authoritarian past and recent memories of the Berlin Wall inhibit the use of more high-profile policing and
surveillance methods. IV Alternative understandings of the state for an emerging critical asylum geography Aware of the risks of
essentializing the state in the ways described, there have been a number of developments both outside and
within geography that have sought to provide a corrective to the essentialized notions of states
that have dominated academic debates about asylum and refugee issues. Within international political
studies, for example, the concept of multilevel governance has been employed as a way to emphasize the multifaceted
nature of the state (Nash, 2000; Bache and Flinders, 2004). Multilevel governance approaches have sought to
emphasize the increasing participation of non-state actors in the determination and implementation of policy
outcomes, the overlapping territorial networks that give rise to state processes and the new forms of coordination, steering and
networking available to the state that allows broad consensus to be built (Stubbs, 2005). Within sociology also, there have been
concerted attempts to recognize the fractured, multiscalar and peopled nature of states.

VENEZ FLOATING PIK

2NC FLOATING PIK VEN

The K solves the aff excluding the geographical securitization of the 1ac solves aff
impact scenarios
Bonfili 10 (Christian Bonfili December 21, 2010 41: 669 Security Dialogue The United States
and Venezuela: The Social Construction of Interdependent Rivalry BRW)
By early 2003, a two-month general strike would almost bring oil activity to a halt. The consequent loss of skilled manpower caused
by the dismissal of 18,000 workers would raise further questions about PDVSAs ability to sustain oil output. To the extent that these
questions emphasized the potential impact of Chvezs oil policies on the sustainability of bilateral trade, they framed US concern
about the strategic need for stability in oil imports. More importantly, they did not, however, set off a process of constructing threat
perceptions around that need even despite Chvezs threats to cut off exports from time to time. Neither did they facilitate
extraordinary responses, further affirming the notion that US concerns have been more technical than political
in nature. This epitomizes how the Chavista challenge has been assessed in the United States, as evidenced, for instance, by the
US Congress inquiry into the effects of potential reductions in Venezuelan oil exports (US Government Accountability Office, 2006),
the Central Intelligence Agencys (2003) assessment of the effects of Venezuelan strikes on future oil production, or the Energy
Information Administrations appraisal of the economic impact of Venezuelan oil-production loss on US refineries (Shore &
Hackworth, 2003). With the sole exception of the CIA Worldwide Threat Assessment of 2003 that is, written in the same year as
the strike where Chvezs radical populism was identified as potentially having a negative impact on future levels of oil output, no
analogous correlation has surfaced in official documents or statements ever since. Yet, such assessment would not presuppose the
conception of energy interdependence as an existential threat. Internalized perceptions of threat
would have entailed dramatic and extraordinary measures in virtue of which established rules of
the game would have been reassessed. The passage of a new hydrocarbons law in 2001 and the
nationalization of oil wells should not be seen as attempts at reducing dependence on the US market by
means of weakening those same rules. Rather, it should be understood as part of a strategy aimed
at enhancing the states regulative capacity and its role in oilled wealth redistribution. So, if energy
interdependence has not been securitized, has it been affected at all? Here it is argued that the economic/energy sector reveals the
effects of politicization that is, the process through which an issue is debated within specific public or
private settings and dealt with by normal politics, without the issue acquiring the status of an
existential concern. In this regard, energy interdependence has been the object of political
consideration framed by potential costs and vulnerabilities associated with dependence on
foreign oil sources, together with the lack of state control over oil wealth. The crucial distinction between this
context and one of securitizing practices lies in the extent to which such considerations have not
favored responses that might fall outside the agreed bounds of interaction between the two countries.
Economic/energy interdependence has not been incorporated into the security agenda as an urgent security concern calling for an
emergency response. As Buzan, Wver & de Wilde (1998: 23) explain, security is the move that takes politics
beyond the established rules of the game and frames the issue either as a special kind of politics
or as above politics. Extraordinary measures similar to those in the political/strategic sector revealing a
shift in the way interaction is being perceived and addressed have been absent on the economic front. This
clearly emphasizes the deeply internalized role identities and shared understandings prevailing in the economic/energy sector. The
main reason for this has been the extent to which the longstanding interaction had produced identity conceptions capable of
securing continuity in actors expectations. Drawing on Wendt (1999: 224229), this article argues that it has been
interdependence on the basis of shared role expectations that has paved the way for positive
perceptions of mutual energy dependence. The sharing of expectations has been framed by a
stock of collective knowledge (Wendt, 1999: 157165), namely, a set of norms, practices, and understandings that
have contributed to the degree of intersubjective correspondence or, in Wendtian terms, the intimacy
between both states (see Wendt, 1999: 224228). This explains, in turn, the reluctance of both Washington
and Caracas to seriously alter the foundations of an established and mutually beneficial energy
relationship. The internalization of functional identities namely, partners based on interests has been
largely framed by reciprocal reliability. The last point bears emphasis, as the social meaning of reliability has
been constructed on the basis of compatible representations by both actors. For instance, Chvez has
stated: I do not believe a revolutionary process should, in order to be such, disregard commitments like foreign debt, or similar ones
made with institutions, corporations, or states (cited in Arenas, 2008: 109). A senior official from the Bush administration noted in
October 2003 that Venezuela had been one of the USAs most reliable oil suppliers, a major commercial partner owing to PDVSAs
investment ties in the United States, and a strategic market for US oil companies (McManus, 2003). As the ultimate indication of
continuity, economic/energy interdependence has remained undamaged. This reinforces the notion that, despite
politicization around VenezuelanUS energy dependence, Chvezs measures were not aimed at
undermining agreed bounds of economic interaction with the United States. Likewise, the United States
did not put into practice extraordinary measures that would have reflected the understanding about the threat posed by its
dependence on Venezuelan oil. As was previously noted, the unbroken energy trade in the context of growing
overall commercial ties emphasizes a healthy and strong economic interdependence. The lack of a
state policy oriented toward the reduction of oil-sharing within the US energy matrix or the development of alternative sources of
energy further reinforces this assessment. Therefore, politicization rather than securitization of mutual energy dependence has
characterized the USVenezuelan economic/energy sector. This implies that neither Chvez nor Bush were either able or politically
motivated to construct interdependence as an existential issue beyond conventional considerations of costs and vulnerabilities.

Rejection of the 1ACs framing of Venezuelan country as an unstable menace in
the international sphere opens up means to actually engage and cooperate with
Venezuelas energy sector
Bonfili 10 (Christian Bonfili December 21, 2010 41: 669 Security Dialogue The United States and Venezuela: The Social
Construction of Interdependent Rivalry BRW)
Therefore, politicization rather than securitization of mutual energy dependence has characterized the USVenezuelan
economic/energy sector. This implies that neither Chvez nor Bush were either able or politically motivated to construct
interdependence as an existential issue beyond conventional considerations of costs and vulnerabilities. As already argued, the
dynamics that have driven interaction in the economic/ energy dimension have stood in stark contrast to those in the political/
strategic one. Interdependence in this particular realm needs to be assessed against the backdrop of
relative strength of constraints and costs involved in interaction, which have been framed by
threat perceptions and rivalry. In this way, both actors calculations of gain and loss relative to aspects of sovereignty,
territorial integrity, and vital security and economic interests, among other things, have informed their mutually referred and
competitive strategies. Largely asymmetric levels of dependence characterize the relationship in the
political/strategic sector, mainly as a result of a highly unbalanced distribution of capabilities between the two states.
Material conditions are important, yet socially constructed perceptions may have a crucial role
in the way and extent to which actors ascribe certain understandings to mutual dependence on
the basis of those same material conditions. Such understandings may conceivably reveal antagonism
when asymmetrical distribution of power framing mutual dependence is perceived as a threat.
For instance, threat perceptions have been constructed in relation to the political and military
ambitions of the Chvez regime, which in turn have paved the way for the construction of
meanings that clearly classify these same ambitions as a major threat to US interests in Latin
America. Specific understandings about what has been labeled Chvezs radical populism have been central to this process. In
his 2004 testimony before Congress, General James Hill (2004) of the US Southern Command warned of Chvezs radical
populism, considering this a new kind of threat to US interests in the region. He emphasized that this socio-political phenomenon
might have dangerous consequences because, by tapping into collective frustration, leaders like Chvez were able to reinforce their
radical positions by inflaming anti-U.S. sentiment. Over time, Chvezs projection of influence would become one of the most
pressing concerns among US policymakers dealing with the region. The US approach towards this perceived menace would evolve
in accordance with Chvezs increasingly radical foreign policy. The evolution of the Bush administrations approach highlights the
way in which understandings were constructed around Chvezs actions within the wider context of chain reactions related to other
perceived threats. In turn, the linkage of perceptions pointing to the various dimensions of the Chvez
menace, the internalization of these perceptions, and the policies implemented in response
paved the way for the gradual securitization of US policy toward Venezuela. Therefore, the
respective processes of constructing threat perceptions have been a core aspect of the social
process driving interaction in the political/ strategic sector. The policies both actors have implemented as a
response to the perceived threat coming from the Other are to be understood as manifestations of such processes. To the
extent that they have involved a disruption of long-established practices in the context of the two
countries relationship, those responses were exceptional in character. Chvezs new national defense
doctrine (see Garrido, 2005; Jcome, 2006; Presidencia de la Repblica, 2004) epitomizes the scope of recent changes, as it reveals
the extent to which Venezuelan authorities have internalized threat perceptions around a potential US invasion. Equally, the new
doctrine reflects a particular understanding of the nature of the would-be conflict; it draws heavily on the concept of asymmetric
warfare and encompasses several unconventional aspects (see Manwaring, 2005). For instance, Chvez has endeavored to create
what would become the largest reserve in the hemisphere, which, along with the armed forces, would be charged with defending the
country against the more powerful US enemy. Crucially, a significant proportion of this reserve would consist of civilians. To this
aim, the Chvez administration has established the Milicia Nacional Bolivariana (Bolivarian National Militia), which consists of the
Reserva Militar (Military Reserve) and the Milicia Territorial (Territorial Militia). These are to contribute to the protection of the
country in a scenario clearly identified with a guerrilla warfare-like strategy conducted by civilians rather than a professional army.
As a sign of internalization of threat perceptions, Chvez expelled US drug officials from their offices in 2004, on
the basis of alleged espionage activities conducted against Chvezs government. In 2005, Chvez suspended the bulk of
cooperation with the US Drug Enforcement Agency (DEA). In truth, this decision presupposed another
significant change in patterns of behavior in the USVenezuelan relationship, as both countries had
cooperated on a variety of counter-narcotics initiatives since the signing of a Memorandum of Understanding in 1978.14 The
breaking off of governmental cooperation with the DEA seemed to be framed by threat perceptions about the
USA carrying out potentially destabilizing actions against the Chvez regime. The DEA was perceived
as a political tool by which those actions could be pursued, and, for the first time, its presence in the country raised concerns about
its intelligence activities beyond the control of Venezuelan authorities. Chvezs decision led the White House to designate the
government of Caracas as having failed to meet its obligations under international counter-narcotics agreements for the last five
years. Consequently, most US assistance has been cut off. As an illustration of the progressive internalization of negative
perceptions, the Bush administration announced an arms embargo against Venezuela in May 2006. In a hearing before the US
Congressional Subcommittee on International Terrorism and Nonproliferation, a State Department official justified the embargo on
grounds of official concern about Chvezs overall actions against terrorism, his public statements in international forums
addressing terrorism, his ties with states sponsoring terrorism, and his conduct toward terrorist organizations (Urbancic, 2006). In
truth, this question seemed to have had a major impact on US perceptions, particularly since the embargo was framed by the
understanding of Chvezs alleged ties to guerrilla movements in Colombia. On the basis of a perceived ideological sympathy on the
part of Chvez for these groups, officials and lawmakers in Washington were concerned over the possibility that arms purchased by
Chvez might fall into their hands. Chvezs refusal to explicitly condemn the actions of the Revolutionary Armed Forces of
Colombia (FARC) further reinforced the process of constructing threat perceptions about his potentially destabilizing tactics in the
Andean region. This seemed to have informed the Bush administrations decision to create a special mission within the CIA to
supervise intelligence activities in Venezuela and Cuba. The establishment of this division was announced by John Negroponte, then
US director of national intelligence; along with those concerned with Iran and North Korea, the mission formed one of only three
specific missions within the Agency. Signaling threat perceptions, Negroponte asserted that such efforts are critical today, as
policymakers have increasingly focused on the challenges that Cuba and Venezuela pose to American foreign policy (Golinger,
2006). The embargo affected Venezuelas defense capacity considerably. Among its restrictions was a provision that denied Caracas
the parts required to maintain its fleet of 21 F-16s. Further precluding Chvezs military ambitions, the White House successfully
pressured Israel against upgrading Venezuelas US-built fleet. Likewise, Spain and Brazil were denied the requisite export licenses to
sell transport and maritime surveillance aircraft that contained US technology, and in 2006 the Swedish Group Saab ended
commercial ties after 20 years. As a result, Chvez turned to Russia as an alternative supplier, mainly because Moscow was less
willing to abide by US pressure. By virtue of a bilateral agreement signed in 2006, Venezuela became the worlds 18thlargest
recipient of military equipment in 2008 compared to its 55th place in 2002.15 Venezuelan arms imports from Moscow worth
US$1.944 billion for the period 200608 made Russia the largest supplier of conventional weapons to the Venezuelan
government.16 In turn, Venezuela was Russias fourth-largest customer in 2008, as demonstrated by the purchase of 24 Sukhoi
fighter jets, 50 transport and attack helicopters, and 100,000 assault rifles (Romero, 2007). Chvezs weapons acquisitions indeed
constituted a major increase in Venezuelan defense expenditure. Yet, the particular character and content of US
interpretation should be understood within the wider context of a process of
constructing threat perceptions about Chvezs foreign policy and its potentially destabilizing
effects in the Andean region. This is because the Bush administrations assessment failed to grasp the
contextual variables framing Chvezs arms deals. The prevailing understanding at the White House happened to
disregard certain factors such as the regions peaceful tradition of interstate relations and Venezuelas historical record as the only
South American state that has not fought a single war since its independence. Furthermore, Chiles arms acquisitions for the period
200308 have seen that country become the largest recipient of military equipment in South America, and the 12th largest
worldwide,17 without the White House expressing any particular concern. This seeming lack of concern underscores, in turn, the
crucial question of how perceptions frame policies on the basis of understandings about both physical objects and behavior. Actors
endow the material world with meaning and purpose in virtue of which social reality is constructed. The last point bears emphasis,
as this article aims to appraise how ideational constellations allow for the construction of social reality, rather than assessing
correlation between perceptions and their material foundations, such as military power. Reaching Out to Cuba and Iran Ever since
Chvez internalized the understanding about the threat posed to his regime by the Bush administration, he has invested a great deal
of political capital in challenging US grand strategy in the post-9/11 global order. In this regard, Chvez disputed the legitimacy of
Bushs war on terror, particularly since his government contested some of the UN resolutions that are considered to be the legal
foundations of the campaign against international terrorism. In truth, a set of factors that included Chvezs criticism of Bushs
militarized approach, his alleged empathy toward Colombian guerrillas, his notorious lack of condemnation of terrorism, and his
growing ties with Cuba, Syria, and Iran sharpened US concerns about a potential axis of proliferation and terrorism. Signaling the
progressive institutionalization of these concerns, the US Congress convened a hearing in 2006 to debate whether Venezuela was a
hub for terrorism. Simultaneously, Chvez embarked on a strategy aimed at fostering partnerships with states that were not under
US sway. Relations with countries such as Cuba and Iran would increasingly become paramount in this strategy. The Bush
administration soon perceived the close relationship between Chvez and Cubas Fidel Castro as a challenge to hemispheric
democratic stability. Their alliance was viewed not only as a front by which to inflate anti-Americanism throughout the region, but
also as further delaying democratic transition in the Caribbean country, which has long been a major US goal. Venezuela and Cuba
have developed a strategic alliance on the basis of both ideology and pragmatism. Bilateral cooperation has at its core a fastgrowing
commercial relationship that has evidenced itself in the amount of oil that Venezuela has been supplying Cuba on a daily basis. A
rather more pressing concern has involved potential assistance by Cuban military and security advisers regarding intelligence tactics
for curbing political opposition (Ynez, 2007). In the case of Iran, Chvez has developed an alliance comparable to that with Cuba.
Yet, Irans nuclear ambitions combined with its natural resources, military capabilities, and strategic location have rendered
Chvezs game rather more threatening in the US perspective. Ideological empathy, common political objectives, and radical foreign
policies have largely paved the way for the partnership with Iran. Cooperation between the two nations has reached unprecedented
levels in recent years (BBC News, 2006). As part of their alliance, Venezuela supported Irans right to pursue nuclear technology, as
demonstrated by its vote along with Syria and Cuba against the resolution of the International Atomic Energy Agency of 2006
that sought to report Iran to the UN Security Council over its failure to comply with sanctions. Rather more controversial has been
Chvezs apparent intent to develop nuclear energy (Harding, 2007). In a 2008 report by the Carnegie Endowment for International
Peace, Venezuela was listed among the countries exploring the nuclear option (Squassoni, 2008). This raised questions about
whether cooperation with Tehran could allow for the covert transfer of know-how with potential military use. Equally worrying has
been the prospect of Chvez seeking Iranian assistance for mining Venezuelas allegedly large reserves of uranium (Ospina Valencia,
2009; Cspedes, 2009; Reuters, 2009). Finally, the establishment of a working group between Moscow and Caracas as part of a
number of bilateral agreements (See RIA Novosti, 2008a,b; BBC News, 2008), in tandem with the fact that AtomStroyExport, a
company owned by the Russian state, was involved in the construction of the Bushehr nuclear power plant in Iran, has certainly
fostered US perceptions of threat involving Venezuelan interest in nuclear technology. In the context of mutually reinforcing
securitizing practices, the Bush administration decided to reactivate the US Fourth Fleet in 2008. Worth noting is the extraordinary
character of this decision, as it was made almost 60 years after that force had ceased operating. Certainly, this reveals a renewed
militarized approach toward the region, emphasizing the increasing internalization of specific understandings about how US
interests in the region are threatened mostly, though not exclusively, by Chvezs policies. To the extent that the US administration
could have opted instead for a non-military response, the decision also underscores the increasing institutionalization of those same
understandings. This article has endeavored to present an alternative framework of analysis for
understanding the USVenezuelan relationship under the Bush and Chvez administrations. It
has argued that economic/energy interdependence coexists with polarization and rivalry as part of the
political/strategic sector. In an effort to grasp the logic sustaining this co-existence, it has emphasized the
pertinence of combining the macro-theorizing of social constructivism with the insights into
security dynamics offered by securitization theory. The main argument developed here is that co-existence
between both security sectors has in turn presupposed two parallel yet divergent social
constellations of understandings, perceptions, and practices that are upheld through both material and
symbolic interaction. Dynamics of interaction have differed from one sector to another on account of a distinct combination
of variables. As a result, two contrasting inner logics have come to drive the USVenezuelan
relationship or, rather, the social process developed through this. Chief among its causes has been the progressive, collective
construction of antagonistic understandings by the Chvez regime vis--vis the Bush administration following the 2002 coup
attempt. The increasingly internalized understanding of US involvement happened to trigger an
intersubjective process that would progressively transform political and strategic interaction.
This is a key aspect of the social structure of the USVenezuelan relationship, in that it reveals the
extent to which both states are embedded in a situation wherein their mutually referred
identities and interests are constituted by shared expectations, understandings, and knowledge
(Wendt, 1995).

Action based off of threat will always reproduce policy failures via construction of
venezuela as the Other rejecting this dynamic leaves the material conditions of
energy independence unharmed while breaking down the status quo
internalization of role identity that frames securitizing practices
Bonfili 10 (Christian Bonfili December 21, 2010 41: 669 Security Dialogue The United States and Venezuela: The Social
Construction of Interdependent Rivalry BRW)
In this way, a possible reading of the current coexistence of interdependence and rivalry could be
that new social meanings informing competitive strategies, and developed through mutually
referred interaction, are effectively being internalized by both actors. Securitization in the political/strategic sector has
certainly involved mutually referred threat assessments, thus revealing the intersubjective character of bilateral security dynamics.
In the case of the United States, negative perceptions have involved concerns about the costs and
vulnerabilities ascribed to Chvezs radical behavior, more generally, and some of his foreign
policy decisions, in particular. As part of a constellation where threats are assessed in the light of other threats, Chvezs
actions paved the way for the construction of meanings that would clearly emphasize the growing menace ascribed to those actions
in relation to US geopolitical and security interests in Latin America. In the case of Venezuela, negative perceptions have been
constructed in relation to covert US involvement in domestic politics aimed at undermining Chvezs regime, together with a
strategy aiming to isolate Caracas within the broader regional context. The increasing US assistance to the Colombian government in
its fight against insurgency and drug-trafficking has also given rise to threat assessments involving subsequent extraordinary
measures on the part of Venezuela. In this way, the mounting US penetration in the Andean region, particularly in Colombia,
happened to be perceived and assessed against the backdrop of the USAs post-9/11 global war on terror, of which the interventions
in Afghanistan and Iraq were conceived as being just an expression. Last but not least, the prospect of a potential military
intervention by the United States has been paramount in the process of constructing perceptions of existential threat, as
demonstrated by the nature of Venezuelas response. In this way, the recent agreement between Colombia and the United States by
which US military personnel would have access to as many as seven bases in Colombian territory has further reinforced security
dynamics between the two countries. Thus, political/strategic interaction has been defined by increasingly
internalized understandings of threat, on which nationally based security discourses have been
framed. Despite being mutually referred, the collective construction of the Other as a threat has been
specific to the national context by virtue of a particular set of governmental practices, political
culture, audiences addressed, institutional settings, symbolic representations, and the
transcendence of bilateral relations within normal policymaking. In this regard, Chvezs security
discourse would involve symbolic representations of US imperialism with reference to Bushs pre-
emptive doctrine and warprone foreign policy. In the United States, negative perceptions were focused on the menace
posed by Chvezs radical populism, his alleged ties to both Colombian insurgency and US adversaries, his anti-American campaign
worldwide, and his growing military ambitions. Notwithstanding differences in style, both Chvez and Bush were successful in
persuading their audiences to accept their claims about the threat they had envisaged and, more importantly, the proposed course of
action to counter it. In addressing the threat, extraordinary measures were legitimized, as demonstrated by the reactivation of the
US Fourth Fleet and Venezuelas new military doctrine. In contrast, pre-existent practices and role identities in the
economic/energy sector have characterized a divergent social constellation vis--vis the political/ strategic one. A relatively balanced
distribution of material forces has, to some extent, contributed to preserving a longstanding and mutually beneficial partnership.
The material conditions of mutual dependence which consist of an impressive network of transactions,
investment flows, transit of resources, and expanding commerce have remained unharmed. More importantly,
continuity of positive understandings and perceptions regarding energy interdependence, which
were reinforced through longstanding practices, has crucially allowed for the maintenance of
intersubjective expectations between Caracas and Washington. From this perspective, continuity signals the
degree of internalization of a predominant role identity in the economic/energy sector namely, members of a partnership based on
interest. Social constructivism (Wendt, 1999: 251307) provides an explanation for this phenomenon. Internalization of this
particular role identity is reflected in the extent to which both actors have interacted in a social
realm framed by rules and norms that are accepted out of both self-interest and belief. Mutual
compliance is the result of internally driven interests and understandings that are reinforced through interaction. There is an
instrumental attitude in both the United States and Venezuela regarding their partnership: they
trade, do business, and cooperate in energy-related matters simply because it is mutually
beneficial. This attitude justifies their acceptance of rules and practices in terms of national
interest, which constitutes a shared meaning, albeit not the only one. In addition, these actors believe that rules and
norms sustaining their partnership are not just necessary on account of costs and benefits, but also legitimate. They thus define their
identities on the basis of shared knowledge namely, ideas about each actors rationality, practices, strategies, expectations, and
preferences. In this regard, shared knowledge is an interactionlevel phenomenon (Wendt, 1999: 160), in that it reflects the way that
all those factors are effectively internalized through interaction. As a way of illustrating this point, it is worth recalling the statements
by Chvez and the senior US official quoted earlier (McManus, 2003). Internalization of partnership based on
interests by Washington and Caracas has involved intersubjective understandings both affecting
actors calculations and expectations and framing their interaction. Thus, the reason for the lack of
securitizing dynamics in the economic/energy sector lies in the extent to which internalization
has gone beyond purely instrumental rationale. Indeed, a deeper, cultural formation involving identities
and expectations, and reproduced over time through mutually referred practices, has precluded
the change of the foundations of this ideational constellation. From the perspective of securitization theory, it
can be argued that orders of mutual accommodation between the two countries have not been upset by securitizing dynamics in the
political/ strategic sector. The one and only change here identified has been the politicization of energy
interdependence as a result of securitization. This underscores that the boundary between the
two security sectors is just for purposes of analysis. It is a means by which to clarify the intrinsic complexity of the social
world and seek plausible explanations of some of its patterns. Social reality simply does not work in this way.


FRAMEWORK

1NC FRAMEWROK

The way that debate teaches students geography is NOT neutralas a critical
geographer, you should attempt to create the debate space as an educationally free
zone, where hegemonic interpretations of maps can be rejected as students are
free to question them.
Bauder and Engel-DiMauro, 8 *Associate Professor, Graduate Program in Immigration & Settlement Studies,
Dept. of Geography **AND Associate Professor of Geography, SUNY New Paltz (Harald Bauder and Salvatore Engel-DiMauro,
2008, Critical Scholarship Practice and Education, Critical Geographies: Introduction,
http://digitalcommons.ryerson.ca/cgi/viewcontent.cgi?article=1002&context=geography)//CC
A particular issue, with which critical geographers have been concerned, is the link between
geographical scholarship and activism. While some critical geographic research has sought to
support activist struggles on the street, other research has chosen activism as a research
topic. In addition, critical geographic scholarship has been concerned with finding ways in which
the university and the very nature in which scholarship is practiced can be transformed
(Banerjee-Guha, 2002; Blomley, 2008; Castree, 2000; Fuller and Kitchin, 2004; Moss et al., 2002, 3; Peet, 1998). Today, critical geographies
embrace wide-ranging topics, themes and theories. Yet, one can and should distinguish between critical
scholarship and uncritical scholarship, which may entail critical thinking but otherwise
lacks the recognition of subjectivity, self-reflexivity and the awareness of social and political
embeddedness (Blomley, 2006). The texts selected in this book belong to this wider range of critical scholarship in the tradition of
critical theory that exists within academic geography. However, the label of critical scholarship is also contested (Katz, 1998). For example, one
author, whose work is included in this book, explicitly rejected the label critical geographer and preferred to be called a socialist geographer.
Critical scholarship, which follows critical approaches and practice, in fact, reaches far beyond the community
that identifies itself as critical geographers. At the risk of excluding critical audiences that do not embrace the label
critical geographies and including authors who do not wish to carry this label, we used the term as the title in this Collection because, in our
eyes, it represents inclusiveness and reflects the nature of the work published in the book. In addition, we chose the title for strategic reasons,
seeking to claim the notion of critical geographies for an open-access publication intended to be owned by the geographic community before
it is appropriated by the corporate publishing industry. Even in selecting a title for this book, we are confronting the problem
that critical scholarship and practice are inseparably intertwined. There is no way around it. In
geographical education, the link between scholarship and practice is particularly important.
Perhaps the ideas of critical theory translate most directly into practice in the context of education. The core of critical scholarship relates to the
ideas developed by enlightenment thinkers in the 18
th
and 19
th
Centuries (Behrens 2002). The German philosopher Immanuel Kant, for
example, showed that the categories and concepts, which make up the human world, are formed within the human subject itself. To be
critical, in this context, means to explore the origin and the limits of reason (Werlen, 1999, 198). Now,
consider Kants famous quote from 1783 (2008/1783, emphasis in the original): Enlightenment is a person's release from his self-imposed
tutelage. Tutelage is a persons inability to use of ones own reason without guidance from someone else. Self-imposed is this tutelage when its
origin does not lie in the lack of reason but in the lack of resolve and courage to use ones reason without guidance from someone else. Sapere
aude! Have courage to use your own reason! that is the motto of enlightenment.3 It is an important objective of critical
education to enable students to use their own reason and not uncritically internalize
dogma, reproduce existing norms or regurgitate conventional knowledge. Independent
thinking and reasoning, however, has to be exercised with caution and discretion, because it harbours the potential to loop back to dogma and
convention, and lead to destruction and violence, as illustrated in the catastrophes of Nazism and Stalinism in the 20
th
Century (Horkheimer and
Adorno, 2002/1944). Another important figure in critical scholarship is Karl Marx, who recognized the role of education and the educator. In
1845, Marx (1969/1845) scribbled eleven theses in response to a book by Ludwig Feuerbach, whom he critiqued for neglecting the influence of
human practice and activity on shaping the human world (first thesis). Before arriving at the famous eleventh thesis The philosophers have only
interpreted the world in various ways; the point is to change it, Marx wrote in the third thesis: The materialist doctrine [followed by Feuerbach]
concerning the changing of circumstances and upbringing forgets that circumstances are changed by people and that it is essential to educate the
educator him or herself. This doctrine must, therefore, divide society into two parts, one of which is superior to society. The coincidence of the
changing of circumstances and of human activity or self-changing can be conceived and rationally understood only as revolutionary practice.
Scholarship and education are activities that influence the course of history and shape the world. Rather than conveying objective truths about
the world from a hypothetical and non-existing vantage point located outside of the world, scholarship and education always occur in a
particular geographical context and point in history. They are practical and revolutionary activities because they shape the manner in which
people understand the world and how people act on the basis of this understanding (see also David Harvey, Chapter 11). The
importance of pedagogy was not lost on geographer and anarchist Peter Kropotkin. In Chapter 2,
which he wrote in 1885, he makes several proposals to teach geography in a manner that cultivates a
sense of commonality of all people as human beings, belonging to the same species. As part
of what would now be called pedagogical activism, Kropotkin insisted that egalitarianism should be
practiced in the classroom and that teaching should lead to the development of self-
teaching. It was not until the past few decades, however, that such classroom strategies have gained wideranging attention in the
educational systems of North America and Europe. As Kropotkin understood, pedagogical practices are directly
connected to ones understanding and practice of geography (Kearns, 2004). But only in the last three decades
have English-speaking geographers begun critiquing the objectivity and truth claims assumed in mainstream geography. They have
begun recognizing the subjective, persuasive and poetic nature of geographical inquiry (e.g.
Barnes and Gregory, 1998). For example, geographers who have drawn on the French historian Michel
Foucault have made the relationship between knowledge and power a central theme of their
scholarly activities. These geographers acknowledge that scientific knowledge, language and
our understanding of the everyday world are inseparably intertwined. Contemporary critical
geographers generally realize that their work as scientists, their roles as educators and their
participation in public life are deeply political. This realization, however, raises questions of
what to do and how to use one s abilities (and reason as Kant pointed out) to change the world. The following
chapters offer a wide range of answers to these questions. They all engage in some way with the idea that critical
scholarship, if unhappy with the circumstances it uncovers, must help create the
conditions for progressive change. The chapters of Part I of this Collection are summarized under the rubric critical
reflections. These chapters look inward, focusing on the practices of academic geographers
themselves. What might look to outsiders as exercises in navel-gazing, are to critical
geographers valuable assessments of their own practices and important discussion on how
to rectify undesirable circumstances. The chapters of Part II fall under the label space and society. These chapters
examine geographical perspectives of society, aiming to facilitate social change. Part III features chapters that engage with our relations with the
rest of nature, or the environment. A common theme throughout many of these chapters is not to take
received ideas about nature or the environment for granted. Rather, these ideas are ideological
reflections of the societies in which they occur, often intertwined with practices of social
distinction and subordination. Another theme consists in understanding environmental problems as deeply social in character,
intimately intertwined with issues of social justice. Finally, the fourth set of chapters represents work on cartography (i.e., map-making) and
Geographic Information Systems that shows the impossibility of neutrality and objectivity in representing the Earths surface.

2NC GEOGRAPHY KEY

Spatial relations do not exist independently but rather through social enactment
and construction. Critical engagement with space helps us navigate a world that is
always becoming
Kitchen and Dodge, 7 *geography professor at the National University of Ireland, **AND geography professor at U
Manchester (Rob Kitchen and Martin Dodge, 2007, Progress in Human Geography, Rethinking maps, 31:331)//CC
Ontogenetic = relating to the origin and development of individual
organisms/units/components/whatever; concerning how things become
We think it productive to take a different tack to think ontologically about cartography. For us, maps, as we illustrate in the next section and explain theoretically in
the following section, have no ontological security, they are ontogenetic in nature. Maps are of-the-
moment, brought into being through practices (embodied, social, technical), always re-made every time they
are engaged with; mapping is a process of constant re-territorialization. As such, maps are
transitory and fleeting, being contingent, relational and context-dependent. Maps are
practices they are always mappings; spatial practices enacted to solve relational problems (e.g.,
how best to create a spatial representation, how to understand a spatial distribution, how to get between A and B, and so on). From this position, Figure 1 is not unquestioningly
a map (an objective, scientific representation (Robinson) or an ideologically laden representation (Harley), or an inscription that does work in the world (Pickles)), it is rather a
set of points, lines and colours that takes form as, and is understood as, a map through mapping practices (an inscription in a constant state of re-inscription). Without
these practices a spatial representation is simply coloured ink on a page (this is not a facetious statement without
the knowledge of what constitutes a map is or how a map works how can it be otherwise?). Practices based on learned knowledge and
skills (re)make the ink into a map and this occurs every time they are engaged with - the set of
points, lines and areas are recognised as a map; they are interpreted, translated and made to do
work in the work. As such, maps are constantly in a state of becoming; constantly being remade.
At the heart of our analysis are two fundamental questions. One, how do individuals know that an arrangement of points, lines and colours constitute a map (rather than a
landscape painting or an advertising poster)? How does the idea of a map and what is understood as a map gain ontological security and gain the semblance of an immutable
mobile? Our thesis is that ontological security is maintained because the knowledge underpinning
cartography and map-use is learned and constantly reaffirmed. A map is never a map with
ontological security assumed, it is bought into the world and made to do work through practices
such as recognising, interpreting, translating, communicating, and so on. It does not re-present
the world or make the world (by shaping how we think about the world), it is a co-constitutive production between
inscription, individual and world; a production that is constantly in motion, always seeking to
appear ontologically secure. Two, how do maps become? How does the constant, co-constitutive production of a map occur? We seek to answer this
question by examining two vignettes outlining unfolding nature of mapping and by the drawing on the concepts of transduction (that understands the unfolding of everyday life
as sets of practices that seek to solve on-going relational problems) and technicity (the power of technologies to help solve those problems) (see Dodge and Kitchin, 2005). The
argument we forward is not being made to demonstrate clever word play or to partake in aimless philosophising1. In contrast, we are outlining what we believe is a
significant conceptual shift in how to think about maps and cartography (and by implication what are commonly
understood as other representational outputs and endeavours); that is a shift from ontology (how things are) to
ontogenesis (how things become) from (secure) representation to (unfolding) practice. This
is not minor argument with little theoretical or practical implications. Rather it involves adopting a radically different view of maps and cartography. In particular, we feel
that the ontological move we detail has value for five reasons. First, we think it is a productive way to think about the
world, including cartography. It acknowledges how life unfolds in multifarious, contingent and relational
ways. Second, we believe that it allows us a fresh perspective on the epistemological bases of
cartography how mapping and cartographic research is undertaken. Third, it denaturalizes and deprofessionalizes
cartography (Pickles, 2004: 17) by re-casting cartography as a broad set of spatial practices, including
gestural and performative mappings such as Aboriginal songlines, along with sketch maps,
counter-maps, and participatory mapping, moving it beyond a narrowly defined conception of
map-making (this is not to denigrate the work of professional cartographers, but to recognise that they work with a narrowly defined set of practices that are simply a
subset of all potential mappings). As such, it provides a way to think critically about the practices of cartography and not simply the end
product (the so-called map). Fourth, it provides a means to examine the effects of mapping without reducing
such analysis to theories of power, instead positioning maps as practices that have diverse
effects within multiple and shifting contexts. Fifth, it provides a theoretical space in which those
who research mapping as a practical form of applied knowledge, and those that seek to critique the map and mapping process,
can meet, something that Perkins (2003: 341) feels is unlikely to happen as things stand. Perkins (2003: 342) makes this claim because he feels addressing how maps
work involves asking different questions to those that relate to power of the medium - one set of questions being technical the other ideological. We do not think that this is
the case - both are questions concerning practice.


Policy focus is arbitrary and begs the question of whether or not policies are
effective means of changing the world, creating a self-serving and insular
academic integrity that changes nothing
Allen, 11 professor of sociology at Manchester Metropolitan University, Dialogues in Human Geography, Against dialogue: Why being critical means taking sides
rather than learning how to play the policy research game, 1(2) 223227)//CC
The problem with the authors argument is that it is based on a Gadamerian hermeneutics of
trust and the associated idea that from dialogue comes a mutual understanding that can provide the consensual basis for social progress. As such, it considers
the politics of cooperation but not the politics of scepticism and conflict in policy research. Insofar as they address the
politics of policy research, then, they merely refer to the importance of negotiating and working with the grain where policy has broadly progressive foundations while, by the
logic of their dialogical openness, assuming these progressive foundations to be omnipresent.1 The result is that they do not problematize the
idea of progressive change or raise any questions about it, such as: _ Who defines what change is and is not? _ Who
defines whether or not it is progressive? _ Who defines whether it is worth getting our hands dirty
in order to achieve such change? Whether or not we think a change is a change, a progressive change or a worthwhile change is
clearly contingent on the political stand that we take on such matters. For instance, the authors claim that their
involvement in policy research has led to progressive changes. However, other (say, Marxist) geographers are not going to be
convinced by the authors arguments that they have achieved anything significant from involvement in policy
research or that it has been worth getting their hands dirty for. So it seems that we cannot get away from the fact that a politics of scepticism and conflict is
entirely legitimate and that, as such, it must be at the centre of any argument about policy
research and not simply brushed aside. Yet it appears to be absent from the authors Gadamerian argument which implies that we should always be open to the idea of
cooperation given the possibilities (what possibilities?) presented by policy research. Contra a hermeneutics of trust that leads to a
mutual understanding upon which progress feeds, I would argue that we need to bring the
politics of power, domination and exploitation back into the centre of the argument and, in doing so, a hermeneutics of suspicion.
We need, in other words, a conflict ontology that explicitly recognizes the fault lines along which the
social world is structured (class, gender, ethnicity and so on) and how knowledge is integral to the maintenance
and legitimation of these fault lines. Knowledge does not so much provide the basis for dialogue and mutual understanding in the service of
achieving progress, then, but transmits power and domination as well as resistance to it. It follows we should eschew Woods and Gardners
notion that our task is simply to be aware of the political context of research while always being open to it and
working with and through it. This is because such a political strategy results in a primary focus on
the management of our academic integrity as we negotiate our way through the
murky business of keeping our pay masters happy. This is self-serving and does little or
nothing to combat oppression. What it does suggest is that we need to take a political stance ourselves which might just necessitate working against the
political context of the research. An example from the authors article might suffice in explaining the differences in our position here.


Pure policy relevance is badcreates agenda-driven research and epistemic
paralysis
Allen, 11 professor of sociology at Manchester Metropolitan University, Dialogues in Human Geography, Against dialogue: Why being critical means taking sides
rather than learning how to play the policy research game, 1(2) 223227)//CC
But this brings us to another problem which is our ability to respond to the need to fight when the necessity dictates. The problem that geographers face is that they
are increasingly being bound into contractual relationships with those who fund their research.
Specifically, geographers are increasingly being required to sign legally binding contracts, with state agencies, in order to be given the opportunity to produce geographical
knowledge. Moreover they are required to legally commit themselves to state agencies prior to entering
the field. This is not a problem for Marxists and the like because they are inherently suspicious
of the state and so prefer not to dirty their hands at all. However, it is a problem for geographers who see research in terms of a dialogue with publics they are engaged
with. Now a dialogue with those publics may convince us that the policy framework is more or less
adequate to their needs which may necessitate a dialogue with policy-makers, in which case there is no problem.
But such dialogues might, and often do, convince us that our only option is to fight policymakers using the critical
understandings that have been generated in dialogue with our publics. But how can we do this if
we are already legally bound to the state? The authors suggest that this is not necessarily a problem because we can always use the materials
accumulated through policy research to produce critical academic publications. But this does not convince Thomas (2010) who argues that this only leaves us
bereft of criteria for judging which of the differing forms of knowledge that we are now prepared to
put our names to constitute the correct form of knowledge. We are simply left in a state of
epistemic paralysis and confusion. Suffice it to say that the issues facing us do not simply concern the politics of the world that we engage with out
there. There is also the issue of the politics of the academic world we inhabit. We have already noted that a key aspect of the authors argument is the idea that singular views of
policy research as bad do not stand up to scrutiny because there is varietywithin it. But this overlooks an important agent in the policy research relationship: the university.
The university and university managers are forces of standardization that,when acting in combination with policy research, are
having deleterious effects on social research. This is happening in a range of ways: university managers are
becoming increasingly intolerant ofwhat they call curiosity driven research (Allen and Imrie, 2010) as they
seek to distance themselves from the image of an ivory tower academia that, according to them, is
disengaged fromwhat they ignorantly refer to as the real world that surrounds it (Allen and Marne, 2010).
They are also placing academics under ever increasing pressure to undertake policy research
that is income generating in all sorts of ways e.g. by threatening higher teaching loads to those who do not undertake policy research,
and by promoting those who dowhilewithholding promotion from those who do not (Allen and Imrie, 2010; Delanty, 2001). These institutional pressures on established
academics, when placed alongside the growth of postgraduate researcher training courses that are moulding the next generation of academics into business researchers
(Cooper, 2010), make clear that a process of standardization is occurring in which academics are being
disciplined into compliance with the brave newworld of policy research. So the politics of the university also matters
and, crucially, it is having a crushing effect on the diversity of academic research by squeezing critical
research out of the academy. This does not feature in the authors argument but, if it did, we might be compelled to draw the
rather different conclusion that it is vital for some geographers to continue to stand in a hostile
relationship to everything that surrounds them and to take sides with those that suffer as a
consequence.


The way we imagine geography shapes politics the 1AC perpetuates geopolitical
dominance
Jones, Jones, and Woods, 04 (Martin Jones* - PhD in Human Geography from the University of Manchester, Rhys
Jones; Professor of Human Geography at the University
of Wales Aberystwyth** - Professor in Human Geography @ the University of Wales Aberystwyth, Michael Woods*** - PhD in
Human Geography from Bristol University; Professor of Human Geography and Director of the Institute of Geography and Earth
Sciences @ the University of Wales Aberystwyth, 2004, AN INTRODUCTION TO POLITICAL GEOGRAPHY Space, place and
politics, http://118.97.161.124/perpus-
fkip/Perpustakaan/Geography/Geografi%20manusia/Pengantar%20Geografi%20Politik.pdf) MD
The third element that helps to constitute geopolitical dominance is the cultural messages
that shape our geographical and political understanding of the world. The field
of critical geopolitics in recent years, in particular, has attempted to draw our attention to the value-
laden messages contained in political speeches (e.g. Agnew 1998: 116) and more popular forms of culture (e.g.
Sharp 1993). Indeed, much has been made of the intimate connections between the geopolitical
imaginings forged in both formal and informal contexts. The most notable example of this interaction is the
alleged influence of the war film Rambo on US foreign policy under the leadership of Ronald Reagan (Sharp 1999: 186). The
similarities between the role of culture within geographies of imperialism and geopolitics are
striking, in this respect. In the same way as ideas of the essentialised categories of difference could be used
to justify acts of colonial exploitation within formal empires, so can the propaganda contained within
political and popular accounts of countries, cultures and religions help to create positions of
geopolitical dominance. There is no better example of this process than the western reaction to the terrorist
attacks that took place on 11 September 2001 (see Box 3.5).


Metaphorical discourse shapes reality
Prez 8 (Louis A., Ph.D. University of New Mexico, Professor of
History at University of North Carolina, "Cuba in the American
Imagination: Metaphor and the Imperial Ethos," pp. 13-15,
slim_)
But it is also true that the purpose of power informs the function of metaphor: it is intrinsic to the very
act of selecting one figurative depiction and not another. Metaphor creates new knowledge by
way of old information and thereby shapes perceptions, precisely the circumstances under
which decisions are made and actions are taken. Once situated within a moral system, with its attending codes of
cultural conduct and social convention, metaphor transforms moral-to-live-by into prescription-to-act-upon. Its very use must
be understood as a matter of intent and purpose, for to choose to mediate reality by way of one
set of cultural representations is also and at the same time necessarily to prompt a culturally determined-
and politically desired-course of conduct. It suggests the condition of possibility. The options are inscribed in the very
production of metaphor; it is meant to imply intent of purpose as a condition intrinsic to its selection. Metaphor does not
necessarily reveal similarities as much as it creates them, and thereupon suggests a range of reasonable
inferences intended to inform opinion and influence behavior. "Metaphors bring about changes in the ways in which we
perceive the world, philosopher Earl MacCormac observes, "and these conceptual changes often bring about changes in the
ways in which we act in the world." That metaphor works at all, that the premise of its representational reach provides a plausible
basis of justification for action, is itself a function of a self-confirming logic. Figurative representation , linguist Raymond
Gibbs argues persuasively, "are not linguistic distortions of literal mental thought but constitute basic schemes by which
people conceptualize their experience and the external world," which in turn "underlies the way
we think, reason, and imagine." In proposing a point of view, metaphor propounds a course of action. Indeed, the
cognitive power of the metaphor must be understood to lie in its capacity to predispose attitude as a condition to dispose conduct, or
acquiesce to the conduct of others. "We define our reality in terms of metaphors," linguists George Lakoff and Mark
Johnson suggest, "and then proceed to act on the basis of the metaphors. We draw inferences, set goals,
make commitments, and execute plans, all on the basis of how we in part structure our experience,
consciously and unconsciously, by means of metaphor." Meaning and moral converge on each other in
dialectical engagement: indeed, the interaction is intrinsic to the conceptual efficacy of memphot. To paraphrase anthropologist
Edward Sapir, as soon as the image is available and readily accessible, the concept becomes easy to
handle. It remains only to expand its implications into accessible domains-and to act upon it.
Metaphorical representation depicts a condition for which the desired response assumes the appearance of self-evident plausibility."
The use of metaphor is more than a matter of rhetorical flourishes and stylistic embellishment. Metaphors have
consequences. They are supposed to. And never more than when they are summoned in the
service of power. They serve to fix more than perspective and point of view. They also possess causal properties. Metaphorical
representations are instrumental in shaping the cognitive context in which people apprehend
the world about them, the way they arrive at an understanding of their time and place, often the
very reason they choose one course of action among others. "Metaphorical activity occurs in sites of
difference," linguist Gunther Kress has written, "whenever there is contention of an ideological kind, whenever an attempt is made
to assimilate an event into one ideological system rather than another," and adds, "The process of "naturalizing the social, of
turning that which is problematic into the obvious." To confront metaphor is not only to engage a mode of thought but also to
contend with a means of moral validation, specifically the way that systems of domination normalize the internal moral logic of
power, Metaphor has been central to the premise of empire . It has served as a source of
plausible purpose by which the colonial polity imagines the creation of empire as self-
explanatory and self-confirming, thereupon transacting the exercise of power as an obligation of
duty and a deed of disinterest. To invoke the figurative was to assemble a stock of usable imagery of power hierarchies,
usable in the sense that it propounded the rationale of domination as a matter of self-evident propriety.
Metaphor concealed the ideological content of language, a process that purported to persuade without the need
to explain and validate the propriety of power as a premise of normality, what anthropologist Christopher Tilley suggested
metaphors "utilized as vehicles of power in the sense of social domination and control." The very raison d'tre of
colonialism was inscribed within pretension to plausibility, derived from time-honored
representations of mission civilatrice: with domination depicted as deliverance, self-interest
represented as selfless purpose, and subjugation rendered as salvation.

Geographical discourse shapes reality
Amit, 10 (Dotan Amit University of British Columbia, April 7, 2010, Putting it on the Map: Imperial Gazing and
Cartographic Meaning, https://circle.ubc.ca/bitstream/id/85316/Putting_it_on_the_Map_-
_Imperial_Gazing_and_Cartographic_Meaning.pdf) MD
In light of the provocation of this new knowledge, Tuathail points out that geography was not only a noun, but also a verb.
Geography involves geo-graphing, or earth-writing. Not something possessed by the
earth, geography is in fact an active writing of the earth, the purpose of which is
to organize and discipline space according to ones own cultural visions and
material interests. 14 Having set out this particular view of geography establishes a
problematic that Tuathail terms geo-power the functioning of geographical knowledge not
as an innocent body of knowledgebut as an ensemble of technologies of power15 deployed
for the function of writing meaning onto space. Here one can identify the source of the impulse
to map newly seized/not-yet territorialized conquests. Prior to any cartographic representation, conquered land
was effectively opaque to the empires gaze and therefore extremely difficult to administer. Moreover, in a culture immersed in
Ocularcentrism, a lack of visual representation of territory made it difficult for that territorial object to acquire popular discursive
meaning within society. It became necessary to deploy more conceptually tangible forms of
geographic description that would limit this incertitude. New territories had to be grasped in
their totality in order to allow for a workable management of foreign spaces. By what attitude, then,
does this cartographic impulse translate into a visible representation that allows imperial powers to visually and conceptually
grasp the realities of foreign lands? Present throughout this process is what Jean Baudrillard has noted
be the original sense of the word production. To produce in this sense is not to materially
manufacture but to render visible and make appearTo set everything up in clear view so it can
be read, can become real and visible.16 To render visible in this context implies that the ones doing the reading
are not implicated in the form or configuration. Rather, they reveal the outcome, as if by excavation. Production in this original
sense is an ocularcentric and logocentric approach that becomes unsustainable, once viewed as an exercise of citation.

Discourse shapes reality associations with underdevelopment affect
institutionalized policies with notions of backwardness
Slater 97 (David, Ph.D from London School of Economics and
Professor Emeritus of Geography at Loughborough University,
Geopolitical imaginations across the North-South divide: issues
of difference, development and power, Political Geography Vol.
16 Issue 8, November 1997, pp. 631-653, Muse, slim_)
Following one persuasion of the post-modern turn, it might be contended that in a world of fragmentations,
pluralities and hybridization, those older, modern terms of North/ South, West/East, First World/Third
World, centres and peripheries seem intrinsically obsolete. They are the residual markers of discourses of social
change that have been shorn of any effective explanatory power; with the eclipse of the meta-narratives of
progress, modernity and socialism, the relevance of those earlier geohistorical categories of world
development has been displaced. The assertion that centre-periphery distinctions can be dissolved without any loss of
analytical scope has provoked a sharp response from a number of Latin American writers who stress the need to re-
think patterns of global inequality, rather than to neglect or deny their continuing significance. It
is certainly the case that these kinds of categorization have a well-established lineage- the North-South
distinction, for example, being popularized through the 1980 Brandt Report on Survival and
International Development. The classification of three worlds of development basically dates from the early
1950s (Pletsch, 19811, the dawn of the development era, and subsequently a plethora of terms such as developing
countries, less-developed countries, overdeveloped and under- developed countries came into regular use.X The
term underdeveloped, as a case in point, has been deployed in two quite distinct ways. First, within a conventional current, it has
been used to signify lack, absence, backwardness, and second, in a more critical, radial manner the term
underdevelopment came to be associated with Western penetration and domination; in fact a new verb was
created-to underdevelop a country. In Walter Rodneys (1972) classic text, entitled How Europe Underdeveloped4frica, it was
argued, for example, that Western interventions were the actual cause of Africas underdevelopment and poverty. In other instances,
it was proposed that underdevelopment in peripheral economies was essentially characterized by a separation between the structure
of production of a country and its structure of needs (Thomas, 1974), an idea that was initially developed by the Egyptian economist
Samir Amin in a wide-ranging series of texts-see, for example, Amin (1973, 1976). Much more recently, it has been commented that
underdevelopment is linked to a fetishization of the foreign, where, for instance, Brazil imports
plastic palm trees from Miami, and Venezuela purchases small plastic sacks of fresh water from
Scotland to accompany the importation of whisky. In the development literature, First World/Third
World, or NorthSouth distinctions, and especially the latter, still elicit a relevance which draws our
attention to issues of global inequality and power relations. According to the World Bank (1995: 53>, in its
World Development Report, divergence in incomes per capita is the dominant feature of modern economic history, the ratio of
income per capita in the richest to that in the poorest countries having increased from 11 in 1870, to 38 in 1960 and to 52 in 1985.
Moreover, as the IJNDP (1995: 14) Human Development Report reminds us, whilst over 75 percent of the worlds
people live in developing countries, they receive only 16 percent of the worlds income, or in another
formulation, the South has a per capita GNP that is 6 percent of the Norths. In this context, UNRISD (1995: 24), in its report on
social development, provides some pertinent information on the people modernization leaves behind-for example, nearly one
third of the population in developing countries lives in absolute poverty; in 1992, six million children
under five years of age died of pneumonia or diarrhoea; over the past decade, 80-90 million people were displaced by programmes
to improve infrastructure (dams, roads, ports and so forth), and since the Second World War, 23 million people have been killed in
the developing world as a result of war. In the 1980s the industrialized countries, with 26 percent of the population, accounted for 78
percent of the worlds production of goods and services, 81 percent of energy consumption and 87 percent of world armaments; one
United States resident consumed as much energy as 7 Mexicans, 55 Indians, 168 Tanzanians and 900 Nepalis (Escobar, 1995: 212).ii
These and related statistics and the realities they reflect have led some authors to connect
development and global disparities to issues of ethics and political change; one observer, for example,
states that the increasing gap between First and Third Worlds raises some of the most difficult moral questions of the modern
world (Hosle, 1992: 229). And it is quite clear that amidst the growing interest in ethics and moral questions, the place of
development, and the resonance of a North/South divide are already acquiring a thematic
weight that sharply contrasts with previous periods. I2 Nevertheless the categories of North and South are also
subject to the argument of the differences within, of the heterogeneity and explosive fragmentations within the South, or the
conflicts and dissonances within the North. Perhaps, then, after Derrida (1976), these terms are more appropriately
seen as under erasure, where there is a line running through them, cancelling them in their old form, but still allowing
them to be read, since we have no alternative more adequate categories to replace them with. On the other hand, this still
leaves open the question of how their contemporary validity might connect with the past whilst
also twisting it, breaking from it in different ways. I shall return to this problem below.

Imperialist discourse normalizes U.S. supremacy and masks the roots of power
relations
Prez 8 (Louis A., Ph.D. University of New Mexico, Professor of
History at University of North Carolina, "Cuba in the American
Imagination: Metaphor and the Imperial Ethos," slim_)
To grasp the meaning of metaphor as the principal mode of the North American engagement
with Cuba is to understand both the moral forms by which power was transacted and the
ideological context in which purpose was articulated. Simply put, metaphor served as an efficacious means with
which to advance North American interests. The narratives were typically inscribed within ordinary and commonplace formulations
of cultural models, those pat- terns of practice by which Americans themselves experienced daily life: where the premise of
power assumed the appearance of self-evident propriety, sustained as a circumstance of self-explanatory
logic. To contemplate the use of metaphor is to appreciate the capacity of figurative language to
shape the moral logic of power as a normative phenomenon. The purpose of metaphor was not explicitly
political. Rather, its principal activity was the creation of a fictive world in which the propriety of
power obtained depictive efficacy as a cultural condition. The exercise of power was represented
as a function not of political ends but of moral ones. This book examines the cultural context of political purpose,
not only as a frame of reference, but, more important, as a way to understand the dialectical process by which culture validated the
use of power as a matter of common- sense normality and commonplace propriety. The premise of power assumed the
form of disinterested purport and obtained plausibility as a matter of cultural practice. To have
deployed metaphor as a mode of cognitive engagement was to conceal the purpose of power,
specifically, to represent the defense of self-interest as a gesture of selfless intent.

Discourse shapes reality accumulation of metaphorical constructs produces
public consensus
Prez 8 (Louis A., Ph.D. University of New Mexico, Professor of
History at University of North Carolina, "Cuba in the American
Imagination: Metaphor and the Imperial Ethos," slim_)
The analysis of the relationship between language and power necessarily involves the examination of the
ways that metaphor produced knowledge and thereupon enabled power to shape a consensus
about the nature of reality. This is, as historian Michael Hunt has persuasively suggested, on one hand,
to take stock of "the need for greater sensitivity to language and especially to the meaning
embedded in key words" and, on the other, "to look beneath the explicit meanings texts convey to the
deeper structures of language and rhetoric that both impart and circumscribe meaning.
Americans embraced imperialism principally by way of an accumulated stock of metaphorical
constructs, mostly as a set of figurative depictions arranged in the form of a narrative to represent
national purpose. This was metaphor as the principal means through which a people persuaded
themselves of the beneficence of their purpose and the propriety of their conduct, that is, the wherewithal to
sustain the self-confidence and moral certainty so central to the maintenance of systems of domination. The ideological function of
metaphor was contained in its use as a source of normative truths, to represent the exercise of North
American power as a matter of moral purpose.

Language surrounding geography shapes practice
Bauder and Engel-Di Mauro, 08 (Harald Bauder* - Professor of Geography at the University of Guelph in
Canada, Salvatore Engel-Di Mauro** -Professor of Geography at the State University of New York at New Paltz, 2008, Critical
Geographies: A Collection of Readings, http://www.praxis-epress.org/CGR/CG_Whole.pdf) MD
As Kropotkin understood, pedagogical practices are directly connected to ones
understanding and practice of geography (Kearns, 2004). But only in the last three decades have
English-speaking geographers begun critiquing the objectivity and truth claims assumed in
mainstream geography. They have begun recognizing the subjective, persuasive and poetic
nature of geographical inquiry (e.g. Barnes and Gregory, 1998). For example, geographers who have drawn
on the French historian Michel Foucault have made the relationship between knowledge and power a
central theme of their scholarly activities. These geographers acknowledge that scientific knowledge, language
and our understanding of the everyday world are inseparably intertwined. Contemporary critical
geographers generally realize that their work as scientists, their roles as educators and their participation in public life are deeply
political. This realization, however, raises questions of what to do and how to use ones abilities (and reason as Kant pointed out)
to change the world.

2NC GEO KEY CUBA

Addressing the subjectivity of our representations is uniquely key to US-Cuban
relations
Slater 97 (David, Ph.D from London School of Economics and
Professor Emeritus of Geography at Loughborough University,
"Geopolitical imaginations across the North-South divide: issues
of difference, development and power," Political Geography Vol.
16 Issue 8, November 1997, pp. 631-653, Muse, slim_)
Inside/outside and terrains of knowledge If the power over other societies manifests itself in the
coalescence of discursive persuasion and external inducements, the constitutive outside will always be
formed around the modalities of power within those other societies which experience penetrations,
incursions, implantations, demarcations, re-orderings and mellifluous adjustments. Outside/inside, external/internal,
foreign/domestic are continuously inter- twined in a series of combinations that can never be completely captured
in one or the other. Let us take a few examples. I. External domination or support for authoritarian, oligarchic regimes
erodes domestic liberty;5 imagined and actual enemies facilitate national security apparatuses, surveil-
lance, and the chilling of debate and deliberation. The Truman-McCarthy period invoked the red
menace to stifle critical thought in every area of culture inside the United States, and the Reagan-Bush era
conjured up the evil empire and terrorism to sanction the secret government of Iran-
Contragate. 2. In another historical context, mentioned above, United States-Cuba relations at the turn of the last
century cannot be properly situated without appreciating the heterogeneity of the internal for Cuba itself,
with the role and impact of diverse political currents within that society. At the end of the 189Os, there were those tendencies inside
Cuba (especially from the dominant colonial sectors) that favoured a close relation with the United States in the form of a
protectorate and semi-sovereignty, whilst other less powerful groupings supported outright annexation and the incorporation of
Cuba into the United States, in direct opposition to a predominant percentage of Afro-Cubans who had fought in the War for
Independence against Spain, and were strongly in favour of genuine national independence from United States power.36 Hence,
for an effective understanding of US-Cuba relations we need to take into account
the way in which a variety of political tendencies in Cuba affected the evolution of
US attitudes and policy , whilst equally, bearing in mind the existence and influence of anti-imperialist groups inside
the United States (Healy, 1970: 213-231). The complex interweaving of inside and outside does not displace the
centrality of certain kinds of power, but it does remind us of the intricate contours and effects of geopolitical
interventions. 3. With reference to my previous commentary on modernization theory, it is clear that the decline in its
influence was linked to the critical impact of dependency perspectives. This, in its turn, captured
the importance of the interaction between an externally- oriented First World theory of social and
political change and a more internally-directed set of ideas and concepts that challenged such a
diffusion and theorized back. Furthermore, this point can be extended to the broader terrain of imperial encounters, where under
the influence of post-colonial studies, it has become more important to reflect on the mutually conditioning nature of these two-way
flows (Bhabha, 1994). It is no longer regarded as acceptable to dwell primarily on the mechanisms of power
employed by the imperial centre, and implicitly treat the dependent periphery as a passive
receiver.

more ev
Bauder and Engel-Di Mauro, 08 (Harald Bauder* - Professor of Geography at the University of Guelph in
Canada, Salvatore Engel-Di Mauro** -Professor of Geography at the State University of New York at New Paltz, 2008, Critical
Geographies: A Collection of Readings, http://www.praxis-epress.org/CGR/CG_Whole.pdf) MD
As Kropotkin understood, pedagogical practices are directly connected to ones
understanding and practice of geography (Kearns, 2004). But only in the last three decades have
English-speaking geographers begun critiquing the objectivity and truth claims assumed in
mainstream geography. They have begun recognizing the subjective, persuasive and poetic
nature of geographical inquiry (e.g. Barnes and Gregory, 1998). For example, geographers who have drawn
on the French historian Michel Foucault have made the relationship between knowledge and power a
central theme of their scholarly activities. These geographers acknowledge that scientific knowledge, language
and our understanding of the everyday world are inseparably intertwined. Contemporary critical
geographers generally realize that their work as scientists, their roles as educators and their participation in public life are deeply
political. This realization, however, raises questions of what to do and how to use ones abilities (and reason as Kant pointed out)
to change the world.

2NC AT: POLICYMAKING

law and space are mutually constitutive understanding the social relationship of
space is a prerequisite to policymaking
Blandy and Sibley, 10 professors at the School of Law of the University of Leeds (Sarah Blandy and David Sibley, 2010,
Social and legal studies, Laws boundaries and the production of space, 19(3))//CC
In this special issue we want to de-territorialize issues of law and space, through broadening out the
task to other disciplines and to consideration of non-material spaces, challenging the orthodoxy that the
very possibility of law is contained within the spaces defined by the nexus of territoriality and sovereignty (as summarized by
Delaney, 2001: 253). It is certainly true that the laws main concern could be said to be boundaries of one
kind or another, as implied by the derivation of nomos (law or custom in ancient Greek) from nemo, meaning to separate or
divide: the nomos opens with a drawing of a line in the soil. This very act initiates a specific concept of law, which derives order from
the notion of space (Vismann, 1997: 46, cited in Fitzpatrick, 2005: 8). The law, as a uniquely powerful discourse,
brings into existence that which it utters (Bourdieu, 1991: 42), then conjures up meanings for the
spaces on either side of the boundary. These meanings are not merely externally imposed and legitimated through
enforcement by law, but are continuously re-created and maintained by social interaction. The mutual
constitutivity of the spatial and the social was established by critical geographers such as Soja (1989) who explored how power
relations are mediated through spatial practice. And it would not be excessive to say that [the law] creates
the social world, but only if we remember that it is this world that first creates the law (Bourdieu,
1987: 839). The legal consciousness literature has elaborated the ways in which law and the social are mutually constituting, so that
law should be seen as a set of conceptual categories and schema that provide parts of the
language and concepts we use for both constructing and interpreting social interaction (Silbey,
2001: 273). Verbal representations of encoded, combined spatial and legal meanings have been
referred to by Blomley (2003) as splices; for example, the term refugee is a splice which implies both
a legal status and a spatial dislocation. A higher level, complementary approach is offered by another neologism, the
nomosphere, a conceptualization which allows exploration of the social mechanisms involved in the circulation of legal meanings
through spatial forms (Delaney, 2004: 852). Thus law and space actively shape and constitute society,
while being themselves continuously socially produced. We would stress that an essential part of this
process is the internalization, at individual and collective scales, of what Shamir (2001) has termed the conceptual
grid which law imposes on space. Most of the articles in this issue are concerned with the interplay between
psychological and socio-spatial bound-aries, from the imagined communities of nation states (Anderson, 1983/1991) to
conceptualizations of the bounded self (Blomley, this issue). Boundaries which have meanings in law and space
also affect the inner lives of individuals, which in turn produce and reproduce those meanings. It
is interesting, then, to consider the relevance of theories of individual behaviour developed in the
context of intimate spaces to questions of group relations at regional, national and international
scales. In previous work, van Houtum has interrogated the fear/desire spectrum of individuals internalized boundaries, in
conjunction with developing his earlier argument that the desire to control borders has turned Europe into a larger scale, well-
guarded gated community (van Houtum and Pijpers, 2005) . In his article in this issue, van Houtums essential point is that
national borders are concerned with laws essential functions of classification, categorization
and filtering - disciplinary practices which, as Michel Foucault would argue, are internalized by those
whose desire is to gain entry, to be on the other side. To some extent, although van Houtums discussion allows for
resisters to this internalization, his view contrasts with Taylors observations on the US-Mexican border which
emphasize the possibilities for subversion and entrepreneurship that the border regime
presents (Taylor, this issue). Borders are zones of both control and agency. Despite their very different
approaches, these two articles are complementary: Van Houtums philosophical, almost abstract, discussion nonetheless
demands application to the plight of contemporary asylum seekers and excluded would- be migrants, and Taylors article is firmly
based in ethnographic observations of everyday border practices - which raise new questions about laws powers to define. Both
articles link the law and boundaries on the ground with psychic boundaries.


2NC EPISTEMOLOGY

The affirmatives understanding of geography is not neutral US policymakers
construct spaces as demonized in order to exclude them from the capitalist order
the result is a legitimization of hegemonic violence
Martin 03 (Martin, Gregory Date: 02/01/2003 Article: Spaces of
Capital: Towards a Critical Geography Journal: Teachers College
record ISSN: 0161-4681 Volume: 105 Issue: 1 Page: 134
http://www.tcrecord.org.proxy.lib.umich.edu/library/Content.a
sp?ContentId=10962 BRW)
For Harvey, challenging such social inequality and uneven development requires recognizing how capitalism
is dependent upon certain kinds of geographical understandings in the public domain. Implicit in
this idea is the notion that such geographical knowledges do not emerge autonomously but are
rather deliberately constructed and maintained by the capitalist class so that it can pursue its
own narrow interests, albeit in the name of universal goodness. For example, in Cartographic Identities, Harvey eerily
reminds us of how access to certain forms of geographical knowledge in the public domain has been
responsible for constructing various demonized spaces in the global economy such as Cuba,
China, Libya, Iran, Iraq and the Evil Empire, of the former Soviet Union. The tragedy here is that A recent poll in the US showed
that the more knowledgeable people were about conditions and circumstances of life in a given country, the less likely they were to
support US government military interventions or economic sanctions (p. 211). Thus, the exercise of military power
requires keeping the American public in a chronic state of geographic ignorance about the role
of the United States as a bearer of a global ethic, when it is really imposing a rational spatial
order that opens up the possibility for capital accumulation. In the second set of essays, Harveys central task
is the reconstruction of Marxist theory, in light of contemporary conditions and historical-geographical experience. Indeed, for
Harvey, it is clear that capitalism has failed to deliver its promise of equality and freedom for all, when
material conditions are just as inhumane today as when Karl Marx wrote the Communist
Manifesto in 1847. Witness not only spiraling social inequality in the home citadels of imperialism
such as the United States but also the trade in human misery and despair in the neo-colonies,
where the majority of the worlds gendered and raced proletariat remain physically or
economically shackled in factories (producing commodities such as Nike shoes and Gap clothing) and in the fields,
almost 150 years since slavery was officially abolished. In fact, despite the far-reaching claims of globalization theorists such as
Anthony Giddens (1999), the world imperialist economy is not becoming homogenous, far from it. This is hardly
surprisingas Marx points out in Capital, Accumulation of wealth at one pole isthe accumulation of
misery, agony of toil, slavery, ignorance, brutality, mental degradation, at the opposite pole, i.e.
on the side of the class the produces its own product in the form of capital (p. 645). It is little wonder that for Harvey the working
out of this contradiction constitutes one of the major forms of motion that will inevitably determine human history and geography.
Deriving his analysis from the labor process, Harvey argues that it is precisely within this context that it is possible to point to the
limits and reversibility of changes in the world economy. For example, in The Geography of Class Power, he points
out that as capital perpetually turns to a spatial fix to resolve its internal contradictions, it not only
expands productive relations on a progressively larger scale but also the bases for socialist revolution. Thus, even as the
bourgeoisie are driven to reorganize geographical space over time to create economic and social
surpluses, uneven development that ultimately threatens to wreak the whole system. Although the
Communist Manifesto implies that capitalist development produces a homogenous working class, the communist movement must
begin to recognize the differentiating power of capital, which has often absorbed class struggle by exacerbating place-bound loyalties
including all manner of gender, religious, ethnic and cultural divisions. Thus, just as uneven development itself is
shaped by particular geographical re-orderings, spatial strategies and geo-politics, class struggle
also unfolds differentially across this varied landscape.

The state and academic research are interlocked empirical research and
qualified authors become a tool to promote imperialist objectives in Latin
America
Sundberg, 03 prof. at the Department of Geography, University of British Columbia (Juanita Sundberg, 9 September 2003,
Looking for the critical geographer, or why bodies and geographies matter to the emergence of critical geographies of Latin
America Science Direct, pg. 17-28)//ahayes
Since the Monroe Doctrine in 1823 and subsequent declarations of manifest destiny, the United
S tates has sought to assert hegemonic power in the Western Hemisphere. Although Latin Americans from
diverse countries have consistently called the US an imperialist force (Cardoso and Faletto, 1979; Dorfman, 1975), diplomats and
scholars within the US have debated the accuracy or appropriateness of this term. For many, US imperialism was an aberration, or
a eeting episode in the late 19th century (Kaplan and Pease, 1993, p. 13). Others submit to the accusation, but suggest that US
policy is driven solely by economic considerations. While some US academics and policy makers
continue to engage in an ongoing pattern of denial about US imperialism (Kaplan and Pease, 1993, p. 11),
others seek to re-conceptualize how we research and analyze imperial encounters on the ground
(Joseph et al., 1998). Historian Mark Bergers study, Under Northern Eyesthe North American equivalent to Saids Orientalism
is an explicit attempt to make the relationship between power and knowledge central to the examination
of the North American study of Latin America (1995, p. 1). Although Latin American studies has been cloaked in
assertions of objectivity and [articulates] a commitment to scientic and rational discourses, Bergers painstakingly detailed
analysis situates scholars within the specic socio-historical contexts that condition their views of Latin America (1995, p. 19).
Arguably, the most important factor shaping the context in which scholars operate is the US governments shifting interests in
individual Latin American countries. As Berger notes, the boundaries between academia and the state have been blurred in Latin
American studies. Not only have academics moved back and forth between academia and the various
agencies of the government, but the state also has attempted to shape the kinds of
research undertaken. For instance, the 1958 Defense Education Act created funding for area studies programs, in
recognition of the strategic value of cultural knowledge about Latin America and other regions of the world (Morris-Suzuki, 2000,
p. 14). 10 Ultimately, scholars suggest, such policies explicitly recognized that the development of area studies programs
could contribute to the successful exercise of US world power (Morris-Suzuki, 2000, p. 14). 11
In light of these close ties, Berger (1995, p. 2), argues that ...North American historical and social science
professions facilitate the creation and maintenance of the national and international
organizations, institutions, inter-state relations and politico-economic structures that sustain
and extend US hegemony in Latin America and around the world. Furthermore, history
and social science disciplines derive their power and authority from their linkages to these
organizations, institutions and political structures. In using Bergers analysis to highlight the
connections between geo-politics and the production of knowledge , my goal is not to
suggest that all United States researchers working in Latin America have the interests of the state at heart; nor do I wish to argue
that place determines outlook. Rather, I want to point out that Latin American studies in the US and
consequently, geographies of Latin America are embedded in and emerge from specic contexts
characterized by asymmetrical power relations. Such a genesis has implications for the kinds of interpretative
categories and research questions underwriting US research in the region. One such consequence is that Latin American
studies consistently positions the West or the US as the implicit referent, i.e. the yardstick by which
to encode and represent Latin America (after Mohanty, 1991, p. 55). As Berger notes, between the mid-19th century
and World War I, Latin American studies reected and reproduced Anglo-Saxonist assumptions about North
American civilization as the highest form of civilization in history (Berger, 1995, p. 30; see also Schoultz,
1998). Scholars looked to biological di erences between North American and Latin American peoples to explain socio-political
phenomena in Latin American countries. After World War II, the majority of scholarship on Latin America was
structured by modernization theory, which cast these ideas in a new light. Notions of racial di erences
were replaced with cultural conceptions of development, progress, although an idealized vision
of the West remained the yardstick by which Latin American nations and people were measured.
Binaries such as developed/underdeveloped emerged to replace earlier versions like advanced/
backward. The discourses have shifted, but the underlying presumption of the superior
white North American self as referent of analysis remain the same (see also Schoultz,
1998). 12 Bergers study illustrates the extent to which research on Latin America coming out of the United States presumes a white,
elite, (masculine) United Statesian referent of analysis. This is also to say that United Statesian representations of
Latin America may say more about US interests and identities than about Latin American
people and society. Clearly , geography, geographical location, and geo-political
context matter greatly in the production of knowledge about Latin America. Although
a Berger-like self-critique is yet to be elaborated, a number of geographers have begun to analyze the
epistemological assumption underlying Anglo-American geographical research on Latin
America.

Traditional objective epistemological standards are flawed in the context of Latin
American geography notions of neutrality mask an underlying oppressive
agenda
Sundberg, 03 prof. at the Department of Geography, University of British Columbia (Juanita Sundberg, 9
September 2003, Looking for the critical geographer, or why bodies and geographies matter to the emergence of
critical geographies of Latin America Science Direct, pg. 17-28)//ah
In current Anglo-American geographical accounts of Latin America, it is common practice to
introduce empirical studies with a description of the location in which the research was conducted, the specic social
groups consulted, time spent, and methods employed. In however, geographers seldom situate themselves
socially or geographically. Consequently, more often than not, it is unclear how the researchers social identity (in
terms of gender, race, 1 and class), institutional aliations, and geographic location play a part in shaping the research
undertaken in Latin America; nor is it clear how particular geo-political congurations create conditions that enable and
support eldwork in a particular country. In short, how these factors shaped the analytical categories, methods, research questions,
as well as the collection and interpretation of data is dicult to comprehend. As a white woman conducting research in northern
Guatemala, I was (made) acutely aware of the ways in which my gender, race, and biography as a privileged United States
citizen shaped all levels of my research from the questions I asked, to my interpretations. And yet, I nd it dicult to
carve out space for this kind of discussion within current writing conventions, particularly in political ecology. This article emerges
out of my frustrated attempts to situate myself as a critical, feminist geographer/political ecologist working in Latin America and to
explain why this is important and indeed necessary. My frustration leads me to believe that the tensions between Latin Americanist
geographical traditions and critical geographical perspectives, including but not isolated to feminism, stem from di ering
epistemological stances towards the production of In focusing on racial identity, my goal is not to substantiate the existence of
biologically di erentiated races but rather to highlight the persistence of social categories and material practices that racialize
particular groups as superior or inferior. See Goldberg (1993). knowledge (Rose, 1993; Barnes and Gregory, 1997). Latin
Americanist geography tends to follow mainstream geography in its pursuit of objective or value-
free knowledge, which is predicated upon the separation of the observer from his or her body,
social position, and geographical location as well as the separation between intellectual work
and political practice (Barnes and Gregory, 1997, p. 19). In contrast, critical geography involves a radical
critique of conventional notions of objectivity, in the sense of selfconsciously wedding
political goals with academic interventions , stressing the connections between power
and knowledge, and advocating self-reexivity (Painter, 2000, p. 126, 127). In this paper, I argue that
prevailing notions of objectivity have (at least) two e ects upon geographies of Latin America: the disappearance of the geographer
as a corporeal being and the obliteration of geography as a constitutive factor in the social and institutional life of the geographer. In
light of these e ects, the task of making Latin Americanist geography compatible with critical approaches begins with an analysis of
how and why the bodies and geographies of geographers themselves matter (to borrow from Butler, 1993; Massey, 1984). To
focus on the geographer as a producer of knowledge is not to advocate the kind of
navel gazing so abhorrent to many scholars . Rather, it is an e ort to call attention to
and critically assess how the geographers embodied social position and geographic location inform the production of
knowledge about and representations of Latin American people and nature. To illustrate how and why
bodies and geographies matter, I draw from two parallel yet distinct bodies of literaturefeminism and post-colonialism
that reveal and confront the connections between power relations, the production of knowledge,
and the (re)production of inequality. In the rst section of the paper, I engage with feminist theory to analyze why
bodies matter. From a feminist perspective, conventional notions of objectivity demand that researchers perform the god trick by
stepping out of their bodies to assume the gaze from nowhere (Haraway, 1991). The god-trick is
problematic, in that claims to objectivity have been used historically by white, Western, males to mask very
particular interests and perspectives. Examples from my own experience, as well as the experiences of other researchers,
illustrate how consideration of the body calls for situated knowledges that can account for their partial position/vision (Berg, 2001;
Haraway, 1991; Nast, 1994; Mullings, 1999). In the third section of the paper, I engage with postcolonial theory to examine why
geography and geographical location matter. To this end, I draw upon Saids (1979) argument that European representations
of the Orient do not reect the Orient as it really is, but rather produce Orientals as inferior
Others through discourses inscribed with racial and cultural hierarchies that
embodied and (re)produced colonial power. 2 For Said, identifying the geographical and
political context from which representations of others emerge is key to understanding how inequality
is (re)produced. In light of the continuing legacies of European colonialism and contemporary practices of
United States imperialism in the Western Hemisphere, is not a parallel critique essential to the construction
of critical geographies of Latin America? In the nal section, I suggest that conventional notions of objectivity are
animated by a presentation of self that runs counter to the aims of critical geography. I then explore the notion of
situated knowledge as a tactic that writes bodies and geographies into academic texts. Ultimately, situating knowledge represents a
political intervention and contribution to the broader goals of emancipatory politics shared by critical human geographers. Before I
begin, a note about my terminology. Throughout this paper, I refer to geographical research on Latin America produced in an Anglo-
American, but more specically United States context. 3 At times I use the term Latin Americanist geography, which refers to
approaches emphasizing a regional perspective (see essays in Knapp, 2002). Not all geographers producing knowledge about Latin
America choose to refer to themselves as Latin Americanists, however (see Sundberg, 2003). Indeed, many prefer to identify
themselves with their theoretical approach, as in urban geography. These di ering designations point to debates within the sub-
discipline about the production of knowledge. The essays in this special issue of Geoforum seek to infuse geographies of Latin
America with critical theoretical perspectives, which, up to this point, have played a minor role within the sub-discipline. 2.
Challenging the god trick or why bodies matter Conventional Anglo-American social science, which
predominates in Latin Americanist geography, denes objectivity by the value neutrality of the
researcher in other words, that the researchers choice of topic and methodology is not inuenced by her/his values,
experiences or material conditions (Staeheli and Lawson, 1995, p. 328). Such notions of objectivity are predicated upon
the assumption that the researchers mind is sep- arate (separable) from his or her body, social situation, and
geographical location, and moreover, that the researcher is separate from, and una ected by the objects of research (Harding,
1986). Consequently, the researchers authority to speak/write about others is predicated upon
discourses of concealment and practices of separation, which are necessary to preserve the aura of objectivity
(Rose, 1993). This conceptualization of the researcher as disembodied, autonomous, and neutral is
particularly problematic for feminists. First, feminist research into Western scientic and philosophical traditions
illustrates that conventional notions of objectivity are predicated upon and reproduce gendered and racialized binaries such as
mind/body, masculine/feminine, reason/emotion, and civilized/uncivilized (Alco , 1996). Such binary systems are
asymmetrical in that historically, only white, Western, bourgeois, heterosexist males were seen as capable of
transcending their bodies and achieving rationality; men of color and all women were not (Haraway, 1989, 1997; Goldberg, 1993).
For this reason, feminists argue, prevailing notions of objectivity have presumed a masculine subject position even as they deny the
researchers corporeality and social identity. Thus white, Western, heterosexist, elite males have used claims to objectivity in ways
that mask and protect very particular interests and perspectives (Rose, 1993). In short, masculinist objectivity, also
called the god trick or the gaze from nowhere, universalizes as truth knowledge that
is partial, selective, and interested (Haraway, 1991). Secondly, research by scholars such as Reiter (1975),
Haraway (1989), Hanson and Pratt (1995), Rose (1993), and Rocheleau et al. (1995)), which takes women into account in the
analysis of social and spatial phenomena, contradicts or confounds previous research, calling into question the
presumed universality of socio-spatial categories and ultimately conventional notions of
objectivity. Such research has led to an increasing awareness of the myriad ways in which the researchers corporeality, social
position, and geographical location profoundly shape analytical categories, research questions, as well as the collection and
interpretation of data.

Epistemology outweighs it shapes practice
Bauder and Engel-Di Mauro, 08 (Harald Bauder* - Professor of Geography at the University of Guelph in
Canada, Salvatore Engel-Di Mauro** -Professor of Geography at the State University of New York at New Paltz, 2008, Critical
Geographies: A Collection of Readings, http://www.praxis-epress.org/CGR/CG_Whole.pdf) MD
Critical geography is both an approach to scholarship and a practice of scholarship. The term
critical refers to a tradition of critical theory. An often cited representative of this tradition is the so-called Frankfurt School. This
school consisted of a network of researchers affiliated with the Institute for Social Research in Frankfurt, Germany, which
operated from 1923 to 1933, moved to New York during the Nazi regime, but reopened in Frankfurt in 1950. Although the label
Frankfurt School is problematic and inexact (Behrens, 2002), it does permit associating some basic ideas with the notion of
critical. According to Herbert Marcuse (1964: x), a prominent member of this school: To investigate the roots of
[social] developments and examine their historical alternatives is part of the aim of a critical theory, a
theory which analyzes society in the light of its used and unused or abused capacities for improving
the human condition. Achieving this aim, however, is complicated by the inability of researchers
to assume objective viewpoints and completely dissociate themselves from the social world
and the technologies they use. In fact, scholars and scientists as well as the institutions in which they
operate are firmly embedded in the social and political world. If scholarship is
uncritical towards its social embeddedness, it is prone to reproduce existing social
order and inadvertently promote political, social and cultural interests. Critical scholarship therefore
does not deny these interests but rather incorporates them into its approach. Critical scholarship addresses the inevitable
dilemma of being a social and political activity by focusing on the tension between the existing social and material world and the
possibility for changing this world. With this focus, critical scholarship realizes its role in society not to blindly
reproduce existing social order, but to create the conditions in which progressive change can occur. This
focus also means that the separation between scholarship and practice cannot be
maintained. Rather, critical scholarship embraces the connection to critical practice.

More evidence scholarship shapes policy
Bauder and Engel-Di Mauro, 08 (Harald Bauder* - Professor of Geography at the University of Guelph in
Canada, Salvatore Engel-Di Mauro** -Professor of Geography at the State University of New York at New Paltz, 2008, Critical
Geographies: A Collection of Readings, http://www.praxis-epress.org/CGR/CG_Whole.pdf) MD
In geographical education, the link between scholarship and practice is particularly important.
Perhaps the ideas of critical theory translate most directly into practice in the context of
education. The core of critical scholarship relates to the ideas developed by enlightenment thinkers in the 18th and 19th
Centuries (Behrens 2002). The German philosopher Immanuel Kant, for example, showed that the categories and
concepts, which make up the human world, are formed within the human subject itself. To be
critical, in this context, means to explore the origin and the limits of reason (Werlen, 1999, 198).
Now, consider Kants famous quote from 1783 (2008/1783, emphasis in the original): Enlightenment is a person's release from
his self-imposed tutelage. Tutelage is a persons inability to use of ones own reason without guidance from someone else. Self-
imposed is this tutelage when its origin does not lie in the lack of reason but in the lack of resolve and courage to use ones reason
without guidance from someone else. Sapere aude! Have courage to use your own reason! that is the motto of enlightenment.3
It is an important objective of critical education to enable students to use their
own reason and not uncritically internalize dogma, reproduce existing norms or
regurgitate conventional knowledge. Independent thinking and reasoning, however, has to be exercised
with caution and discretion, because it harbours the potential to loop back to dogma and convention, and lead to destruction and
violence, as illustrated in the catastrophes of Nazism and Stalinism in the 20th Century (Horkheimer and Adorno, 2002/1944).
Another important figure in critical scholarship is Karl Marx, who recognized the role of education and the educator. In 1845, Marx
(1969/1845) scribbled eleven theses in response to a book by Ludwig Feuerbach, whom he critiqued for neglecting the influence
of human practice and activity on shaping the human world (first thesis). Before arriving at the famous eleventh thesis The
philosophers have only interpreted the world in various ways; the point is to change it, Marx wrote in the third thesis: The
materialist doctrine [followed by Feuerbach] concerning the changing of circumstances and upbringing forgets that circumstances
are changed by people and that it is essential to educate the educator him or herself. This doctrine must, therefore, divide society
into two parts, one of which is superior to society. The coincidence of the changing of circumstances and of human activity or
self-changing can be conceived and rationally understood only as revolutionary practice. Scholarship and
education are activities that influence the course of history and shape the world. Rather than
conveying objective truths about the world from a hypothetical and non-existing vantage point located outside of the world,
scholarship and education always occur in a particular geographical context and point in
history. They are practical and revolutionary activities because they shape the manner in
which people understand the world and how people act on the basis of this understanding (see
also David Harvey, Chapter 11).



ANSWERS


at: empirics

Empirics go neg geography creates the norms for subjugation of the global south
Sparke, 8 Department of Geography and the Jackson School of International (Matthew Sparke, 10 June 2008, Everywhere
But Always Somewhere: Critical Geographies of the Global South, http://faculty.washington.edu/sparke/Everywhere.pdf)//ah
The Global South is everywhere, but it is also always somewhere, and that somewhere, located at the intersection of entangled
political geographies of dispossession and repossession, has to be mapped with persistent geographical responsibility. Following
Gayatr Chakravorty Spivaks tracings of the shadow of a geographical pattern in deconstruction (Spivak, 1976: lxxxii), I have
elsewhere outlined such geographical responsibility in terms of a call to track critically and persistently the openended graphing
ofthe geo (Sparke, 2005). But what doesit mean to track the geo-graphy of the Global South in this persistently critical way? Firstly it
means critiquing the maps that have routinely represented the Heterogeneous spaces of the Global
South in the interests of colonial and neocolonial control. These maps of control include all the instrumental
cartographies, moral topographies, and imaginative geographies that have provided the practical guides and
promotional props for colonial practice from the times of Terra Nullius, the Orient and the Heart of
Darkness to such contemporary conceits as the Axis of Evil, the Clash of Civilizations, and,
that gleeful globalist gloss on contemporary capitalism, the Level Playing Field. All these
geographical visions have in different ways overwritten and obscured the huge heterogeneity of the Global
South, replacing it with pernicious NewWorld declarations, Us-Them oppositions and West is
Best assumptions. The arrogance ofsuch over-mapping issimilar in its overreach to the Map ofthe Empirein the famousstory
by Jorge Luis Borges, a story of imperial cartographers who aspired to map everything in such detail that their map actually grew to
the size ofthe Empire and covered up the land (Borges, 1998). But while this story of over-mapping was used by Borgesto ridicule the
futility of representational totalization, and while he depicts the Map of the Empire being therefore abandoned as uselessin the
Deserts of the West, the maps of control covering up the Global South have by contrast actually been very useful to their
cartographers. They have served practically to remake the world anew, justifying imperial violence all
over the planet and underpinning political-economic practices that have ranged from
the trans-Atlantic trading of Africans, cotton and sugar to the contemporary Highly Indebted Poor Country
surveillance protocols deployed by the World Bank and IMF to limit the reach of debt relief. In all of these diverse contexts the
signature gesture of the maps of dominance has been simple: divide and dispossess. Whether legitimating
inhuman violence by liberal sin colonial India (Metha, 1999), or inhuman violence by neoliberals in neocolonial
Iraq (Gregory, 2004;Roberts et al, 2003), maps that map the Global South as a space of exception outside
the bounds of humanity and human rights have remained key to the ideological legitimation and
military organization of imperial rule. The maps might be richly detailed and realist in the old imperial and Cold War
area studies traditions(that often mixed ethnographic care with colonial interest), orthey might be highly fictional and idealist in the
neo-imperial Iraq War tradition (that has mixed militaristic spin with crass commercial interest). But, in whatever way the
over-mappings have overreached, theirterritorializing imperatives have always worked in the
interests of dispossession.

at: globalization solves

Globalization is a myth geographical divisions remain the basis for neoliberal
exploitation
Jones, Jones, and Woods, 04 (Martin Jones* - PhD in Human Geography from the University of Manchester, Rhys
Jones; Professor of Human Geography at the University
of Wales Aberystwyth** - Professor in Human Geography @ the University of Wales Aberystwyth, Michael Woods*** - PhD in
Human Geography from Bristol University; Professor of Human Geography and Director of the Institute of Geography and Earth
Sciences @ the University of Wales Aberystwyth, 2004, AN INTRODUCTION TO POLITICAL GEOGRAPHY Space, place and
politics, http://118.97.161.124/perpus-
fkip/Perpustakaan/Geography/Geografi%20manusia/Pengantar%20Geografi%20Politik.pdf) MD
Others have criticised the booster school, rejecting the argument that globalisation is a new
phenomenon that is transforming contemporary political geographies. Hypercritics (Dicken et al., 1997)
or sceptics (Held et al., 1999) such as Hirst and Thompson (1996) use historical statistics of world flows of
trade, investment and labour in order to underplay the signicance of more recent global
patterns of trade. They claim that because levels of economic interdependence remain stable,
no significant changes have occurred since the nineteenth century and it is, therefore, wrong
to suggest that the contemporary period has witnessed the formation of a particularly
integrated economy. For Hirst and Thompson, globalisation is a myth, used as a politically
convenient rationale for practising neoliberal economic strategies. Furthermore, from
this perspective, the nation-state remains a controller of cross-border activity and it is misleading to suggest
that it has lost all meaning and signicance within the contemporary world.
AT: PERM

The geographical context of the affirmative is intrinsic to the affs advantages
geographical assumptions manifest themselves on all levels of public policy
complete rejection is key
Smith and Desbiens 99 (Neil Smith and Caroline Desbiens 1999. The International Critical
Geography Group: Forbidden Optimism? Environment and Planning: Society and Space 18, 379
382. - http://www.praxis-epress.org/CGR/6-Smith%26Desbiens.pdf BRW)
This is a political optimism quite different from that of contemporary globalization discourses. Establishment visions of
globalization generally herald the end of geography as an economic variable; theirs is a world
beyond geography rather than a world in which geography is again and again manifested in
new and changing forms. Local economies and cultures are to be bound into global networks in such
a way that particularisms can and should be overcome to facilitate the efficient flow of capital. Whether benign or
malignant, these particularisms are always subject to cultural, political, economic, and ultimately
even military flattening. Where geographical difference is increasingly catered in pre packaged
form, the message is that the spatial configurations of race and gender, sexuality and class,
nationality and religion are either irrelevant or inimical to global oneness, as are the social theory and
political activism that have brought them to the forefront. And yet the defense and assertion of local difference is at times equally
violent and reactionary: Serb genocide in the 1990s and the NATO response in Kosovo play out both sides of this dynamic.
Globalization is not beyond geography but is instead an intensely geographical project. Our
ambition for an International Critical Geography (ICG) is to express an alternative social dialectic of global and local, while
affirming the importance of scale in our attempts to connect and organize politically. The need and
desire to reach across separated contexts should not deflect our attention from the fact that geographical difference is
expressed at all levels, from the interpersonal to the institutional, from the national to the
international, and everywhere in between. If poststructuralism and identity politics have
highlighted and acted upon smaller geographies through which subjects are made and individual perspectives take
shape, our task is to develop a political practice that is rooted in thes e separate locations and yet
remains relational and wide ranging. We understand that geography determines the possibilities as
well as the limitations of an international critical movement; our aspirations are for a grounded approach to
political change, therefore our critical practice is also a self reflexive one. It addresses the worlds geographical
expression at its core, that is, in the various situated perspectives that we bring to our global, geographical ambition. Our purpose is
to develop new theor etical tools and revivify the political activism that makes such an ambition a reality.


AT: REALISM

Realism produces insecurity within states turning its own impacts
Booth 08(Ken Booth Pub Date: February 29, 2008 Theory of World Security (Cambridge Studies in International Relations) -
BRW)
This snapshot of the main ideas of realism provides the basis for understanding the framework within which traditional security
studies developed in the main centres in which the subject has been studied in the Anglo-American world, and in those regions
whose politics seem to have been scripted by realism (notably the Middle East and most of Asia). Although realism dominates the
academies, and its expertise is frequently a passport into the offices of power, its key ideas have attracted considerable criticism""
These can be summarised as follows: realism is not realistic (it does not provide an accurate picture of the
world); it is a misnomer (it is an ideology masquerading as a theory of knowledge); it is a static theory
(without a theory of change); it is reductive (it leaves out much of the picture); its methodology is
unsophisticated (it sacrifices richness for efficiency of explanation); it fails the test of practice (it does
not offer a reliable recipe book); its unspoken assumptions are regressive (it leaves no space for gender or class); its
agenda is narrow (it over-concentrates on the military dimension to the exclusion of other threats); its ethics
are hostile to the human interest (by placing the 'cold monster' of the state at the centre); and it is intellectually
rigid (its proponents have marginalised or silenced other approaches). If these criticisms are valid, then the hold of
the family of realism on world politics must be lessened in an era of growing complexity, confusion, and crisis. Many
certainties have been under challenge since the winding down of the Cold War; philosophical confidence in the West has been shaky
for even longer; the academic study of international relations has been bent by divisions over theory, agenda, and method since the
1980s; and globalisation is one of several processes that have undermined further the case for regarding 'the international' as an
autonomous realm of politics. At such a time it is foolhardy to attempt to shoehorn a unique and still-changing period of history into
a traditional and flawed theoretical category. The comprehensive 'symbolic order that realism represents, to use
Lacan's concept, continues to be accepted though it is out of sync with reality. It is a case of trying to
maintain a sort of sanity at the expense of sense. Realism is not calculated to deal with the
challenges faced by human society, globally, in this period of world-historical danger (about which I will say much more in
later chapters). Like the ideas that sustained feudalism towards the end of the middle ages, or the divine right of
kings b
y
the late eighteenth century, realism is rationally out of time. But politically it is not, for it is interested in power,
and power is interested in it. The world it helped build is being challenged, but its symbolic order could hang on for the foreseeable
future; it is not foretold, moreover, that it will be followed by a more rational alternative. Like the other ideas that made us,
realism has been pregnant with its opposite. It is supposed to produce security, but it generates
insecurity externally and combines with statism to legitimise insecurity internally. Global
business-as-usual, in which realism is king, is a recipe for predictable global turmoil. 'All that is solid
melts into air, observed Marx.
8
' Today, the problem is that the glaciers are melting too quickly, and the ideas that constructed world
politics not quickly enough.


AT: DETERRITORIALIZING


Our view is not deterritorialization the geographic gulag of the American
military state ensures a Global South present everywhere
Sparke, no date professor of geography at the Jackson School of International Studies (Matthew Sparke, no date [after
2006], Everywhere But Always Somewhere: Critical Geographies of the Global South,
http://faculty.washington.edu/sparke/Everywhere.pdf)//CC
While the consolidation ofthe wealth and privilege of the transnational capitalist
class has grown in tandem with the dispossession of the Global South (see Harvey, 2005), this elite
classremains a tiny fraction of the global populace, no doubt even smaller than the 0.1% noted by Shiva.
Nevertheless, it does exist, and moving from elite golf courses to elite airport loungesto elite private jets, it does enjoy a
certain cocooned geography that, as Shiva suggests vis--vis Friedman, helps bring flatnessinto view. Given
this very particular and privileged geography, might it not be mistaken to assert asI did at the start
here that the Global South is everywhere? Does not thisrisk reduplicating the
groundlessimagery of a global Multitude existing everywhere acrossthe smooth space of Empire?
To be sure, we can point to the co-presence of golf course gardeners, airport cleaners and even jet-booking office workersin
outsourcing centerssuch as Bangalore,but no one from the Global South, it mightstill be protested, actually getsto
travel the world from the deterritorializing heights of corporate jets. Yet here, I would submit,
the formulation of a Global South that is everywhere but alwayssomewhere still remains useful. In
theirsuccessful bid to create an unaccountable global gulag the American military andCIA have
used the hidden hand ofthe market to help with their hidden handcuffing of people targeted
forimprisonment and torture without trial(Amnesty International, 2006). As a result, victims ofso-called
extraordinary rendition have been transported by private corporate jets all overthe planet, and
those who have lived to tell theirstories of kidnapping and torture have also highlighted the
cruel ironies of being manacled en route to the leather upholstery of a corporate jet(Mayer, 2005;
Sparke, 2006). In contrast, then, to the corporate cosmopolitanism of Friedman and hisfans, extraordinary rendition
therefore presents us with the carceral cosmopolitanism ofthe Global South: everywhere, but
alwayssomewhere,struggling to take back and sustain human geographiesin spaces of inhuman
violence and dispossession.


AT: ACCURATE
Accuracy is NOT the question the affirmative overmaps the Global South to
legitimate their project of dispossession and colonial violence
Sparke, no date professor of geography at the Jackson School of International Studies (Matthew Sparke, no date [after
2006], Everywhere But Always Somewhere: Critical Geographies of the Global South,
http://faculty.washington.edu/sparke/Everywhere.pdf)//CC
The Global South is everywhere, but it is also alwayssomewhere, and that somewhere, located at the
intersection of entangled political geographies of dispossession and repossession, hasto be mapped with persistent
geographical responsibility. Following GayatriChakravorty Spivakstracings of the shadow of a geographical pattern in
deconstruction (Spivak, 1976: lxxxii), I have elsewhere outlined such geographical responsibility in terms of a call to track critically
and persistently the openended graphing ofthe geo (Sparke, 2005). But what doesit mean to track the geo-graphy of
the Global South in this persistently critical way? Firstly it means critiquing the mapsthat have routinely
represented the heterogeneousspaces of the Global South in the interests of colonial and
neocolonial control. These maps of control include all the instrumental cartographies, moral
topographies, and imaginative geographiesthat have provided the practical guides and
promotional propsfor colonial practice from the times of Terra Nullius, the Orient and the Heart of Darkness to
such contemporary conceits asthe Axis of Evil, the Clash of Civilizations, and, that gleeful globalist gloss on contemporary
capitalism, the Level Playing Field. All these geographical visions have in different ways overwritten and
obscured the huge heterogeneity of the Global South, replacing it with perniciousNewWorld declarations, Us-
Them oppositions and West is Best assumptions. The arrogance ofsuch over-mapping issimilar in its overreach to
the Map ofthe Empirein the famousstory by Jorge Luis Borges, a story ofimperial cartographers who aspired to map
everything in such detail that their map actually grew to the size ofthe Empire and covered up the land (Borges, 1998). But while
thisstory of over-mapping was used by Borgesto ridicule the futility ofrepresentational totalization, and while he depictsthe Map of
the Empire being therefore abandoned as uselessin the Deserts ofthe West, the maps of control covering up the
Global South have by contrast actually been very useful to their cartographers. They have served
practically to remake the world anew, justifying imperial violence all over the planet and
underpinning political-economic practicesthat have ranged from the trans-Atlantic trading of
Africans, cotton and sugar to the contemporary Highly Indebted PoorCountrysurveillance
protocols deployed by the World Bank and IMF to limit the reach of debtrelief. In all of these diverse
contextsthe signature gesture of the maps of dominance has been simple: divide and dispossess.
Whether legitimating inhuman violence by liberalsin colonialIndia (Metha, 1999), or inhuman violence by
neoliberalsin neocolonialIraq (Gregory, 2004;Roberts et al, 2003), mapsthat map the Global South as a space
of exception outside the bounds of humanity and human rights have remained key to the ideological
legitimation and military organization ofimperialrule. The maps might be richly detailed and
realist in the old imperial and Cold War area studiestraditions(that often mixed ethnographic care with colonial interest),
orthey might be highly fictional and idealist in the neo-imperialIraq War tradition (that has mixed militaristic spin
with crass commercial interest). But, inwhatever way the over-mappings have overreached,
theirterritorializing imperatives have always worked in the interests of dispossession.


AFF

ROMANTIFICATION TURN

The radical politics of the alt is a romantification of resistance - dooming alt
solvency to mental masturbation
Sparke 08 (Matthew Sparke published may of 2008 Political
geography political geographies of globalization III: resistance
* Department of Geography, Box 353550, University of
Washington, Seattle, WA 98105, USA
http://search.proquest.com.proxy.lib.umich.edu/pqrl/docview/
230709084/13EE75FEC2A14F99E92/1?accountid=14667 BRW)
From the broad themes of neocolonial dominance and neoliberal governance that formed the foci of my two previous reports
(Sparke, 2004; 2006), this review of recent work on the political geographies of globalization turns to the still
wider and more contested terrain of resistance. Whereas the neo in both neocolonial and neoliberal invited
reflections on questions of historical change and continuity, there is no obvious neologism qualifying the various
forms of resistance that have emerged in relation to contemporary globalization. This is a telling irony.
Critical claims about the forms of dominance and governance associated with global integration
are frequently met with complaining calls to complicate the critiques with attention to
resistance. Yet in the rush to refer to the r-word the category itself is too often left uncomplicated. Questions about the
significance of resistant agency, its geo-historical reach, limits, conditions, organization and
impact are all often unanswered at the very same time as the rhetoric of resistance obscures the objects against which
resistant agents are said to resist. While the basic idea of resistance rests on notions of people pushing back,
the allure of the r-word itself can in this way ironically become a regulative pull that disciplines
critics: a pull, in other words, away from examining the messy middle grounds where control and opposition, structure and
agency, hegemony and counter-hegemonic action, are all variously mediated. This problem of pull can be usefully
ascribed to the romance of resistance. It is a romance that is initiated by assumptions about
autonomous action and animated by diverse forms of idealism; a romance that ultimately
imagines agency in the existential and ageographical terms of some seminal and heroically
universalized human spirit, and thus a romance that also tends to pre-empt empirical research
with metaphorical moves that make descriptions of socio-economic forces, racial and sexual
subjectifi cation, or even just everyday life seem somehow beside the point (for a related queer critique
of the romance of community, see Joseph, 2002).

FRAGMENTATION TURN

Localization of geographies makes fragments views of situations makes correct
interpretation impossible
Johnston, 07 PhD, University of Bristol, UK (R.J. Johnston, 24 Feb 2007 Australian geography seen from afar: through a
glass darkly Australian Geographer, 28:1, 29-37)
The argument advanced here represents the currently divided academic world. Because most of us
work in very small fragments of that world, and neither our teaching nor our research encourages
wide exploration beyond the confines of our particular interests and their borderlands with the relevant
fragments of other disciplines, we do not see large pictures . To most of us there is no such thing as
geography, other than as a vaguely defined discipline to which we are attached as much for
political and economic (that is, job security) reasons as for intellectual ones. We have no overview, no
appreciation in any detail of what is being done in 'our discipline' outside 'my fragment'. Not surprisingly, therefore, few
geographers working outside Australia have a clear, coherent view of Australian geographers' current
concerns. And there is nothing peculiarly Australian about that situation: it applies everywhere, even within British
geography and between the 'big two'British and North American geography (as illustrated by Stoddart's 1996 comments on
Richards and Wrigley 1996).

ALT TAKEOUTS

Critical geography has been institutionalized your alternative is no longer a
radical shift from status quo politics
Blomley 07 (Nicholas Blomley (2007) Critical geography: anger and hope * Department of
Geography, Simon Fraser University, Burnaby, BC V5A 1S6, Canada Progress in Human
Geography 31(1) pp BRW)
Critical geography has also made it, becoming deeply entrenched within the academy. At the same
time, many share Clokes (2002: 588) frustration with our apparent inability to retain a critical political edge in human geography.
For Baeten (2002), urban geography (his focus) is losing its political sting. It has been institutionalized, streamlined
and finetuned . . . it now fails to crystallize in a convincing political project that would provide a
credible alternative for the poverty generating capitalist shaping of todays city (p. 148). Some point to
structural factors by way of explanation. Sayer (2000) identifies several systemic changes at play, such as the collapse of traditional
class politics and the rise of political movements around feminism, ecology, sexuality and the increasingly multicultural nature
of many societies, all of which have foregrounded complex ethical and political issues that went beyond the traditional class and
distributional agendas of the old Left. Others point to the absorptive capacities of the dominant society: the critic is supposed to
be a maverick and outsider who challenges friends and enemies alike; he [sic] is self sentenced to
intellectual and political solitude(Walzer, 1988: 12). Yet liberal culture absorbs criticism, finds it interesting, even
titillating . . . The angry and alienated social critic bangs his head against a rubber wall. He
encounters infinite tolerance when what he would like is the respect of resistance (p. 16). Castree
(2000) notes the professionalization of the Anglo-American geographic left through dynamics of tenure, departmental and
disciplinary socialization, the monopolization of knowledge and accreditation. As a result, yesterdays untenured
radicals are todays critical professors, fully integrated into the day-to-day structure of the
tertiary sector (p. 961). He notes that this professionalization has its down side (notably in fostering a detached
academicism), but refuses to characterize this as a sellout, noting that the arrival of the geographical Left has
generated very real material benefits in terms of teaching, research, employment and publishing. As someone who remembers
the institutionalized suspicion toward critical scholarship of the late 1980s, I take his point. Yet others view such
institutionalization in a more jaundiced light. Euan Hague (2001) compares the first, self-confidently radical issues
of Antipode with its later manifestation as Antipode, Inc., lamenting the loss of the irreverence, optimism and creativity of earlier
years of radical geography (cf. Waterstone, 2002).3 On taking over the editorship of Antipode, Peck and Wills (2000) noted the
changing landscape for radical geography, but concluded in more pragmatic terms: Being radical in the late 1960s involved a very
different cluster of beliefs, ideas and affiliations than might be expected today. They note: The optimism, excitement,
and audacity of those times have been eroded, and radical geographers now often have more modest
ambitions and expectations of change (p. 2).




Institutional isolation proves that Critical geography cannot be applied to policy
solutions the universalistic claims of the alt play with fire
Blomley 07 (Nicholas Blomley (2007) Critical geography: anger and hope * Department of
Geography, Simon Fraser University, Burnaby, BC V5A 1S6, Canada Progress in Human
Geography 31(1) pp BRW)
Yet Castree and Wright (2005), when they became editors, sounded a more affirmative note. Noting that effective opposition to
powers multifarious operations seems to us notable for its paucity rather than its profusion (p. 1), and recognizing the irony and
paradox that the journal burgeons at a time when, outside the academy, Left-wing ideas are
marginal, at best, they still refuse a glum defeatism (p. 7). They argue that the very incorporation of
critical scholarship within the academy, while not without its attendant costs, creates a space for the left,
protected by academic freedoms, to create new knowledge about the world that might make a
difference: Just because we cannot be certain how much what we say matters beyond the precincts of the university, we should not
assume that our analyses and ideas count for nothing at all. Just because radical scholarship has now become a recognized
currency in academic promotions does not mean it is nothing more than this (p. 6). This seems, to me, to be a point worth
underscoring. While academics are only one group of meaning entrepreneurs (Baron, 2002), they are institutionally well placed.
We must guard against a messianic and elitist view of the power of the academic to effect social change. Yet we can also recognize the
transformative power of ideas, particularly when well written (Mitchell, 2006).4 Some attribute this deradicalization to the practice
and epistemology of critical geography. Purcell (2003) laments the institutional divisions among critical
geographers, the effect of which is to produce islands of practice, separate from one another: scholars
whose main focus is capitalism are too often intellectually and politically isolated
from scholars of patriarchy, racism or heternormativity, he laments. Similarly, those who
investigate more global scale relations too rarely collaborate with scholars of local-scale processes (p. 319). Purcell calls for a more
synthetic critical geography, predicated on collaboration across research traditions, allowing for the integration of diverse theoretical
frames and analytical scales. Conversely, other observers worry that critical geography has simply become
too diffuse and open-ended (Urtibe-Ortega, 1998). In her survey of critical geographers, Wendy Gibbons
(2001) reports that many respondents saw the critical label as too diffuse and inclusive, preferring radical as a personal badge. For
others, theory is to blame, some complaining at the dangers of overtheorization. As Gregory (2004: 249) reminds us:
The world does not exist in order to provide illustrations of our theories. Others lament the Jesuitical
tendencies of contemporary social theory. Waterstone (2002: 663) rhetorically asks whether it is really necessary (and for whom or
for what purposes) that we specify more carefully the exact articulations between Lefebvre and Nietzsche and Heidegger? (p. 663).
Peet complains against obscure topics dressed in weird philosophical clothing (2000: 952). The subtitle of Storpers (2001) paper
from the false promises of Marxism to the mirage of the cultural turn suggests that he is taking few prisoners in his critique of
the poverty of radical theory, although he concludes by suggesting something of a rapprochement between cultural-turn
and political-economy radicals.5 Certainly, the rejection of the certainties of modernist theory where, in the last analysis,
all can be attributed to an essential structural logic, has led to political drift within critical
geography. Sayer (2007), however, provides a constructive response to the antiessentialist critique. While we should reject
determinism, he argues, and ideas that phenomena have fixed and invariant essences, it is the case that what any person or object
can do is constrained and enabled we make our history, but not under conditions of our own choosing. Anti-essentialism, if it
ignores this, is in danger of losing its critical purchase (cf. Eagleton, 2006). While there are clear dangers in
inappropriate universalistic claims, Sayer notes, we also need to identify some common
human capacities for flourishing and suffering if we are to do critical social science. To
argue that such capacities are always culturally relative or socially constructed is to forgo the use of ethical categories such as justice
and oppression. There is urgency to such arguments. Justice seems in short supply, in a world of sharpening and almost
incomprehensible suffering. Yet given that most of us remain unastonished by the fact that famines in the South are as
routine as they are preventable; that Bill Gates earns more each hour than all the workers in Liberia do in a week; that
sexism is rampant, despite the advances made by feminism; or that murderous
discrimination remains so common worldwide as to seem a natural part of the
human condition, the Left has failed (Castree and Wright, 2005: 1): todays radical
geographers, perhaps, have few objective reasons to be optimistic (p. 4).







Critical Geography fails to offer a concrete blueprint for change the alternative is
all talk and no walk recreating violent failures of the system
Blomley 07 (Nicholas Blomley (2007) Critical geography: anger and hope * Department of Geography, Simon Fraser
University, Burnaby, BC V5A 1S6, Canada Progress in Human Geography 31(1) pp BRW)
Amin and Thrifts argument, for Smith, represents a manifesto for a neo-critical geography that fits us all comfortably within the
fold of a supposedly ethical Blairite capitalism (p. 898). Their rejection of hierarchy and embrace of a flatter world is, for Smith,
too close to a liberal logic of pluralism: It would indeed be nice if the world were flat and non-hierarchical. Many of us have long
been struggling for just such a result . . . But it is precisely the self-serving trick of neo-liberalism to assume
that such a flat world is already there, hierarchy is gone, equality rules. The world may be flat for those
who can afford a business class ticket to fly around it, gazing down on a seemingly flat surface . . . For those in Bombay shanties . . .
the price of the same business class ticket to see the world as flat is . . . prohibitive . . . Insofar as neo-critical geographers
see no hierarchy, then, they can show us no location of power that needs to be talked back to, challenged, or transformed (N.
Smith, 2005: 894). Despite their rejection of a gate-keeper politics, Smith condemns Amin and Thrifts cursory dismissal of
Marxism, and lambasts their us and them division between an hierarchical and heterarchical Left as an intellectual and
disciplinary embarrassment (p. 897). Despite their appeal for no more policing,6 Amin and Thrift do precisely that, Smith argues.
III Utopia Smith notes that we cannot stop at critique: we need a sense of how to put things together
even in the insistent continuance of critique . . . Eyes on the prize (N. Smith, 2005: 89899). How to win the
prize is one question. The exact nature of the prize, however, remains more elusive. For Oscar Wilde, any
map that did not have utopia on it was not worth looking at. Yet utopianism is regarded in many quarters with a good deal of
suspicion. And with good reason: unitary totalizing blueprints have too often proved
disastrous. Yet the utopian impulse remains omnipresent, and is no exclusive domain of the left. Capitalism,
Ollman (2005) notes, is adept at turning human dreams and aspirations into lotteries and sporting events. For Harvey (2000)
neoliberalism is a deeply utopian and teleological project, premised on process (individual liberties, realized through the market)
and risk-taking. Other domains of science rely upon explorations of the imaginary (Baeten, 2002): sustainability, for example, is a
deeply utopian concept. In the mid-1970s, Zygmunt Bauman (1976) described socialism as intrinsically utopian. Contemporary
critical geographers, however, are better at mapping current dystopias than
imagining utopic alternatives. The demise of utopian thinking, however, can have debilitating
effects, being symptomatic of a closing down of the imaginative horizons of critical thinking and
even a slide into a reactionary acquiescence to dominant understandings and representations . . .
and to the injustices of existing conditions (Pinder, 2002: 237). Gerry Pratt (2004) cites Meghan Morris, who worries that the
tendency of critical scholars to retell the same old story of capitalism, racism and
patriarchy, can create the impression that nothing has, or can ever be otherwise.
The ethical, utopian, political impulse of feminism, Morris argues, is the belief that things the systematic
production of social difference can and must be changed. Feminist theory is a limited resource if it lacks the subtlety not only to
diagnose the specificity of this production, but the vitality to animate social change (p. 9).7 For Barnes (2001) critique should be
directed from a sense of what a better world would be like (p. 12). Despite the battering that leftist utopian thinking has received, it
has not been entirely abandoned in critical geography. The authoritarianism of high modernist utopianism, some note, can be
abandoned without giving up the dream of better worlds. For Sayer (2000), utopias need not signal either the unattainable or an
authoritarian blueprint, but can be thought of as a thought experiment in living otherwise. From a critical realist perspective, this
does not amount to prediction or projection, given the contingency of social conditions; what can reasonably be requested is that we
explore as far as possible what the causal powers and liabilities of alternative forms of social organization are likely to be (p. 162).
He cautions against extending principles appropriate to one social sphere to another, and urges careful judgement in thinking
through normative standpoints. (Does it privilege a particular group? Is it feasible and desirable?) For Harvey (2000: 17), [t]he
inability to find an optimism of the intellect [rephrasing Gramsci] with which to work through alternatives has become one of the
most serious barriers to progressive politics. He seeks a dialectical utopianism that is spatiotemporal, drawing from the existent
internal contradictions of capitalist society. Harvey aims to avoid either the constricted utopias of spatial form that treat space as a
mere container for social action, or a purely processual utopia that evades closure. He embraces a dialectic that mediates between
form and process, rooted in current realities yet pointing toward liberatory possibilities, identifying several interlinked theatres of
insurgency,8 none of which is uniquely privileged, in which human beings can think and act, though in radically different ways, as
architects of their individual and collective fates (p. 234). The emphasis, as noted here, is on the transformative power of the
insurgent architect, rather than on vanguardism and a radical revolutionary break. This, he notes wryly, is a long revolution (p.
238). The utopian possibilities of the city has also inspired critical geographers (Lees, 2004; Pile, 2005).
Pinder (2002) identifies several strands of urbanism utopian thinking that coalesce around expressions of
desire for a better way of being and living. Desire, for Pinder, works utopically in revealing the gap between present
conditions and desired alternatives. Rather than a blueprint utopia, this transgressive utopianism found,
for example, in the writings of Lefebvre and the situationists (cf. Merrifield, 2006), or more recently in Leonie Sandercocks call for
cosmopolis is resistant to closure and always in process, Pinder argues. As expressed in the writings of Iris
Young, for example, it is fully conscious of the divide between that which is desired and the world as it exists, yet thinks of the former
as an already incipient yet unrealized possibility, latent within current realities. Loretta Lees (2004) identifies a number of utopian
strands within urban theory, such as scholarship that finds a form of emancipatory alienation in the shock of urban experience, or
Benjamins transgressive readings of consumer culture. Contributors to her edited volume seek utopic and emancipatory
geographies in many urban places, including film, planning and sex. Brown (2004), for example, brings together gay cruising and
the Situationist-inspired Reclaim the streets movement, to suggest that both foster new forms of homoerotic communality that can
potentially contribute to a re-evaluation of meaningful human interaction and community formation (p. 92). Such work also alerts
us to the need to take seriously the remarkably creative work of critical geographers outside the academy, actively engaged in a
spatialized politics of anger and hope in the many sites of civil society. The recent exploration of the right to the city is also utopian,
insofar as it invites us to imagine a city structured according to more democratic and inclusive forms of copresence and possibility.
Yet, to the extent that such accounts draw from Lefebvres grounded theorizations of space and the moment (Merrifield, 2006) they
depart from blue-sky utopianism. Utopia can be built, some geographers suggest, with the masters tools: the most effective political
moments, for Warren (2004), take recognizable moments of the current world and refashion them in innovative sometimes
shocking ways in order to transcend the complacency of the status quo (p. 10). She seizes upon the utopian potential of GIS,
rejecting the technologically essentialist criticism of the technology as innately compromised, arguing that it can be understood as
part of the longer trajectory of peoples struggles with and against the machine within industrial capitalism (p. 5). The where of
utopia varies: despite its name, utopia has a geography. As noted, many privilege the city as the crucial site. Others are much more
localized: Pratt (2004) finds a utopian vitality, refreshingly, in the good company of those who have committed their daily life to
social change (p. 9): the activists of the Philippine Womens Centre whose work she illuminates through feminist theory. This
localization echoes Saffords (2004) moving evocation of the places of hope, the little plots of ground (the congregation, classrooms,
streets, factories) from which activism and optimism emerge. Others turn to the nation: effective criticism, for Walzer (1988), is
rooted in a shared national discourse, rather than an appeal to class. Similarly, Unger and Wests (1998) utopic proposals are
squarely American, resting on a radical extension of a national religion of possibility: America this monument to the genius of
ordinary men and women, this place where hope becomes capacity, this long halting, turn of the no into the yes needs citizens who
love it enough to reimagine and remake it (p. 93), they claim. A global utopia is evoked in the statement of purpose (Smith and
Desbiens, 1999) of the International Critical Geography group, a loose network of like-minded geographers from Europe, Asia
and North America. The manifestos title, A world to win, has a triple meaning: It expresses our political ambition in
geographical terms; it indicates the global breadth of that ambition; and it makes clear that changing the
world requires a lot of work but that victory is there for the winning. Internationalism is embraced because we believe that for too
long it has been possible to divide people with similar interests on the basis of national difference and because the social systems
and assumptions of exploitation and oppression, as expressed in the celebration of globalization, are international. To be a critical
scholar, it is claimed, means, in part, to demand and fight for social change aimed at dismantling
prevalent systems of capitalist exploitation; oppression on the basis of gender,
race and sexual preference; imperialism, national chauvinism, environmental
destruction. Critical scholarship refuses the selfimposed isolation of much academic research, believing that social science
belongs to the people and not the increasingly corporate universities and embraces existing social movements outside the academy
aimed at social change. Programmatically [w]e are critical because we seek to build an alternative kind of society which exalts social
differences while disconnecting the economic and social prospects of individuals and groups from such difference. Granted, such
a utopian vision is sketchy at best, but then, manifestoes often are. Yet they can still be inspirational and world-
making (Rorty, 1999: 201209). There is, for me, something refreshing in the ICGs affirmative and optimistic zeal. For too long,
utopia has been the exclusive domain of a neoliberal capitalist ascendancy. It is time to recover (or, more accurately, acknowledge
the already existing) utopian impulse at the centre of critical scholarship.



Lack of a specific alt takes out solvency
Blomley 07 (Nicholas Blomley - Department of Geography, Simon Fraser University, 2007, Critical geography: anger and
hope, http://walk2geographies.files.wordpress.com/2009/03/critical-geography_anger-and-
hope_blomley2007progressinhumangeography1.pdf) MD
In the mid-1970s, Zygmunt Bauman (1976) described socialism as intrinsically utopian. Contemporary critical geographers,
however, are better at mapping current dystopias than imagining utopic alternatives. The demise
of utopian thinking, however, can have debilitating effects , being symptomatic of a closing
down of the imaginative horizons of critical thinking and even a slide into a reactionary acquiescence to dominant
understandings and representations . . . and to the injustices of existing conditions (Pinder, 2002: 237). Gerry Pratt (2004) cites
Meghan Morris, who worries that the tendency of critical scholars to retell the same old story of
capitalism, racism and patriarchy, can create the impression that nothing has, or can ever be otherwise.
The ethical, utopian, political impulse of feminism, Morris argues, is the belief that things the systematic production of social
difference can and must be changed. Feminist theory is a limited resource if it lacks the subtlety not only to
diagnose the specicity of this production, but the vitality to animate social change (p. 9).7 For
Barnes (2001) critique should be directed from a sense of what a better world would be like (p. 12).





Critical geography fails understanding of the global order are too soft and
alternatives have become overgeneralized
Olson, 9 Institute of Geography, University of Edinburgh (Elizabeth Olson, 2009, Radical Geography and its Critical
Standpoints: Embracing the Normative Antipode, Vol. 41 No. 1, pg 180198)//ah
In this symposium, we have invited a selection of leading geographers to give their own views on the question of the normative
foundations of radical geography.1 What do we mean by this? Radical (or what is now better known as critical) geography
claims to be critical, not merely of other approaches to geography, but of the very practices and
social arrangements that it studiesbe it urbanisation, migration, development, colonialism or
whatever. However, like other critical social science, radical geography has either become increasingly
reticent about making its critiques and their standpoints or rationales explicit, or has softened its critiques,
so that in some quarters being critical has reduced to trying merely to unsettle some ideas or to being
reflexive. Not surprisingly, Barry Barnes, the sociologist, has referred to the ever-so slightly critical theory of today (Barnes
1999:127). From a different perspective, Jane Wills (2006:907) argues that radical, critical and Left have been
largely stripped of their meanings, with all the things that seem progressive and anti-authoritarian now considered to
be left geography. We would like to build on this point and suggest that tensions over terminology derive from much deeper
uncertainties about how critique is understood in contemporary critical/radical geographies. Whatever term
one prefersand we prefer radical geography for reasons that we hope are apparent in our arguments belowour overriding
concern centres around the decline of critique. The present condition of non-critical critical geographies has its
foundations in the divorce of positive social science from normative political and moral philosophy with the
development of the modern academic division of labour. This divorce has been damaging for both sides, not only
institutionalising the factvalue dichotomy but allowing the social sciences to become
deskilled in understanding normativity, and philosophy to become overly abstracted from
concrete social practices , presenting a generally individualistic analysis of the social good,
ignoring the forms of social organisation within which people act. This situation cries out for
dialogue. The argument we develop here takes off from an earlier plea for a normative turn in geography (Sayer and Storper 1997).
Sayer and Storper discussed some of the reasons for the reluctance to be normative and advocated a turn to ethicsa call also made
by David Smith and others. Here we argue for a different but complementary ideathat normative critical thought needs some
conception of the human good or flourishing, and that this is not necessarily at odds with the descriptive and explanatory aims of
social science. We are well aware that this will sound strange to many, for in modern thought (particularly liberalism) the very idea
that a conception of what is good or bad could be something that could be rationally defended has
come to seem strange and has been widely rejected. Are not ideas about good and bad just subjective or
dependent on ones culture? Radical geographers regularly use terms like oppressive, racist or
exploitative in their descriptions of social practices, but generally without saying why these things are
bad. Do they just subjectively happen not to like them? Or do they object only because they transgress local norms? Both those
reasons are feeble, and if confronted by such practices in daily life, they would not invoke them. Rather they would argue that they
cause harm or restrict peoples flourishing. However, for various reasons, most of critical social science, including radical geography,
tends to avoid such arguments.



CEDE THE POLITICAL

The k cedes the political and ignores oppression
Chouinard, 94 Professor, School of Geography & Earth Sciences (Vera Chouinard, 1994, Environment and Planning D:
Society and Space 2-6. 1)/ah
Clearly, then, one of the dangers of reinventing ourselves in postmodern ways is that we will be
seduced by representations of radical research which distort past work and are relatively
empty of substantive proposals for building progressive and transformative geographies (see also
Harvey, 1992). In the process we are likely to jettison prematurely the many valuable legacies of the New Left,
including a clear political understanding that our projects must be deliberately and self-reflexively
constructed to connect with struggles against oppression and exploitation. McDowell (1992) makes the
related and important point that the adoption of new textual and interpretive strategies, without greater engagement with
radical traditions like feminism, risks creating academic approaches which are elitist,
closed, and divorced from efforts to confront and change the politics of science .
Ironically enough, there is often a marked disjuncture between representations of interpretive and
poststructuralist approaches as progressive, and their actual political substance. Indeed there is sobering evidence
that the interpretive turn is in many instances a detour around and retreat from political
engagement in struggles outside the academy. Palmer (1990), reviewing developments in social theory and in social history,
observes that the adoption of poststructuralist and postmodern approaches by eminent scholars on the Left
has been closely tied to a retreat from politics. Fraser (1989), examining the work of the French Derrideans,
demonstrates that the interpretive or postmodern turn has been associated with an extremely confused treatment of political
questions and decreased emphasis on the politics of academic work. Closer to home, in geography, I have been struck by how
seldom we discuss, in print or at conferences, the implications of our reinvented approaches for the politics of
academic work. And yet surely it is precisely during a period of major revision and reconstruction of
our approaches that we most need to discuss political matters. That is unless, of course, part of the hidden or perhaps
not fully recognized agenda of at least some postmodern shifts is the jettisoning of radical political project


The alt fails critical geography isolates meathods of change within the university
without extrapolation onto society this generates a hierarchical left and links to
all of our cede the political arguments
Blomley 07 (Nicholas Blomley (2007) Critical geography: anger and hope * Department of
Geography, Simon Fraser University, Burnaby, BC V5A 1S6, Canada Progress in Human
Geography 31(1) pp BRW)
Yet it is also the case that our history, as Howard Zinn (2004) reminds us, has been one not only of cruelty, but of sacrifice, courage
and compassion: There is a tendency to think that what we see in the present moment will continue. We forget how often . . . we
have been astonished by the sudden crumbling of institutions, by extraordinary changes in peoples thoughts, by unexpected
eruptions of rebellion against tyrannies, by the quick collapse of systems of power that seemed invincible (p. 70). II Critical? My
dictionary defines critical as censorious, fault-finding; pertaining to a crisis, involving risk or
suspense; and marking a transition from one state to another. As I noted in an earlier review (Blomley,
2006b), critical geographers are adept at the first task, but seem more reluctant to take
risks or imagine transitions. In this review, which follows my preliminary attempt at outlining the lineaments of
critical geography (Blomley, 2006b), I try to take the latter task more seriously. Social criticism, for Walzer (1988), entails at least
two tasks: to question the platitudes and myths of a society, and to express the aspirations of a people. Quoting Breytenbach, the
critic holds his [sic] words up to us like mirrors. The view of society that is offered is an unstinting and unsettling one, stripping the
veil from the reality: The critic looks first and then he forces the rest of us to look (p. 231). Yet, Walzer argues, in mirroring the
world, the critic reveals the disjuncture between an ideal and an actual world: [t]he point of holding up the mirror is
to demonstrate that the ideal world is not here, or were not there. The stories that we tell
ourselves about the realization of freedom and equality are untrue: one has only to look in the
glass and see (p. 231). The three tasks of the social critic, for Walzer, are to expose the deceits and illusions of his or her society,
to give expression to a peoples sense of how they ought to live, and to insist that there are other
forms of falseness and other, equally legitimate, hopes and aspirations (p. 232). Effective
criticism, for Walzer, must abandon the mountaintop, as well as the wish to command and direct. The
goal is to make the world visible, rather than making it over (cf. Fay, 1987). A number of critical geographers
have begun to rethink the normative and the political in more sustained and systematic ways (Wills, 2007). This has prompted a
number of recent conversations concerning the future of critical/left geography. In an opening salvo, Thrift and Amin (2005) refuse
to be nostalgic for some imagined past, when the Left spoke with one voice. They reject those who would give Left geography
marks for revolutionary content (presumably 7/10 in the 1960s but only 4/10 now), and see no crisis in Left geography
(p. 221). They welcome the Lefts current multiplicity of voices, arguing that this produces an unending, always-changing (p. 220)
political conversation without any ontological anchor. Rather than seeing the pluralism and diversity of critical geography as a
problem, they embrace it as a basis for an agonistic politics: our disagreements, they argue, can provide the basis for connection
(p. 222). Through a survey of the changing nature of political commitments and economic and social conditions, they identify a
series of emergent forms of political engagements that are sustained, they argue, by political commitments that, they believe,
provide a basis for a heterarchical Left. These include a democratic experimentalism, a transversal
politics, a refusal to privilege certain scales of activism, and a belief in the constitutive and productive power of disagreement. They
firmly reject both Marxism as a privileged pivot point (see Hudson, 2006, in response), and a gate-keeper politics that
seeks to regulate membership in the club of the Left. Yet they similarly reject a free-for-all Left
politics, insisting on the retention of certain values of the Left (notably, an optimistic engagement with politics; reflexivity; and a
necessary orientation to a critique of power and exploitation (p. 221). Amin and Thrifts optimistic intervention has, not
surprisingly, come under scrutiny. Watts (2005) also argues that a changing world requires a changing Left:
If the Left is to mean something politically . . . it needs fresh concepts, or, at the
least, old concepts reworked mercilessly in the light of the present (p. 651). He does not
offer an agenda, but a sketch of the brute realities to be confronted. While he cautiously accepts the need for a reflexive and self-
critical Left, as argued for by Amin and Thrift, he insists that some of the old certainties must surely remain, notably a commitment
to democracy, social justice, green politics, and internationalism, combined with a rejection of the solicitations of the market
and its commodification of everything. Unlike Amin and Thrift, Watts holds to a class-centred Left: it is only from
the irreducible centrality of class, exploitation and the contradictory reproduction of capitalism as a dynamic and changing system
that a sense of alternatives can emerge(p. 652). A Left without apology and guarantees from this side of the Atlantic seems to
me to now strike a great common accord he insists (p. 651). N. Smith (2005) is far more damning of Amin and Thrifts neo-
critical geography, as he terms it. He laments the co-optation of the Lefts best ideas by neo-liberalisms
ideological mulching machine. The radical upsurge in geography, he notes, is not exempt.


AT: SPILLOVER

Critical geography movements dont spill over
Fuller* and Kitchin**, 12 *Duncan Fuller Division of Geography, Lipman Building, Northumbria University and Rob
Kitchin**, Department of Geography and NIRSA, National University of Ireland Maynooth, (2012, Radical Theory/Critical Praxis:
Academic Geography Beyond the Academy? http://eprints.nuim.ie/3947/1/RK_ACME_-
_Geography_beyond_the_academy_2004.pdf)//ah
In this chapter we have sought to outline a historical context and provide a broad picture of contemporary debates
concerning the development and trajectory of radical/critical theory and praxis within geography. In
particular we have concentrated on examining the extent to which radical/critical geographies ideological
intent is presently being realised whether radical/critical geographies do make a difference beyond
the academy and detailing the structural threats to different forms of critical praxis. While we would
acknowledge that many academics do contribute to wider society in all kinds of ways, we would contend that it is often in roles
divorced from their research and praxis. While ideological rhetoric often eludes to academia seeking social,
political, environmental change, the mechanisms through which this change is to occur are often
conservative in nature, limited to teaching, writing academic articles, and occasionally policy
work ( that often reinforces the status quo rather than challenging it ). It is change sought
through a traditional academic role, which in itself reproduces notions of what it means to be an
academic. Within this context, the marriage of academic and activist often seems alien. The academic
theorises and suggests, but the move onto the streets or into the community as an academic/activist is
limited. This is not to suggest that no such forays occur, with perhaps the most sustained critical praxis beyond the academy
enacted by feminist geographers. But it is to suggest that the ideological intent of much radical/critical geography
is stifled, its potential unfilled and limited to the classroom and the pages of journals (not that
these are not worthy pursuits they are but that they are only two out of many possible
courses of action). The chapters that follow all engage with this theme how radical/critical geography can realise its
ideological potential; how radical theory can be translated into critical praxis in ways beyond teaching and writing. Taken together
they provide many useful insights into the role of radical/critical geographers both within the academy and beyond, the different
ways in which academics can seek to make a difference, and provide lessons based on their own forays at realising the ideological
intent of radical/critical geographies.

AT: CRITICAL GEO FIRST

An absolute focus on theory trades off productive engagement spatial analysis
shifts focus from oppression to academic theory
Fuller* and Kitchin**, 12 *Duncan Fuller Division of Geography, Lipman Building, Northumbria University and Rob
Kitchin**, Department of Geography and NIRSA, National University of Ireland Maynooth, (2012, Radical Theory/Critical Praxis:
Academic Geography Beyond the Academy? http://eprints.nuim.ie/3947/1/RK_ACME_-
_Geography_beyond_the_academy_2004.pdf)//ah
This focus on theory though holds potential dangers . In the chapter that follows, David Sibley
poignantly and critically reflects on his career as an academic through a psychoanalytical exploration of the madness of institutions.
Specifically, he strives to make sense of how his story of research and writing has entailed a shift from
ethnographyDuncan Fuller and Rob Kitchin 12 and involvement with excluded minorities (Gypsies in particular)
towards a (now more negatively perceived) increased concern with theory. Sibley notes that his initial
contact with Gypsies appeared to him to be quite distinct from an academic life that (at that time) was
devoted to obscure exercises in spatial analysi s. Moreover, any desire to fuse the two
together was truncated by a belief (now adjudged as arrogance) that few academics would be motivated
to get involved with such issues as a result of reading anything I might have written. Despite this, reservations were
(nervously) cast aside, and with one eye at least on the need for what he terms, academic legitimacy, Sibley embarked
on the process of striving to inform his experiences through drawing on good enough theory. The problem, as the remainder of the
chapter highlights, is that there is no such thing as good enough theory when theories have to be
continually produced as a necessary element in the academic accumulation process; the pressures to enter into the
theory production process, and the various rewards received as a result, meant that Sibley found himself unable to
resist a move from practical involvement to theoretical elaboration, shifting from people to
texts. Looking back, Sibley interprets this tendency as being inescapably intertwined with changes in the university system and
its increasing deference to market forces. Here he draws upon psychoanalysis to expose the madness of taken-for-granted everyday
practices within universities, and the processes of institutional change that have lead to the formation of strong boundaries and
hierarchies as a defence against environmental uncertainty disorder and chaos put crudely, the wrong type of research.
Through these processes of change, Sibley argues that universities are increasingly characterised by the vertical organisation of
activity, with power controlled at the top, and with a myriad of systems of surveillance, accountability and control being employed to
keep a check on deviance and resistance. As a result, long term involvement with communities has become
increasingly discouraged, penalised, offered lip-service, or just made plain near impossible as a result of what Sibley
describes as a kind of psychosis which accompanies the increasing insulation of academic institutions as they
focus increasingly on production and the creation of value, narrowly defined by the state and the market. As
Sibley concludes, Geography, like other increasingly insulated disciplines, becomes part of the
problem and the case for resistance becomes more compelling . Similarly, in Chapter Four,
Chris Wilbert and Teresa Hoskyns reflect upon the recent invocations of yet another apparent crisis in social and cultural
geography, and human geography more generally, a crisis borne out of the very real threat to the houses of knowledge enacted by
university restructuring programmes. They suggest that crisis may be compounded by a perceived lack of contact between the
inhabitants of these houses and the real world, but that, in particular, it may be seen to relate to the perceived irrelevance of human
geography to policy needs/wants at all scales, and the lack of, or nervousness surrounding critique in such work. Here they
contend that, despite suggestions that there has perhaps never been a more potentially fruitful time for geographers to get
involved in policy work, such work lacks a political cutting edge. For Wilbert and Hoskyns, this lack of cutting
edge relates to a disavowal that critique can be, indeed should be, a central aspect
of engaging in policy, or indeed any other work . What follows, by way of Adorno and his work on
legitimacy and critique, is a stinging attack on the limits of current relevant geographical enquiry. They argue that the
conjoining of theory and practice is all to often accidentally lost, that critique is all too often
constrained/dismissed in the face of the perceived need for it to be acceptable, constructiveAcademic
Geography Beyond the Academy? 13 and responsible (read cuddly, not too radical/dangerous, and neoliberalist-embracing), and
that relevant work is all too often equated with being legitimate, where legitimate dictates who is legitimately seen to be able to
engage seriously in critique, as well as what kinds of things can be legitimately critiqued and how. It would be all too easy (and
ironic) for such views to be dismissed as yet another attack on the call for more relevant public policy work, but the authors are
clear that they are not against policy focused geography per se. What they do object to, however, is the way much policy work
seemingly accepts the status quo in return for RAE ratings and research income, how supposedly participatory initiatives are all too
often undermined from their very beginning, and how radical alternatives become mired within notions of relevance that seem
narrow, exclusionary and morally judgmental without being reflexive about the situatedness of
such judgments.

Вам также может понравиться